Feat: DM pour les TST
continuous-integration/drone/push Build is passing Details

This commit is contained in:
Bertrand Benjamin 2020-10-15 22:15:28 +02:00
parent 3dade0b9bc
commit a8c0291023
45 changed files with 6091 additions and 0 deletions

View File

@ -0,0 +1,141 @@
\documentclass[a5paper,10pt]{article}
\usepackage{myXsim}
\usepackage{tasks}
% Title Page
\title{DM1 \hfill AIOUAZ Ahmed}
\tribe{TST}
\date{Toussain 2020}
\begin{document}
\maketitle
\begin{exercise}[subtitle={Fractions}]
Faire les calculs avec les fraction suivants
\begin{multicols}{3}
\begin{enumerate}
\item $A = \dfrac{- 5}{5} - \dfrac{- 2}{5}$
\item $B = \dfrac{7}{3} - \dfrac{8}{9}$
\item $C = \dfrac{- 4}{2} + \dfrac{2}{1}$
\item $D = \dfrac{6}{10} + 9$
\item $E = \dfrac{- 7}{5} \times \dfrac{- 6}{4}$
\item $F = \dfrac{- 10}{5} \times - 8$
\end{enumerate}
\end{multicols}
\end{exercise}
\begin{solution}
\begin{enumerate}
\item
\[
\dfrac{- 5}{5} - \dfrac{- 2}{5}=\dfrac{- 5}{5} + \dfrac{2}{5}=\dfrac{- 5 + 2}{5}=\dfrac{- 3}{5}
\]
\item
\[
\dfrac{7}{3} - \dfrac{8}{9}=\dfrac{7}{3} - \dfrac{8}{9}=\dfrac{7 \times 3}{3 \times 3} - \dfrac{8}{9}=\dfrac{21}{9} - \dfrac{8}{9}=\dfrac{21 - 8}{9}=\dfrac{21 - 8}{9}=\dfrac{13}{9}
\]
\item
\[
\dfrac{- 4}{2} + \dfrac{2}{1}=\dfrac{- 4}{2} + \dfrac{2 \times 2}{1 \times 2}=\dfrac{- 4}{2} + \dfrac{4}{2}=\dfrac{- 4 + 4}{2}=\dfrac{0}{2}
\]
\item
\[
\dfrac{6}{10} + 9=\dfrac{6}{10} + \dfrac{9}{1}=\dfrac{6}{10} + \dfrac{9 \times 10}{1 \times 10}=\dfrac{6}{10} + \dfrac{90}{10}=\dfrac{6 + 90}{10}=\dfrac{96}{10}
\]
\item
\[
\dfrac{- 7}{5} \times \dfrac{- 6}{4}=\dfrac{- 7 \times - 6}{5 \times 4}=\dfrac{42}{20}
\]
\item
\[
\dfrac{- 10}{5} \times - 8=\dfrac{- 10 \times - 8}{5}=\dfrac{80}{5}
\]
\end{enumerate}
\end{solution}
\begin{exercise}[subtitle={Développer réduire}]
Développer puis réduire les expressions suivantes
\begin{multicols}{2}
\begin{enumerate}
\item $A = (- 1x + 10)(- 7x + 10)$
\item $B = (8x - 1)(8x - 1)$
\item $C = (- 1x - 3)^{2}$
\item $D = - 7 + x(- 3x - 6)$
\item $E = 2x^{2} + x(3x - 9)$
\item $F = 4(x + 4)(x - 4)$
\end{enumerate}
\end{multicols}
\end{exercise}
\begin{solution}
\begin{enumerate}
\item
\begin{align*}
A &= (- 1x + 10)(- 7x + 10)\\&= - x \times - 7x - x \times 10 + 10 \times - 7x + 10 \times 10\\&= - 1 \times - 7 \times x^{1 + 1} + 10 \times - 1 \times x + 10 \times - 7 \times x + 100\\&= - 10x - 70x + 7x^{2} + 100\\&= (- 10 - 70) \times x + 7x^{2} + 100\\&= 7x^{2} - 80x + 100
\end{align*}
\item
\begin{align*}
B &= (8x - 1)(8x - 1)\\&= 8x \times 8x + 8x \times - 1 - 1 \times 8x - 1 \times - 1\\&= 8 \times 8 \times x^{1 + 1} - 1 \times 8 \times x - 1 \times 8 \times x + 1\\&= - 8x - 8x + 64x^{2} + 1\\&= (- 8 - 8) \times x + 64x^{2} + 1\\&= 64x^{2} - 16x + 1
\end{align*}
\item
\begin{align*}
C &= (- 1x - 3)^{2}\\&= (- x - 3)(- x - 3)\\&= - x \times - x - x \times - 3 - 3 \times - x - 3 \times - 3\\&= - 1 \times - 1 \times x^{1 + 1} - 3 \times - 1 \times x - 3 \times - 1 \times x + 9\\&= 3x + 3x + 1x^{2} + 9\\&= (3 + 3) \times x + x^{2} + 9\\&= x^{2} + 6x + 9
\end{align*}
\item
\begin{align*}
D &= - 7 + x(- 3x - 6)\\&= - 7 + x \times - 3x + x \times - 6\\&= - 3x^{2} - 6x - 7
\end{align*}
\item
\begin{align*}
E &= 2x^{2} + x(3x - 9)\\&= 2x^{2} + x \times 3x + x \times - 9\\&= 2x^{2} + 3x^{2} - 9x\\&= 2x^{2} + 3x^{2} - 9x\\&= (2 + 3) \times x^{2} - 9x\\&= 5x^{2} - 9x
\end{align*}
\item
\begin{align*}
F &= 4(x + 4)(x - 4)\\&= (4x + 4 \times 4)(x - 4)\\&= (4x + 16)(x - 4)\\&= 4x \times x + 4x \times - 4 + 16x + 16 \times - 4\\&= - 4 \times 4 \times x - 64 + 4x^{2} + 16x\\&= - 16x - 64 + 4x^{2} + 16x\\&= 4x^{2} - 16x + 16x - 64\\&= 4x^{2} + (- 16 + 16) \times x - 64\\&= 4x^{2} - 64 + 0x\\&= 4x^{2} - 64
\end{align*}
\end{enumerate}
\end{solution}
\begin{exercise}[subtitle={Étude de fonctions}]
Soit $f(x) = - 3x^{2} - 24x + 60$ une fonction définie sur $\R$.
\begin{enumerate}
\item Calculer les valeurs suivantes
\[
f(1) \qquad f(-2)
\]
\item Dériver la fonction $f$
\item Étudier le signe de $f'$ puis en déduire les variations de $f$.
\item Est-ce que $f$ admet un maximum? un minimum? Calculer sa valeur.
\end{enumerate}
\end{exercise}
\begin{solution}
\begin{enumerate}
\item On remplace $x$ par les valeurs demandées
\[
f(1) = - 3 \times 1^{2} - 24 \times 1 + 60=- 3 \times 1 - 24 + 60=- 3 + 36=33
\]
\[
f(-1) = - 3 \times - 1^{2} - 24 \times - 1 + 60=- 3 \times 1 + 24 + 60=- 3 + 84=81
\]
\item Pas de solutions automatiques.
\item Pas de solutions automatiques.
\end{enumerate}
\end{solution}
%\printsolutionstype{exercise}
\end{document}
%%% Local Variables:
%%% mode: latex
%%% TeX-master: "master"
%%% End:

View File

@ -0,0 +1,141 @@
\documentclass[a5paper,10pt]{article}
\usepackage{myXsim}
\usepackage{tasks}
% Title Page
\title{DM1 \hfill BAHBAH Zakaria}
\tribe{TST}
\date{Toussain 2020}
\begin{document}
\maketitle
\begin{exercise}[subtitle={Fractions}]
Faire les calculs avec les fraction suivants
\begin{multicols}{3}
\begin{enumerate}
\item $A = \dfrac{- 9}{7} - \dfrac{- 10}{7}$
\item $B = \dfrac{6}{6} - \dfrac{- 5}{24}$
\item $C = \dfrac{- 10}{5} + \dfrac{7}{4}$
\item $D = \dfrac{- 5}{9} - 10$
\item $E = \dfrac{6}{10} \times \dfrac{- 7}{9}$
\item $F = \dfrac{- 6}{4} \times 8$
\end{enumerate}
\end{multicols}
\end{exercise}
\begin{solution}
\begin{enumerate}
\item
\[
\dfrac{- 9}{7} - \dfrac{- 10}{7}=\dfrac{- 9}{7} + \dfrac{10}{7}=\dfrac{- 9 + 10}{7}=\dfrac{1}{7}
\]
\item
\[
\dfrac{6}{6} - \dfrac{- 5}{24}=\dfrac{6}{6} + \dfrac{5}{24}=\dfrac{6 \times 4}{6 \times 4} + \dfrac{5}{24}=\dfrac{24}{24} + \dfrac{5}{24}=\dfrac{24 + 5}{24}=\dfrac{29}{24}
\]
\item
\[
\dfrac{- 10}{5} + \dfrac{7}{4}=\dfrac{- 10 \times 4}{5 \times 4} + \dfrac{7 \times 5}{4 \times 5}=\dfrac{- 40}{20} + \dfrac{35}{20}=\dfrac{- 40 + 35}{20}=\dfrac{- 5}{20}
\]
\item
\[
\dfrac{- 5}{9} - 10=\dfrac{- 5}{9} + \dfrac{- 10}{1}=\dfrac{- 5}{9} + \dfrac{- 10 \times 9}{1 \times 9}=\dfrac{- 5}{9} + \dfrac{- 90}{9}=\dfrac{- 5 - 90}{9}=\dfrac{- 95}{9}
\]
\item
\[
\dfrac{6}{10} \times \dfrac{- 7}{9}=\dfrac{6 \times - 7}{10 \times 9}=\dfrac{- 42}{90}
\]
\item
\[
\dfrac{- 6}{4} \times 8=\dfrac{- 6 \times 8}{4}=\dfrac{- 48}{4}
\]
\end{enumerate}
\end{solution}
\begin{exercise}[subtitle={Développer réduire}]
Développer puis réduire les expressions suivantes
\begin{multicols}{2}
\begin{enumerate}
\item $A = (10x - 9)(- 4x - 9)$
\item $B = (1x - 9)(1x - 9)$
\item $C = (9x - 10)^{2}$
\item $D = - 1 + x(- 6x - 10)$
\item $E = - 10x^{2} + x(- 7x - 3)$
\item $F = 7(x - 7)(x + 2)$
\end{enumerate}
\end{multicols}
\end{exercise}
\begin{solution}
\begin{enumerate}
\item
\begin{align*}
A &= (10x - 9)(- 4x - 9)\\&= 10x \times - 4x + 10x \times - 9 - 9 \times - 4x - 9 \times - 9\\&= 10 \times - 4 \times x^{1 + 1} - 9 \times 10 \times x - 9 \times - 4 \times x + 81\\&= - 90x + 36x - 40x^{2} + 81\\&= (- 90 + 36) \times x - 40x^{2} + 81\\&= - 40x^{2} - 54x + 81
\end{align*}
\item
\begin{align*}
B &= (1x - 9)(1x - 9)\\&= x \times x + x \times - 9 - 9x - 9 \times - 9\\&= x^{2} + 81 + (- 9 - 9) \times x\\&= x^{2} - 18x + 81
\end{align*}
\item
\begin{align*}
C &= (9x - 10)^{2}\\&= (9x - 10)(9x - 10)\\&= 9x \times 9x + 9x \times - 10 - 10 \times 9x - 10 \times - 10\\&= 9 \times 9 \times x^{1 + 1} - 10 \times 9 \times x - 10 \times 9 \times x + 100\\&= - 90x - 90x + 81x^{2} + 100\\&= (- 90 - 90) \times x + 81x^{2} + 100\\&= 81x^{2} - 180x + 100
\end{align*}
\item
\begin{align*}
D &= - 1 + x(- 6x - 10)\\&= - 1 + x \times - 6x + x \times - 10\\&= - 6x^{2} - 10x - 1
\end{align*}
\item
\begin{align*}
E &= - 10x^{2} + x(- 7x - 3)\\&= - 10x^{2} + x \times - 7x + x \times - 3\\&= - 10x^{2} - 7x^{2} - 3x\\&= - 10x^{2} - 7x^{2} - 3x\\&= (- 10 - 7) \times x^{2} - 3x\\&= - 17x^{2} - 3x
\end{align*}
\item
\begin{align*}
F &= 7(x - 7)(x + 2)\\&= (7x + 7 \times - 7)(x + 2)\\&= (7x - 49)(x + 2)\\&= 7x \times x + 7x \times 2 - 49x - 49 \times 2\\&= 2 \times 7 \times x - 98 + 7x^{2} - 49x\\&= 14x - 98 + 7x^{2} - 49x\\&= 7x^{2} + 14x - 49x - 98\\&= 7x^{2} + (14 - 49) \times x - 98\\&= 7x^{2} - 35x - 98
\end{align*}
\end{enumerate}
\end{solution}
\begin{exercise}[subtitle={Étude de fonctions}]
Soit $f(x) = - 7x^{2} - 98x - 280$ une fonction définie sur $\R$.
\begin{enumerate}
\item Calculer les valeurs suivantes
\[
f(1) \qquad f(-2)
\]
\item Dériver la fonction $f$
\item Étudier le signe de $f'$ puis en déduire les variations de $f$.
\item Est-ce que $f$ admet un maximum? un minimum? Calculer sa valeur.
\end{enumerate}
\end{exercise}
\begin{solution}
\begin{enumerate}
\item On remplace $x$ par les valeurs demandées
\[
f(1) = - 7 \times 1^{2} - 98 \times 1 - 280=- 7 \times 1 - 98 - 280=- 7 - 378=- 385
\]
\[
f(-1) = - 7 \times - 1^{2} - 98 \times - 1 - 280=- 7 \times 1 + 98 - 280=- 7 - 182=- 189
\]
\item Pas de solutions automatiques.
\item Pas de solutions automatiques.
\end{enumerate}
\end{solution}
%\printsolutionstype{exercise}
\end{document}
%%% Local Variables:
%%% mode: latex
%%% TeX-master: "master"
%%% End:

View File

@ -0,0 +1,141 @@
\documentclass[a5paper,10pt]{article}
\usepackage{myXsim}
\usepackage{tasks}
% Title Page
\title{DM1 \hfill BALLOFFET Kenza}
\tribe{TST}
\date{Toussain 2020}
\begin{document}
\maketitle
\begin{exercise}[subtitle={Fractions}]
Faire les calculs avec les fraction suivants
\begin{multicols}{3}
\begin{enumerate}
\item $A = \dfrac{- 10}{3} - \dfrac{8}{3}$
\item $B = \dfrac{3}{3} - \dfrac{2}{15}$
\item $C = \dfrac{1}{4} + \dfrac{- 3}{3}$
\item $D = \dfrac{6}{10} + 10$
\item $E = \dfrac{- 7}{2} \times \dfrac{5}{1}$
\item $F = \dfrac{2}{4} \times 1$
\end{enumerate}
\end{multicols}
\end{exercise}
\begin{solution}
\begin{enumerate}
\item
\[
\dfrac{- 10}{3} - \dfrac{8}{3}=\dfrac{- 10}{3} - \dfrac{8}{3}=\dfrac{- 10 - 8}{3}=\dfrac{- 10 - 8}{3}=\dfrac{- 18}{3}
\]
\item
\[
\dfrac{3}{3} - \dfrac{2}{15}=\dfrac{3}{3} - \dfrac{2}{15}=\dfrac{3 \times 5}{3 \times 5} - \dfrac{2}{15}=\dfrac{15}{15} - \dfrac{2}{15}=\dfrac{15 - 2}{15}=\dfrac{15 - 2}{15}=\dfrac{13}{15}
\]
\item
\[
\dfrac{1}{4} + \dfrac{- 3}{3}=\dfrac{1 \times 3}{4 \times 3} + \dfrac{- 3 \times 4}{3 \times 4}=\dfrac{3}{12} + \dfrac{- 12}{12}=\dfrac{3 - 12}{12}=\dfrac{- 9}{12}
\]
\item
\[
\dfrac{6}{10} + 10=\dfrac{6}{10} + \dfrac{10}{1}=\dfrac{6}{10} + \dfrac{10 \times 10}{1 \times 10}=\dfrac{6}{10} + \dfrac{100}{10}=\dfrac{6 + 100}{10}=\dfrac{106}{10}
\]
\item
\[
\dfrac{- 7}{2} \times \dfrac{5}{1}=\dfrac{- 7 \times 5}{2 \times 1}=\dfrac{- 35}{2}
\]
\item
\[
\dfrac{2}{4} \times 1=\dfrac{2}{4}
\]
\end{enumerate}
\end{solution}
\begin{exercise}[subtitle={Développer réduire}]
Développer puis réduire les expressions suivantes
\begin{multicols}{2}
\begin{enumerate}
\item $A = (7x - 8)(- 3x - 8)$
\item $B = (5x + 7)(- 9x + 7)$
\item $C = (- 10x - 9)^{2}$
\item $D = 1 + x(8x + 2)$
\item $E = 10x^{2} + x(- 6x - 4)$
\item $F = 1(x - 7)(x + 5)$
\end{enumerate}
\end{multicols}
\end{exercise}
\begin{solution}
\begin{enumerate}
\item
\begin{align*}
A &= (7x - 8)(- 3x - 8)\\&= 7x \times - 3x + 7x \times - 8 - 8 \times - 3x - 8 \times - 8\\&= 7 \times - 3 \times x^{1 + 1} - 8 \times 7 \times x - 8 \times - 3 \times x + 64\\&= - 56x + 24x - 21x^{2} + 64\\&= (- 56 + 24) \times x - 21x^{2} + 64\\&= - 21x^{2} - 32x + 64
\end{align*}
\item
\begin{align*}
B &= (5x + 7)(- 9x + 7)\\&= 5x \times - 9x + 5x \times 7 + 7 \times - 9x + 7 \times 7\\&= 5 \times - 9 \times x^{1 + 1} + 7 \times 5 \times x + 7 \times - 9 \times x + 49\\&= 35x - 63x - 45x^{2} + 49\\&= (35 - 63) \times x - 45x^{2} + 49\\&= - 45x^{2} - 28x + 49
\end{align*}
\item
\begin{align*}
C &= (- 10x - 9)^{2}\\&= (- 10x - 9)(- 10x - 9)\\&= - 10x \times - 10x - 10x \times - 9 - 9 \times - 10x - 9 \times - 9\\&= - 10 \times - 10 \times x^{1 + 1} - 9 \times - 10 \times x - 9 \times - 10 \times x + 81\\&= 90x + 90x + 100x^{2} + 81\\&= (90 + 90) \times x + 100x^{2} + 81\\&= 100x^{2} + 180x + 81
\end{align*}
\item
\begin{align*}
D &= 1 + x(8x + 2)\\&= 1 + x \times 8x + x \times 2\\&= 8x^{2} + 2x + 1
\end{align*}
\item
\begin{align*}
E &= 10x^{2} + x(- 6x - 4)\\&= 10x^{2} + x \times - 6x + x \times - 4\\&= 10x^{2} - 6x^{2} - 4x\\&= 10x^{2} - 6x^{2} - 4x\\&= (10 - 6) \times x^{2} - 4x\\&= 4x^{2} - 4x
\end{align*}
\item
\begin{align*}
F &= 1(x - 7)(x + 5)\\&= (x - 7)(x + 5)\\&= x \times x + x \times 5 - 7x - 7 \times 5\\&= x^{2} - 35 + (5 - 7) \times x\\&= x^{2} - 2x - 35
\end{align*}
\end{enumerate}
\end{solution}
\begin{exercise}[subtitle={Étude de fonctions}]
Soit $f(x) = - 9x^{2} + 54x - 72$ une fonction définie sur $\R$.
\begin{enumerate}
\item Calculer les valeurs suivantes
\[
f(1) \qquad f(-2)
\]
\item Dériver la fonction $f$
\item Étudier le signe de $f'$ puis en déduire les variations de $f$.
\item Est-ce que $f$ admet un maximum? un minimum? Calculer sa valeur.
\end{enumerate}
\end{exercise}
\begin{solution}
\begin{enumerate}
\item On remplace $x$ par les valeurs demandées
\[
f(1) = - 9 \times 1^{2} + 54 \times 1 - 72=- 9 \times 1 + 54 - 72=- 9 - 18=- 27
\]
\[
f(-1) = - 9 \times - 1^{2} + 54 \times - 1 - 72=- 9 \times 1 - 54 - 72=- 9 - 126=- 135
\]
\item Pas de solutions automatiques.
\item Pas de solutions automatiques.
\end{enumerate}
\end{solution}
%\printsolutionstype{exercise}
\end{document}
%%% Local Variables:
%%% mode: latex
%%% TeX-master: "master"
%%% End:

View File

@ -0,0 +1,141 @@
\documentclass[a5paper,10pt]{article}
\usepackage{myXsim}
\usepackage{tasks}
% Title Page
\title{DM1 \hfill BENHATTAL Chakir}
\tribe{TST}
\date{Toussain 2020}
\begin{document}
\maketitle
\begin{exercise}[subtitle={Fractions}]
Faire les calculs avec les fraction suivants
\begin{multicols}{3}
\begin{enumerate}
\item $A = \dfrac{5}{3} - \dfrac{5}{3}$
\item $B = \dfrac{6}{7} - \dfrac{6}{14}$
\item $C = \dfrac{- 7}{8} + \dfrac{- 1}{7}$
\item $D = \dfrac{- 8}{2} + 10$
\item $E = \dfrac{- 2}{9} \times \dfrac{- 9}{8}$
\item $F = \dfrac{- 3}{3} \times - 10$
\end{enumerate}
\end{multicols}
\end{exercise}
\begin{solution}
\begin{enumerate}
\item
\[
\dfrac{5}{3} - \dfrac{5}{3}=\dfrac{5}{3} - \dfrac{5}{3}=\dfrac{5 - 5}{3}=\dfrac{5 - 5}{3}=\dfrac{0}{3}
\]
\item
\[
\dfrac{6}{7} - \dfrac{6}{14}=\dfrac{6}{7} - \dfrac{6}{14}=\dfrac{6 \times 2}{7 \times 2} - \dfrac{6}{14}=\dfrac{12}{14} - \dfrac{6}{14}=\dfrac{12 - 6}{14}=\dfrac{12 - 6}{14}=\dfrac{6}{14}
\]
\item
\[
\dfrac{- 7}{8} + \dfrac{- 1}{7}=\dfrac{- 7 \times 7}{8 \times 7} + \dfrac{- 1 \times 8}{7 \times 8}=\dfrac{- 49}{56} + \dfrac{- 8}{56}=\dfrac{- 49 - 8}{56}=\dfrac{- 57}{56}
\]
\item
\[
\dfrac{- 8}{2} + 10=\dfrac{- 8}{2} + \dfrac{10}{1}=\dfrac{- 8}{2} + \dfrac{10 \times 2}{1 \times 2}=\dfrac{- 8}{2} + \dfrac{20}{2}=\dfrac{- 8 + 20}{2}=\dfrac{12}{2}
\]
\item
\[
\dfrac{- 2}{9} \times \dfrac{- 9}{8}=\dfrac{- 2 \times - 9}{9 \times 8}=\dfrac{18}{72}
\]
\item
\[
\dfrac{- 3}{3} \times - 10=\dfrac{- 3 \times - 10}{3}=\dfrac{30}{3}
\]
\end{enumerate}
\end{solution}
\begin{exercise}[subtitle={Développer réduire}]
Développer puis réduire les expressions suivantes
\begin{multicols}{2}
\begin{enumerate}
\item $A = (- 10x + 2)(9x + 2)$
\item $B = (2x - 7)(- 9x - 7)$
\item $C = (4x - 8)^{2}$
\item $D = - 3 + x(9x - 8)$
\item $E = - 4x^{2} + x(6x - 5)$
\item $F = - 10(x + 5)(x + 9)$
\end{enumerate}
\end{multicols}
\end{exercise}
\begin{solution}
\begin{enumerate}
\item
\begin{align*}
A &= (- 10x + 2)(9x + 2)\\&= - 10x \times 9x - 10x \times 2 + 2 \times 9x + 2 \times 2\\&= - 10 \times 9 \times x^{1 + 1} + 2 \times - 10 \times x + 2 \times 9 \times x + 4\\&= - 20x + 18x - 90x^{2} + 4\\&= (- 20 + 18) \times x - 90x^{2} + 4\\&= - 90x^{2} - 2x + 4
\end{align*}
\item
\begin{align*}
B &= (2x - 7)(- 9x - 7)\\&= 2x \times - 9x + 2x \times - 7 - 7 \times - 9x - 7 \times - 7\\&= 2 \times - 9 \times x^{1 + 1} - 7 \times 2 \times x - 7 \times - 9 \times x + 49\\&= - 14x + 63x - 18x^{2} + 49\\&= (- 14 + 63) \times x - 18x^{2} + 49\\&= - 18x^{2} + 49x + 49
\end{align*}
\item
\begin{align*}
C &= (4x - 8)^{2}\\&= (4x - 8)(4x - 8)\\&= 4x \times 4x + 4x \times - 8 - 8 \times 4x - 8 \times - 8\\&= 4 \times 4 \times x^{1 + 1} - 8 \times 4 \times x - 8 \times 4 \times x + 64\\&= - 32x - 32x + 16x^{2} + 64\\&= (- 32 - 32) \times x + 16x^{2} + 64\\&= 16x^{2} - 64x + 64
\end{align*}
\item
\begin{align*}
D &= - 3 + x(9x - 8)\\&= - 3 + x \times 9x + x \times - 8\\&= 9x^{2} - 8x - 3
\end{align*}
\item
\begin{align*}
E &= - 4x^{2} + x(6x - 5)\\&= - 4x^{2} + x \times 6x + x \times - 5\\&= - 4x^{2} + 6x^{2} - 5x\\&= - 4x^{2} + 6x^{2} - 5x\\&= (- 4 + 6) \times x^{2} - 5x\\&= 2x^{2} - 5x
\end{align*}
\item
\begin{align*}
F &= - 10(x + 5)(x + 9)\\&= (- 10x - 10 \times 5)(x + 9)\\&= (- 10x - 50)(x + 9)\\&= - 10x \times x - 10x \times 9 - 50x - 50 \times 9\\&= 9 \times - 10 \times x - 450 - 10x^{2} - 50x\\&= - 90x - 450 - 10x^{2} - 50x\\&= - 10x^{2} - 90x - 50x - 450\\&= - 10x^{2} + (- 90 - 50) \times x - 450\\&= - 10x^{2} - 140x - 450
\end{align*}
\end{enumerate}
\end{solution}
\begin{exercise}[subtitle={Étude de fonctions}]
Soit $f(x) = - 2x^{2} + 8x + 24$ une fonction définie sur $\R$.
\begin{enumerate}
\item Calculer les valeurs suivantes
\[
f(1) \qquad f(-2)
\]
\item Dériver la fonction $f$
\item Étudier le signe de $f'$ puis en déduire les variations de $f$.
\item Est-ce que $f$ admet un maximum? un minimum? Calculer sa valeur.
\end{enumerate}
\end{exercise}
\begin{solution}
\begin{enumerate}
\item On remplace $x$ par les valeurs demandées
\[
f(1) = - 2 \times 1^{2} + 8 \times 1 + 24=- 2 \times 1 + 8 + 24=- 2 + 32=30
\]
\[
f(-1) = - 2 \times - 1^{2} + 8 \times - 1 + 24=- 2 \times 1 - 8 + 24=- 2 + 16=14
\]
\item Pas de solutions automatiques.
\item Pas de solutions automatiques.
\end{enumerate}
\end{solution}
%\printsolutionstype{exercise}
\end{document}
%%% Local Variables:
%%% mode: latex
%%% TeX-master: "master"
%%% End:

View File

@ -0,0 +1,141 @@
\documentclass[a5paper,10pt]{article}
\usepackage{myXsim}
\usepackage{tasks}
% Title Page
\title{DM1 \hfill CLAIN Avinash}
\tribe{TST}
\date{Toussain 2020}
\begin{document}
\maketitle
\begin{exercise}[subtitle={Fractions}]
Faire les calculs avec les fraction suivants
\begin{multicols}{3}
\begin{enumerate}
\item $A = \dfrac{4}{4} - \dfrac{- 9}{4}$
\item $B = \dfrac{2}{6} - \dfrac{1}{60}$
\item $C = \dfrac{4}{6} + \dfrac{- 3}{5}$
\item $D = \dfrac{- 8}{7} - 8$
\item $E = \dfrac{- 1}{3} \times \dfrac{- 4}{2}$
\item $F = \dfrac{2}{6} \times 8$
\end{enumerate}
\end{multicols}
\end{exercise}
\begin{solution}
\begin{enumerate}
\item
\[
\dfrac{4}{4} - \dfrac{- 9}{4}=\dfrac{4}{4} + \dfrac{9}{4}=\dfrac{4 + 9}{4}=\dfrac{13}{4}
\]
\item
\[
\dfrac{2}{6} - \dfrac{1}{60}=\dfrac{2}{6} - \dfrac{1}{60}=\dfrac{2 \times 10}{6 \times 10} - \dfrac{1}{60}=\dfrac{20}{60} - \dfrac{1}{60}=\dfrac{20 - 1}{60}=\dfrac{20 - 1}{60}=\dfrac{19}{60}
\]
\item
\[
\dfrac{4}{6} + \dfrac{- 3}{5}=\dfrac{4 \times 5}{6 \times 5} + \dfrac{- 3 \times 6}{5 \times 6}=\dfrac{20}{30} + \dfrac{- 18}{30}=\dfrac{20 - 18}{30}=\dfrac{2}{30}
\]
\item
\[
\dfrac{- 8}{7} - 8=\dfrac{- 8}{7} + \dfrac{- 8}{1}=\dfrac{- 8}{7} + \dfrac{- 8 \times 7}{1 \times 7}=\dfrac{- 8}{7} + \dfrac{- 56}{7}=\dfrac{- 8 - 56}{7}=\dfrac{- 64}{7}
\]
\item
\[
\dfrac{- 1}{3} \times \dfrac{- 4}{2}=\dfrac{- 1 \times - 4}{3 \times 2}=\dfrac{4}{6}
\]
\item
\[
\dfrac{2}{6} \times 8=\dfrac{2 \times 8}{6}=\dfrac{16}{6}
\]
\end{enumerate}
\end{solution}
\begin{exercise}[subtitle={Développer réduire}]
Développer puis réduire les expressions suivantes
\begin{multicols}{2}
\begin{enumerate}
\item $A = (9x - 7)(3x - 7)$
\item $B = (- 10x - 7)(9x - 7)$
\item $C = (3x - 7)^{2}$
\item $D = 9 + x(- 1x - 4)$
\item $E = - 2x^{2} + x(10x + 6)$
\item $F = 2(x - 9)(x + 7)$
\end{enumerate}
\end{multicols}
\end{exercise}
\begin{solution}
\begin{enumerate}
\item
\begin{align*}
A &= (9x - 7)(3x - 7)\\&= 9x \times 3x + 9x \times - 7 - 7 \times 3x - 7 \times - 7\\&= 9 \times 3 \times x^{1 + 1} - 7 \times 9 \times x - 7 \times 3 \times x + 49\\&= - 63x - 21x + 27x^{2} + 49\\&= (- 63 - 21) \times x + 27x^{2} + 49\\&= 27x^{2} - 84x + 49
\end{align*}
\item
\begin{align*}
B &= (- 10x - 7)(9x - 7)\\&= - 10x \times 9x - 10x \times - 7 - 7 \times 9x - 7 \times - 7\\&= - 10 \times 9 \times x^{1 + 1} - 7 \times - 10 \times x - 7 \times 9 \times x + 49\\&= 70x - 63x - 90x^{2} + 49\\&= (70 - 63) \times x - 90x^{2} + 49\\&= - 90x^{2} + 7x + 49
\end{align*}
\item
\begin{align*}
C &= (3x - 7)^{2}\\&= (3x - 7)(3x - 7)\\&= 3x \times 3x + 3x \times - 7 - 7 \times 3x - 7 \times - 7\\&= 3 \times 3 \times x^{1 + 1} - 7 \times 3 \times x - 7 \times 3 \times x + 49\\&= - 21x - 21x + 9x^{2} + 49\\&= (- 21 - 21) \times x + 9x^{2} + 49\\&= 9x^{2} - 42x + 49
\end{align*}
\item
\begin{align*}
D &= 9 + x(- 1x - 4)\\&= 9 + x \times - x + x \times - 4\\&= - x^{2} - 4x + 9
\end{align*}
\item
\begin{align*}
E &= - 2x^{2} + x(10x + 6)\\&= - 2x^{2} + x \times 10x + x \times 6\\&= - 2x^{2} + 10x^{2} + 6x\\&= - 2x^{2} + 10x^{2} + 6x\\&= (- 2 + 10) \times x^{2} + 6x\\&= 8x^{2} + 6x
\end{align*}
\item
\begin{align*}
F &= 2(x - 9)(x + 7)\\&= (2x + 2 \times - 9)(x + 7)\\&= (2x - 18)(x + 7)\\&= 2x \times x + 2x \times 7 - 18x - 18 \times 7\\&= 7 \times 2 \times x - 126 + 2x^{2} - 18x\\&= 14x - 126 + 2x^{2} - 18x\\&= 2x^{2} + 14x - 18x - 126\\&= 2x^{2} + (14 - 18) \times x - 126\\&= 2x^{2} - 4x - 126
\end{align*}
\end{enumerate}
\end{solution}
\begin{exercise}[subtitle={Étude de fonctions}]
Soit $f(x) = 6x^{2} + 18x - 168$ une fonction définie sur $\R$.
\begin{enumerate}
\item Calculer les valeurs suivantes
\[
f(1) \qquad f(-2)
\]
\item Dériver la fonction $f$
\item Étudier le signe de $f'$ puis en déduire les variations de $f$.
\item Est-ce que $f$ admet un maximum? un minimum? Calculer sa valeur.
\end{enumerate}
\end{exercise}
\begin{solution}
\begin{enumerate}
\item On remplace $x$ par les valeurs demandées
\[
f(1) = 6 \times 1^{2} + 18 \times 1 - 168=6 \times 1 + 18 - 168=6 - 150=- 144
\]
\[
f(-1) = 6 \times - 1^{2} + 18 \times - 1 - 168=6 \times 1 - 18 - 168=6 - 186=- 180
\]
\item Pas de solutions automatiques.
\item Pas de solutions automatiques.
\end{enumerate}
\end{solution}
%\printsolutionstype{exercise}
\end{document}
%%% Local Variables:
%%% mode: latex
%%% TeX-master: "master"
%%% End:

View File

@ -0,0 +1,141 @@
\documentclass[a5paper,10pt]{article}
\usepackage{myXsim}
\usepackage{tasks}
% Title Page
\title{DM1 \hfill COLASSI Alexis}
\tribe{TST}
\date{Toussain 2020}
\begin{document}
\maketitle
\begin{exercise}[subtitle={Fractions}]
Faire les calculs avec les fraction suivants
\begin{multicols}{3}
\begin{enumerate}
\item $A = \dfrac{- 9}{7} - \dfrac{- 4}{7}$
\item $B = \dfrac{- 10}{8} - \dfrac{8}{80}$
\item $C = \dfrac{9}{8} + \dfrac{- 8}{7}$
\item $D = \dfrac{- 2}{3} + 1$
\item $E = \dfrac{8}{3} \times \dfrac{- 1}{2}$
\item $F = \dfrac{1}{8} \times 9$
\end{enumerate}
\end{multicols}
\end{exercise}
\begin{solution}
\begin{enumerate}
\item
\[
\dfrac{- 9}{7} - \dfrac{- 4}{7}=\dfrac{- 9}{7} + \dfrac{4}{7}=\dfrac{- 9 + 4}{7}=\dfrac{- 5}{7}
\]
\item
\[
\dfrac{- 10}{8} - \dfrac{8}{80}=\dfrac{- 10}{8} - \dfrac{8}{80}=\dfrac{- 10 \times 10}{8 \times 10} - \dfrac{8}{80}=\dfrac{- 100}{80} - \dfrac{8}{80}=\dfrac{- 100 - 8}{80}=\dfrac{- 100 - 8}{80}=\dfrac{- 108}{80}
\]
\item
\[
\dfrac{9}{8} + \dfrac{- 8}{7}=\dfrac{9 \times 7}{8 \times 7} + \dfrac{- 8 \times 8}{7 \times 8}=\dfrac{63}{56} + \dfrac{- 64}{56}=\dfrac{63 - 64}{56}=\dfrac{- 1}{56}
\]
\item
\[
\dfrac{- 2}{3} + 1=\dfrac{- 2}{3} + \dfrac{1}{1}=\dfrac{- 2}{3} + \dfrac{1 \times 3}{1 \times 3}=\dfrac{- 2}{3} + \dfrac{3}{3}=\dfrac{- 2 + 3}{3}=\dfrac{1}{3}
\]
\item
\[
\dfrac{8}{3} \times \dfrac{- 1}{2}=\dfrac{8 \times - 1}{3 \times 2}=\dfrac{- 8}{6}
\]
\item
\[
\dfrac{1}{8} \times 9=\dfrac{1 \times 9}{8}=\dfrac{9}{8}
\]
\end{enumerate}
\end{solution}
\begin{exercise}[subtitle={Développer réduire}]
Développer puis réduire les expressions suivantes
\begin{multicols}{2}
\begin{enumerate}
\item $A = (8x + 8)(4x + 8)$
\item $B = (7x + 4)(5x + 4)$
\item $C = (- 5x - 5)^{2}$
\item $D = 5 + x(5x - 5)$
\item $E = 8x^{2} + x(- 4x + 8)$
\item $F = 4(x + 5)(x - 5)$
\end{enumerate}
\end{multicols}
\end{exercise}
\begin{solution}
\begin{enumerate}
\item
\begin{align*}
A &= (8x + 8)(4x + 8)\\&= 8x \times 4x + 8x \times 8 + 8 \times 4x + 8 \times 8\\&= 8 \times 4 \times x^{1 + 1} + 8 \times 8 \times x + 8 \times 4 \times x + 64\\&= 64x + 32x + 32x^{2} + 64\\&= (64 + 32) \times x + 32x^{2} + 64\\&= 32x^{2} + 96x + 64
\end{align*}
\item
\begin{align*}
B &= (7x + 4)(5x + 4)\\&= 7x \times 5x + 7x \times 4 + 4 \times 5x + 4 \times 4\\&= 7 \times 5 \times x^{1 + 1} + 4 \times 7 \times x + 4 \times 5 \times x + 16\\&= 28x + 20x + 35x^{2} + 16\\&= (28 + 20) \times x + 35x^{2} + 16\\&= 35x^{2} + 48x + 16
\end{align*}
\item
\begin{align*}
C &= (- 5x - 5)^{2}\\&= (- 5x - 5)(- 5x - 5)\\&= - 5x \times - 5x - 5x \times - 5 - 5 \times - 5x - 5 \times - 5\\&= - 5 \times - 5 \times x^{1 + 1} - 5 \times - 5 \times x - 5 \times - 5 \times x + 25\\&= 25x + 25x + 25x^{2} + 25\\&= (25 + 25) \times x + 25x^{2} + 25\\&= 25x^{2} + 50x + 25
\end{align*}
\item
\begin{align*}
D &= 5 + x(5x - 5)\\&= 5 + x \times 5x + x \times - 5\\&= 5x^{2} - 5x + 5
\end{align*}
\item
\begin{align*}
E &= 8x^{2} + x(- 4x + 8)\\&= 8x^{2} + x \times - 4x + x \times 8\\&= 8x^{2} - 4x^{2} + 8x\\&= 8x^{2} - 4x^{2} + 8x\\&= (8 - 4) \times x^{2} + 8x\\&= 4x^{2} + 8x
\end{align*}
\item
\begin{align*}
F &= 4(x + 5)(x - 5)\\&= (4x + 4 \times 5)(x - 5)\\&= (4x + 20)(x - 5)\\&= 4x \times x + 4x \times - 5 + 20x + 20 \times - 5\\&= - 5 \times 4 \times x - 100 + 4x^{2} + 20x\\&= - 20x - 100 + 4x^{2} + 20x\\&= 4x^{2} - 20x + 20x - 100\\&= 4x^{2} + (- 20 + 20) \times x - 100\\&= 4x^{2} - 100 + 0x\\&= 4x^{2} - 100
\end{align*}
\end{enumerate}
\end{solution}
\begin{exercise}[subtitle={Étude de fonctions}]
Soit $f(x) = 9x^{2} + 81x + 72$ une fonction définie sur $\R$.
\begin{enumerate}
\item Calculer les valeurs suivantes
\[
f(1) \qquad f(-2)
\]
\item Dériver la fonction $f$
\item Étudier le signe de $f'$ puis en déduire les variations de $f$.
\item Est-ce que $f$ admet un maximum? un minimum? Calculer sa valeur.
\end{enumerate}
\end{exercise}
\begin{solution}
\begin{enumerate}
\item On remplace $x$ par les valeurs demandées
\[
f(1) = 9 \times 1^{2} + 81 \times 1 + 72=9 \times 1 + 81 + 72=9 + 153=162
\]
\[
f(-1) = 9 \times - 1^{2} + 81 \times - 1 + 72=9 \times 1 - 81 + 72=9 - 9=0
\]
\item Pas de solutions automatiques.
\item Pas de solutions automatiques.
\end{enumerate}
\end{solution}
%\printsolutionstype{exercise}
\end{document}
%%% Local Variables:
%%% mode: latex
%%% TeX-master: "master"
%%% End:

View File

@ -0,0 +1,141 @@
\documentclass[a5paper,10pt]{article}
\usepackage{myXsim}
\usepackage{tasks}
% Title Page
\title{DM1 \hfill COUBAT Alexis}
\tribe{TST}
\date{Toussain 2020}
\begin{document}
\maketitle
\begin{exercise}[subtitle={Fractions}]
Faire les calculs avec les fraction suivants
\begin{multicols}{3}
\begin{enumerate}
\item $A = \dfrac{- 1}{2} - \dfrac{- 3}{2}$
\item $B = \dfrac{- 5}{5} - \dfrac{- 7}{30}$
\item $C = \dfrac{- 1}{4} + \dfrac{3}{3}$
\item $D = \dfrac{7}{10} - 7$
\item $E = \dfrac{- 8}{2} \times \dfrac{- 4}{1}$
\item $F = \dfrac{1}{7} \times 5$
\end{enumerate}
\end{multicols}
\end{exercise}
\begin{solution}
\begin{enumerate}
\item
\[
\dfrac{- 1}{2} - \dfrac{- 3}{2}=\dfrac{- 1}{2} + \dfrac{3}{2}=\dfrac{- 1 + 3}{2}=\dfrac{2}{2}
\]
\item
\[
\dfrac{- 5}{5} - \dfrac{- 7}{30}=\dfrac{- 5}{5} + \dfrac{7}{30}=\dfrac{- 5 \times 6}{5 \times 6} + \dfrac{7}{30}=\dfrac{- 30}{30} + \dfrac{7}{30}=\dfrac{- 30 + 7}{30}=\dfrac{- 23}{30}
\]
\item
\[
\dfrac{- 1}{4} + \dfrac{3}{3}=\dfrac{- 1 \times 3}{4 \times 3} + \dfrac{3 \times 4}{3 \times 4}=\dfrac{- 3}{12} + \dfrac{12}{12}=\dfrac{- 3 + 12}{12}=\dfrac{9}{12}
\]
\item
\[
\dfrac{7}{10} - 7=\dfrac{7}{10} + \dfrac{- 7}{1}=\dfrac{7}{10} + \dfrac{- 7 \times 10}{1 \times 10}=\dfrac{7}{10} + \dfrac{- 70}{10}=\dfrac{7 - 70}{10}=\dfrac{- 63}{10}
\]
\item
\[
\dfrac{- 8}{2} \times \dfrac{- 4}{1}=\dfrac{- 8 \times - 4}{2 \times 1}=\dfrac{32}{2}
\]
\item
\[
\dfrac{1}{7} \times 5=\dfrac{1 \times 5}{7}=\dfrac{5}{7}
\]
\end{enumerate}
\end{solution}
\begin{exercise}[subtitle={Développer réduire}]
Développer puis réduire les expressions suivantes
\begin{multicols}{2}
\begin{enumerate}
\item $A = (- 2x - 6)(- 4x - 6)$
\item $B = (- 10x - 3)(- 5x - 3)$
\item $C = (10x - 7)^{2}$
\item $D = - 4 + x(10x - 3)$
\item $E = - 3x^{2} + x(- 1x - 5)$
\item $F = - 3(x + 2)(x + 8)$
\end{enumerate}
\end{multicols}
\end{exercise}
\begin{solution}
\begin{enumerate}
\item
\begin{align*}
A &= (- 2x - 6)(- 4x - 6)\\&= - 2x \times - 4x - 2x \times - 6 - 6 \times - 4x - 6 \times - 6\\&= - 2 \times - 4 \times x^{1 + 1} - 6 \times - 2 \times x - 6 \times - 4 \times x + 36\\&= 12x + 24x + 8x^{2} + 36\\&= (12 + 24) \times x + 8x^{2} + 36\\&= 8x^{2} + 36x + 36
\end{align*}
\item
\begin{align*}
B &= (- 10x - 3)(- 5x - 3)\\&= - 10x \times - 5x - 10x \times - 3 - 3 \times - 5x - 3 \times - 3\\&= - 10 \times - 5 \times x^{1 + 1} - 3 \times - 10 \times x - 3 \times - 5 \times x + 9\\&= 30x + 15x + 50x^{2} + 9\\&= (30 + 15) \times x + 50x^{2} + 9\\&= 50x^{2} + 45x + 9
\end{align*}
\item
\begin{align*}
C &= (10x - 7)^{2}\\&= (10x - 7)(10x - 7)\\&= 10x \times 10x + 10x \times - 7 - 7 \times 10x - 7 \times - 7\\&= 10 \times 10 \times x^{1 + 1} - 7 \times 10 \times x - 7 \times 10 \times x + 49\\&= - 70x - 70x + 100x^{2} + 49\\&= (- 70 - 70) \times x + 100x^{2} + 49\\&= 100x^{2} - 140x + 49
\end{align*}
\item
\begin{align*}
D &= - 4 + x(10x - 3)\\&= - 4 + x \times 10x + x \times - 3\\&= 10x^{2} - 3x - 4
\end{align*}
\item
\begin{align*}
E &= - 3x^{2} + x(- 1x - 5)\\&= - 3x^{2} + x \times - x + x \times - 5\\&= - 3x^{2} - x^{2} - 5x\\&= - 3x^{2} - x^{2} - 5x\\&= (- 3 - 1) \times x^{2} - 5x\\&= - 4x^{2} - 5x
\end{align*}
\item
\begin{align*}
F &= - 3(x + 2)(x + 8)\\&= (- 3x - 3 \times 2)(x + 8)\\&= (- 3x - 6)(x + 8)\\&= - 3x \times x - 3x \times 8 - 6x - 6 \times 8\\&= 8 \times - 3 \times x - 48 - 3x^{2} - 6x\\&= - 24x - 48 - 3x^{2} - 6x\\&= - 3x^{2} - 24x - 6x - 48\\&= - 3x^{2} + (- 24 - 6) \times x - 48\\&= - 3x^{2} - 30x - 48
\end{align*}
\end{enumerate}
\end{solution}
\begin{exercise}[subtitle={Étude de fonctions}]
Soit $f(x) = - 6x^{2} + 72x - 162$ une fonction définie sur $\R$.
\begin{enumerate}
\item Calculer les valeurs suivantes
\[
f(1) \qquad f(-2)
\]
\item Dériver la fonction $f$
\item Étudier le signe de $f'$ puis en déduire les variations de $f$.
\item Est-ce que $f$ admet un maximum? un minimum? Calculer sa valeur.
\end{enumerate}
\end{exercise}
\begin{solution}
\begin{enumerate}
\item On remplace $x$ par les valeurs demandées
\[
f(1) = - 6 \times 1^{2} + 72 \times 1 - 162=- 6 \times 1 + 72 - 162=- 6 - 90=- 96
\]
\[
f(-1) = - 6 \times - 1^{2} + 72 \times - 1 - 162=- 6 \times 1 - 72 - 162=- 6 - 234=- 240
\]
\item Pas de solutions automatiques.
\item Pas de solutions automatiques.
\end{enumerate}
\end{solution}
%\printsolutionstype{exercise}
\end{document}
%%% Local Variables:
%%% mode: latex
%%% TeX-master: "master"
%%% End:

View File

@ -0,0 +1,141 @@
\documentclass[a5paper,10pt]{article}
\usepackage{myXsim}
\usepackage{tasks}
% Title Page
\title{DM1 \hfill COULLON Anis}
\tribe{TST}
\date{Toussain 2020}
\begin{document}
\maketitle
\begin{exercise}[subtitle={Fractions}]
Faire les calculs avec les fraction suivants
\begin{multicols}{3}
\begin{enumerate}
\item $A = \dfrac{- 10}{10} - \dfrac{- 1}{10}$
\item $B = \dfrac{8}{8} - \dfrac{5}{16}$
\item $C = \dfrac{- 2}{9} + \dfrac{8}{8}$
\item $D = \dfrac{10}{6} - 8$
\item $E = \dfrac{- 5}{8} \times \dfrac{- 4}{7}$
\item $F = \dfrac{5}{7} \times 6$
\end{enumerate}
\end{multicols}
\end{exercise}
\begin{solution}
\begin{enumerate}
\item
\[
\dfrac{- 10}{10} - \dfrac{- 1}{10}=\dfrac{- 10}{10} + \dfrac{1}{10}=\dfrac{- 10 + 1}{10}=\dfrac{- 9}{10}
\]
\item
\[
\dfrac{8}{8} - \dfrac{5}{16}=\dfrac{8}{8} - \dfrac{5}{16}=\dfrac{8 \times 2}{8 \times 2} - \dfrac{5}{16}=\dfrac{16}{16} - \dfrac{5}{16}=\dfrac{16 - 5}{16}=\dfrac{16 - 5}{16}=\dfrac{11}{16}
\]
\item
\[
\dfrac{- 2}{9} + \dfrac{8}{8}=\dfrac{- 2 \times 8}{9 \times 8} + \dfrac{8 \times 9}{8 \times 9}=\dfrac{- 16}{72} + \dfrac{72}{72}=\dfrac{- 16 + 72}{72}=\dfrac{56}{72}
\]
\item
\[
\dfrac{10}{6} - 8=\dfrac{10}{6} + \dfrac{- 8}{1}=\dfrac{10}{6} + \dfrac{- 8 \times 6}{1 \times 6}=\dfrac{10}{6} + \dfrac{- 48}{6}=\dfrac{10 - 48}{6}=\dfrac{- 38}{6}
\]
\item
\[
\dfrac{- 5}{8} \times \dfrac{- 4}{7}=\dfrac{- 5 \times - 4}{8 \times 7}=\dfrac{20}{56}
\]
\item
\[
\dfrac{5}{7} \times 6=\dfrac{5 \times 6}{7}=\dfrac{30}{7}
\]
\end{enumerate}
\end{solution}
\begin{exercise}[subtitle={Développer réduire}]
Développer puis réduire les expressions suivantes
\begin{multicols}{2}
\begin{enumerate}
\item $A = (5x - 3)(4x - 3)$
\item $B = (3x + 5)(- 9x + 5)$
\item $C = (8x - 5)^{2}$
\item $D = - 3 + x(5x + 7)$
\item $E = 10x^{2} + x(- 1x - 7)$
\item $F = - 6(x + 2)(x + 9)$
\end{enumerate}
\end{multicols}
\end{exercise}
\begin{solution}
\begin{enumerate}
\item
\begin{align*}
A &= (5x - 3)(4x - 3)\\&= 5x \times 4x + 5x \times - 3 - 3 \times 4x - 3 \times - 3\\&= 5 \times 4 \times x^{1 + 1} - 3 \times 5 \times x - 3 \times 4 \times x + 9\\&= - 15x - 12x + 20x^{2} + 9\\&= (- 15 - 12) \times x + 20x^{2} + 9\\&= 20x^{2} - 27x + 9
\end{align*}
\item
\begin{align*}
B &= (3x + 5)(- 9x + 5)\\&= 3x \times - 9x + 3x \times 5 + 5 \times - 9x + 5 \times 5\\&= 3 \times - 9 \times x^{1 + 1} + 5 \times 3 \times x + 5 \times - 9 \times x + 25\\&= 15x - 45x - 27x^{2} + 25\\&= (15 - 45) \times x - 27x^{2} + 25\\&= - 27x^{2} - 30x + 25
\end{align*}
\item
\begin{align*}
C &= (8x - 5)^{2}\\&= (8x - 5)(8x - 5)\\&= 8x \times 8x + 8x \times - 5 - 5 \times 8x - 5 \times - 5\\&= 8 \times 8 \times x^{1 + 1} - 5 \times 8 \times x - 5 \times 8 \times x + 25\\&= - 40x - 40x + 64x^{2} + 25\\&= (- 40 - 40) \times x + 64x^{2} + 25\\&= 64x^{2} - 80x + 25
\end{align*}
\item
\begin{align*}
D &= - 3 + x(5x + 7)\\&= - 3 + x \times 5x + x \times 7\\&= 5x^{2} + 7x - 3
\end{align*}
\item
\begin{align*}
E &= 10x^{2} + x(- 1x - 7)\\&= 10x^{2} + x \times - x + x \times - 7\\&= 10x^{2} - x^{2} - 7x\\&= 10x^{2} - x^{2} - 7x\\&= (10 - 1) \times x^{2} - 7x\\&= 9x^{2} - 7x
\end{align*}
\item
\begin{align*}
F &= - 6(x + 2)(x + 9)\\&= (- 6x - 6 \times 2)(x + 9)\\&= (- 6x - 12)(x + 9)\\&= - 6x \times x - 6x \times 9 - 12x - 12 \times 9\\&= 9 \times - 6 \times x - 108 - 6x^{2} - 12x\\&= - 54x - 108 - 6x^{2} - 12x\\&= - 6x^{2} - 54x - 12x - 108\\&= - 6x^{2} + (- 54 - 12) \times x - 108\\&= - 6x^{2} - 66x - 108
\end{align*}
\end{enumerate}
\end{solution}
\begin{exercise}[subtitle={Étude de fonctions}]
Soit $f(x) = - 7x^{2} - 70x - 175$ une fonction définie sur $\R$.
\begin{enumerate}
\item Calculer les valeurs suivantes
\[
f(1) \qquad f(-2)
\]
\item Dériver la fonction $f$
\item Étudier le signe de $f'$ puis en déduire les variations de $f$.
\item Est-ce que $f$ admet un maximum? un minimum? Calculer sa valeur.
\end{enumerate}
\end{exercise}
\begin{solution}
\begin{enumerate}
\item On remplace $x$ par les valeurs demandées
\[
f(1) = - 7 \times 1^{2} - 70 \times 1 - 175=- 7 \times 1 - 70 - 175=- 7 - 245=- 252
\]
\[
f(-1) = - 7 \times - 1^{2} - 70 \times - 1 - 175=- 7 \times 1 + 70 - 175=- 7 - 105=- 112
\]
\item Pas de solutions automatiques.
\item Pas de solutions automatiques.
\end{enumerate}
\end{solution}
%\printsolutionstype{exercise}
\end{document}
%%% Local Variables:
%%% mode: latex
%%% TeX-master: "master"
%%% End:

View File

@ -0,0 +1,141 @@
\documentclass[a5paper,10pt]{article}
\usepackage{myXsim}
\usepackage{tasks}
% Title Page
\title{DM1 \hfill DINGER Sölen}
\tribe{TST}
\date{Toussain 2020}
\begin{document}
\maketitle
\begin{exercise}[subtitle={Fractions}]
Faire les calculs avec les fraction suivants
\begin{multicols}{3}
\begin{enumerate}
\item $A = \dfrac{9}{3} - \dfrac{- 4}{3}$
\item $B = \dfrac{8}{8} - \dfrac{- 9}{16}$
\item $C = \dfrac{- 9}{10} + \dfrac{6}{9}$
\item $D = \dfrac{- 4}{8} - 6$
\item $E = \dfrac{7}{5} \times \dfrac{9}{4}$
\item $F = \dfrac{10}{3} \times 5$
\end{enumerate}
\end{multicols}
\end{exercise}
\begin{solution}
\begin{enumerate}
\item
\[
\dfrac{9}{3} - \dfrac{- 4}{3}=\dfrac{9}{3} + \dfrac{4}{3}=\dfrac{9 + 4}{3}=\dfrac{13}{3}
\]
\item
\[
\dfrac{8}{8} - \dfrac{- 9}{16}=\dfrac{8}{8} + \dfrac{9}{16}=\dfrac{8 \times 2}{8 \times 2} + \dfrac{9}{16}=\dfrac{16}{16} + \dfrac{9}{16}=\dfrac{16 + 9}{16}=\dfrac{25}{16}
\]
\item
\[
\dfrac{- 9}{10} + \dfrac{6}{9}=\dfrac{- 9 \times 9}{10 \times 9} + \dfrac{6 \times 10}{9 \times 10}=\dfrac{- 81}{90} + \dfrac{60}{90}=\dfrac{- 81 + 60}{90}=\dfrac{- 21}{90}
\]
\item
\[
\dfrac{- 4}{8} - 6=\dfrac{- 4}{8} + \dfrac{- 6}{1}=\dfrac{- 4}{8} + \dfrac{- 6 \times 8}{1 \times 8}=\dfrac{- 4}{8} + \dfrac{- 48}{8}=\dfrac{- 4 - 48}{8}=\dfrac{- 52}{8}
\]
\item
\[
\dfrac{7}{5} \times \dfrac{9}{4}=\dfrac{7 \times 9}{5 \times 4}=\dfrac{63}{20}
\]
\item
\[
\dfrac{10}{3} \times 5=\dfrac{10 \times 5}{3}=\dfrac{50}{3}
\]
\end{enumerate}
\end{solution}
\begin{exercise}[subtitle={Développer réduire}]
Développer puis réduire les expressions suivantes
\begin{multicols}{2}
\begin{enumerate}
\item $A = (4x + 7)(- 4x + 7)$
\item $B = (- 8x + 1)(6x + 1)$
\item $C = (9x + 3)^{2}$
\item $D = - 4 + x(5x - 10)$
\item $E = - 3x^{2} + x(- 2x - 5)$
\item $F = 9(x - 4)(x + 8)$
\end{enumerate}
\end{multicols}
\end{exercise}
\begin{solution}
\begin{enumerate}
\item
\begin{align*}
A &= (4x + 7)(- 4x + 7)\\&= 4x \times - 4x + 4x \times 7 + 7 \times - 4x + 7 \times 7\\&= 4 \times - 4 \times x^{1 + 1} + 7 \times 4 \times x + 7 \times - 4 \times x + 49\\&= 28x - 28x - 16x^{2} + 49\\&= (28 - 28) \times x - 16x^{2} + 49\\&= 0x - 16x^{2} + 49\\&= - 16x^{2} + 49
\end{align*}
\item
\begin{align*}
B &= (- 8x + 1)(6x + 1)\\&= - 8x \times 6x - 8x \times 1 + 1 \times 6x + 1 \times 1\\&= - 8 \times 6 \times x^{1 + 1} - 8x + 6x + 1\\&= - 48x^{2} - 8x + 6x + 1\\&= - 48x^{2} + (- 8 + 6) \times x + 1\\&= - 48x^{2} - 2x + 1
\end{align*}
\item
\begin{align*}
C &= (9x + 3)^{2}\\&= (9x + 3)(9x + 3)\\&= 9x \times 9x + 9x \times 3 + 3 \times 9x + 3 \times 3\\&= 9 \times 9 \times x^{1 + 1} + 3 \times 9 \times x + 3 \times 9 \times x + 9\\&= 27x + 27x + 81x^{2} + 9\\&= (27 + 27) \times x + 81x^{2} + 9\\&= 81x^{2} + 54x + 9
\end{align*}
\item
\begin{align*}
D &= - 4 + x(5x - 10)\\&= - 4 + x \times 5x + x \times - 10\\&= 5x^{2} - 10x - 4
\end{align*}
\item
\begin{align*}
E &= - 3x^{2} + x(- 2x - 5)\\&= - 3x^{2} + x \times - 2x + x \times - 5\\&= - 3x^{2} - 2x^{2} - 5x\\&= - 3x^{2} - 2x^{2} - 5x\\&= (- 3 - 2) \times x^{2} - 5x\\&= - 5x^{2} - 5x
\end{align*}
\item
\begin{align*}
F &= 9(x - 4)(x + 8)\\&= (9x + 9 \times - 4)(x + 8)\\&= (9x - 36)(x + 8)\\&= 9x \times x + 9x \times 8 - 36x - 36 \times 8\\&= 8 \times 9 \times x - 288 + 9x^{2} - 36x\\&= 72x - 288 + 9x^{2} - 36x\\&= 9x^{2} + 72x - 36x - 288\\&= 9x^{2} + (72 - 36) \times x - 288\\&= 9x^{2} + 36x - 288
\end{align*}
\end{enumerate}
\end{solution}
\begin{exercise}[subtitle={Étude de fonctions}]
Soit $f(x) = 2x^{2} + 2x - 40$ une fonction définie sur $\R$.
\begin{enumerate}
\item Calculer les valeurs suivantes
\[
f(1) \qquad f(-2)
\]
\item Dériver la fonction $f$
\item Étudier le signe de $f'$ puis en déduire les variations de $f$.
\item Est-ce que $f$ admet un maximum? un minimum? Calculer sa valeur.
\end{enumerate}
\end{exercise}
\begin{solution}
\begin{enumerate}
\item On remplace $x$ par les valeurs demandées
\[
f(1) = 2 \times 1^{2} + 2 \times 1 - 40=2 \times 1 + 2 - 40=2 - 38=- 36
\]
\[
f(-1) = 2 \times - 1^{2} + 2 \times - 1 - 40=2 \times 1 - 2 - 40=2 - 42=- 40
\]
\item Pas de solutions automatiques.
\item Pas de solutions automatiques.
\end{enumerate}
\end{solution}
%\printsolutionstype{exercise}
\end{document}
%%% Local Variables:
%%% mode: latex
%%% TeX-master: "master"
%%% End:

View File

@ -0,0 +1,141 @@
\documentclass[a5paper,10pt]{article}
\usepackage{myXsim}
\usepackage{tasks}
% Title Page
\title{DM1 \hfill EYRAUD Cynthia}
\tribe{TST}
\date{Toussain 2020}
\begin{document}
\maketitle
\begin{exercise}[subtitle={Fractions}]
Faire les calculs avec les fraction suivants
\begin{multicols}{3}
\begin{enumerate}
\item $A = \dfrac{6}{8} - \dfrac{- 10}{8}$
\item $B = \dfrac{- 1}{2} - \dfrac{- 2}{8}$
\item $C = \dfrac{- 1}{5} + \dfrac{- 8}{4}$
\item $D = \dfrac{- 3}{9} + 7$
\item $E = \dfrac{- 10}{9} \times \dfrac{7}{8}$
\item $F = \dfrac{3}{3} \times - 6$
\end{enumerate}
\end{multicols}
\end{exercise}
\begin{solution}
\begin{enumerate}
\item
\[
\dfrac{6}{8} - \dfrac{- 10}{8}=\dfrac{6}{8} + \dfrac{10}{8}=\dfrac{6 + 10}{8}=\dfrac{16}{8}
\]
\item
\[
\dfrac{- 1}{2} - \dfrac{- 2}{8}=\dfrac{- 1}{2} + \dfrac{2}{8}=\dfrac{- 1 \times 4}{2 \times 4} + \dfrac{2}{8}=\dfrac{- 4}{8} + \dfrac{2}{8}=\dfrac{- 4 + 2}{8}=\dfrac{- 2}{8}
\]
\item
\[
\dfrac{- 1}{5} + \dfrac{- 8}{4}=\dfrac{- 1 \times 4}{5 \times 4} + \dfrac{- 8 \times 5}{4 \times 5}=\dfrac{- 4}{20} + \dfrac{- 40}{20}=\dfrac{- 4 - 40}{20}=\dfrac{- 44}{20}
\]
\item
\[
\dfrac{- 3}{9} + 7=\dfrac{- 3}{9} + \dfrac{7}{1}=\dfrac{- 3}{9} + \dfrac{7 \times 9}{1 \times 9}=\dfrac{- 3}{9} + \dfrac{63}{9}=\dfrac{- 3 + 63}{9}=\dfrac{60}{9}
\]
\item
\[
\dfrac{- 10}{9} \times \dfrac{7}{8}=\dfrac{- 10 \times 7}{9 \times 8}=\dfrac{- 70}{72}
\]
\item
\[
\dfrac{3}{3} \times - 6=\dfrac{3 \times - 6}{3}=\dfrac{- 18}{3}
\]
\end{enumerate}
\end{solution}
\begin{exercise}[subtitle={Développer réduire}]
Développer puis réduire les expressions suivantes
\begin{multicols}{2}
\begin{enumerate}
\item $A = (7x - 6)(- 2x - 6)$
\item $B = (6x + 8)(10x + 8)$
\item $C = (5x + 5)^{2}$
\item $D = - 1 + x(- 4x + 3)$
\item $E = - 4x^{2} + x(2x - 3)$
\item $F = - 7(x - 8)(x + 7)$
\end{enumerate}
\end{multicols}
\end{exercise}
\begin{solution}
\begin{enumerate}
\item
\begin{align*}
A &= (7x - 6)(- 2x - 6)\\&= 7x \times - 2x + 7x \times - 6 - 6 \times - 2x - 6 \times - 6\\&= 7 \times - 2 \times x^{1 + 1} - 6 \times 7 \times x - 6 \times - 2 \times x + 36\\&= - 42x + 12x - 14x^{2} + 36\\&= (- 42 + 12) \times x - 14x^{2} + 36\\&= - 14x^{2} - 30x + 36
\end{align*}
\item
\begin{align*}
B &= (6x + 8)(10x + 8)\\&= 6x \times 10x + 6x \times 8 + 8 \times 10x + 8 \times 8\\&= 6 \times 10 \times x^{1 + 1} + 8 \times 6 \times x + 8 \times 10 \times x + 64\\&= 48x + 80x + 60x^{2} + 64\\&= (48 + 80) \times x + 60x^{2} + 64\\&= 60x^{2} + 128x + 64
\end{align*}
\item
\begin{align*}
C &= (5x + 5)^{2}\\&= (5x + 5)(5x + 5)\\&= 5x \times 5x + 5x \times 5 + 5 \times 5x + 5 \times 5\\&= 5 \times 5 \times x^{1 + 1} + 5 \times 5 \times x + 5 \times 5 \times x + 25\\&= 25x + 25x + 25x^{2} + 25\\&= (25 + 25) \times x + 25x^{2} + 25\\&= 25x^{2} + 50x + 25
\end{align*}
\item
\begin{align*}
D &= - 1 + x(- 4x + 3)\\&= - 1 + x \times - 4x + x \times 3\\&= - 4x^{2} + 3x - 1
\end{align*}
\item
\begin{align*}
E &= - 4x^{2} + x(2x - 3)\\&= - 4x^{2} + x \times 2x + x \times - 3\\&= - 4x^{2} + 2x^{2} - 3x\\&= - 4x^{2} + 2x^{2} - 3x\\&= (- 4 + 2) \times x^{2} - 3x\\&= - 2x^{2} - 3x
\end{align*}
\item
\begin{align*}
F &= - 7(x - 8)(x + 7)\\&= (- 7x - 7 \times - 8)(x + 7)\\&= (- 7x + 56)(x + 7)\\&= - 7x \times x - 7x \times 7 + 56x + 56 \times 7\\&= 7 \times - 7 \times x + 392 - 7x^{2} + 56x\\&= - 49x + 392 - 7x^{2} + 56x\\&= - 7x^{2} - 49x + 56x + 392\\&= - 7x^{2} + (- 49 + 56) \times x + 392\\&= - 7x^{2} + 7x + 392
\end{align*}
\end{enumerate}
\end{solution}
\begin{exercise}[subtitle={Étude de fonctions}]
Soit $f(x) = 7x^{2} + 28x - 315$ une fonction définie sur $\R$.
\begin{enumerate}
\item Calculer les valeurs suivantes
\[
f(1) \qquad f(-2)
\]
\item Dériver la fonction $f$
\item Étudier le signe de $f'$ puis en déduire les variations de $f$.
\item Est-ce que $f$ admet un maximum? un minimum? Calculer sa valeur.
\end{enumerate}
\end{exercise}
\begin{solution}
\begin{enumerate}
\item On remplace $x$ par les valeurs demandées
\[
f(1) = 7 \times 1^{2} + 28 \times 1 - 315=7 \times 1 + 28 - 315=7 - 287=- 280
\]
\[
f(-1) = 7 \times - 1^{2} + 28 \times - 1 - 315=7 \times 1 - 28 - 315=7 - 343=- 336
\]
\item Pas de solutions automatiques.
\item Pas de solutions automatiques.
\end{enumerate}
\end{solution}
%\printsolutionstype{exercise}
\end{document}
%%% Local Variables:
%%% mode: latex
%%% TeX-master: "master"
%%% End:

View File

@ -0,0 +1,141 @@
\documentclass[a5paper,10pt]{article}
\usepackage{myXsim}
\usepackage{tasks}
% Title Page
\title{DM1 \hfill FERREIRA Léo}
\tribe{TST}
\date{Toussain 2020}
\begin{document}
\maketitle
\begin{exercise}[subtitle={Fractions}]
Faire les calculs avec les fraction suivants
\begin{multicols}{3}
\begin{enumerate}
\item $A = \dfrac{4}{7} - \dfrac{7}{7}$
\item $B = \dfrac{9}{10} - \dfrac{6}{20}$
\item $C = \dfrac{10}{3} + \dfrac{- 1}{2}$
\item $D = \dfrac{1}{5} + 3$
\item $E = \dfrac{- 1}{4} \times \dfrac{- 7}{3}$
\item $F = \dfrac{3}{7} \times 9$
\end{enumerate}
\end{multicols}
\end{exercise}
\begin{solution}
\begin{enumerate}
\item
\[
\dfrac{4}{7} - \dfrac{7}{7}=\dfrac{4}{7} - \dfrac{7}{7}=\dfrac{4 - 7}{7}=\dfrac{4 - 7}{7}=\dfrac{- 3}{7}
\]
\item
\[
\dfrac{9}{10} - \dfrac{6}{20}=\dfrac{9}{10} - \dfrac{6}{20}=\dfrac{9 \times 2}{10 \times 2} - \dfrac{6}{20}=\dfrac{18}{20} - \dfrac{6}{20}=\dfrac{18 - 6}{20}=\dfrac{18 - 6}{20}=\dfrac{12}{20}
\]
\item
\[
\dfrac{10}{3} + \dfrac{- 1}{2}=\dfrac{10 \times 2}{3 \times 2} + \dfrac{- 1 \times 3}{2 \times 3}=\dfrac{20}{6} + \dfrac{- 3}{6}=\dfrac{20 - 3}{6}=\dfrac{17}{6}
\]
\item
\[
\dfrac{1}{5} + 3=\dfrac{1}{5} + \dfrac{3}{1}=\dfrac{1}{5} + \dfrac{3 \times 5}{1 \times 5}=\dfrac{1}{5} + \dfrac{15}{5}=\dfrac{1 + 15}{5}=\dfrac{16}{5}
\]
\item
\[
\dfrac{- 1}{4} \times \dfrac{- 7}{3}=\dfrac{- 1 \times - 7}{4 \times 3}=\dfrac{7}{12}
\]
\item
\[
\dfrac{3}{7} \times 9=\dfrac{3 \times 9}{7}=\dfrac{27}{7}
\]
\end{enumerate}
\end{solution}
\begin{exercise}[subtitle={Développer réduire}]
Développer puis réduire les expressions suivantes
\begin{multicols}{2}
\begin{enumerate}
\item $A = (- 8x - 7)(- 1x - 7)$
\item $B = (5x - 1)(- 2x - 1)$
\item $C = (- 3x + 7)^{2}$
\item $D = - 4 + x(1x + 1)$
\item $E = - 10x^{2} + x(5x - 1)$
\item $F = 10(x + 5)(x - 10)$
\end{enumerate}
\end{multicols}
\end{exercise}
\begin{solution}
\begin{enumerate}
\item
\begin{align*}
A &= (- 8x - 7)(- 1x - 7)\\&= - 8x \times - x - 8x \times - 7 - 7 \times - x - 7 \times - 7\\&= - 8 \times - 1 \times x^{1 + 1} - 7 \times - 8 \times x - 7 \times - 1 \times x + 49\\&= 56x + 7x + 8x^{2} + 49\\&= (56 + 7) \times x + 8x^{2} + 49\\&= 8x^{2} + 63x + 49
\end{align*}
\item
\begin{align*}
B &= (5x - 1)(- 2x - 1)\\&= 5x \times - 2x + 5x \times - 1 - 1 \times - 2x - 1 \times - 1\\&= 5 \times - 2 \times x^{1 + 1} - 1 \times 5 \times x - 1 \times - 2 \times x + 1\\&= - 5x + 2x - 10x^{2} + 1\\&= (- 5 + 2) \times x - 10x^{2} + 1\\&= - 10x^{2} - 3x + 1
\end{align*}
\item
\begin{align*}
C &= (- 3x + 7)^{2}\\&= (- 3x + 7)(- 3x + 7)\\&= - 3x \times - 3x - 3x \times 7 + 7 \times - 3x + 7 \times 7\\&= - 3 \times - 3 \times x^{1 + 1} + 7 \times - 3 \times x + 7 \times - 3 \times x + 49\\&= - 21x - 21x + 9x^{2} + 49\\&= (- 21 - 21) \times x + 9x^{2} + 49\\&= 9x^{2} - 42x + 49
\end{align*}
\item
\begin{align*}
D &= - 4 + x(1x + 1)\\&= - 4 + x \times x + x \times 1\\&= x^{2} + x - 4
\end{align*}
\item
\begin{align*}
E &= - 10x^{2} + x(5x - 1)\\&= - 10x^{2} + x \times 5x + x \times - 1\\&= - 10x^{2} + 5x^{2} - x\\&= - 10x^{2} + 5x^{2} - x\\&= (- 10 + 5) \times x^{2} - x\\&= - 5x^{2} - x
\end{align*}
\item
\begin{align*}
F &= 10(x + 5)(x - 10)\\&= (10x + 10 \times 5)(x - 10)\\&= (10x + 50)(x - 10)\\&= 10x \times x + 10x \times - 10 + 50x + 50 \times - 10\\&= - 10 \times 10 \times x - 500 + 10x^{2} + 50x\\&= - 100x - 500 + 10x^{2} + 50x\\&= 10x^{2} - 100x + 50x - 500\\&= 10x^{2} + (- 100 + 50) \times x - 500\\&= 10x^{2} - 50x - 500
\end{align*}
\end{enumerate}
\end{solution}
\begin{exercise}[subtitle={Étude de fonctions}]
Soit $f(x) = - x^{2} - 5x + 14$ une fonction définie sur $\R$.
\begin{enumerate}
\item Calculer les valeurs suivantes
\[
f(1) \qquad f(-2)
\]
\item Dériver la fonction $f$
\item Étudier le signe de $f'$ puis en déduire les variations de $f$.
\item Est-ce que $f$ admet un maximum? un minimum? Calculer sa valeur.
\end{enumerate}
\end{exercise}
\begin{solution}
\begin{enumerate}
\item On remplace $x$ par les valeurs demandées
\[
f(1) = - 1 \times 1^{2} - 5 \times 1 + 14=- 1 \times 1 - 5 + 14=- 1 + 9=8
\]
\[
f(-1) = - 1 \times - 1^{2} - 5 \times - 1 + 14=- 1 \times 1 + 5 + 14=- 1 + 19=18
\]
\item Pas de solutions automatiques.
\item Pas de solutions automatiques.
\end{enumerate}
\end{solution}
%\printsolutionstype{exercise}
\end{document}
%%% Local Variables:
%%% mode: latex
%%% TeX-master: "master"
%%% End:

View File

@ -0,0 +1,141 @@
\documentclass[a5paper,10pt]{article}
\usepackage{myXsim}
\usepackage{tasks}
% Title Page
\title{DM1 \hfill FILALI Zakaria}
\tribe{TST}
\date{Toussain 2020}
\begin{document}
\maketitle
\begin{exercise}[subtitle={Fractions}]
Faire les calculs avec les fraction suivants
\begin{multicols}{3}
\begin{enumerate}
\item $A = \dfrac{- 2}{6} - \dfrac{2}{6}$
\item $B = \dfrac{9}{4} - \dfrac{2}{20}$
\item $C = \dfrac{- 4}{8} + \dfrac{- 6}{7}$
\item $D = \dfrac{- 7}{6} - 1$
\item $E = \dfrac{5}{2} \times \dfrac{5}{1}$
\item $F = \dfrac{1}{9} \times 4$
\end{enumerate}
\end{multicols}
\end{exercise}
\begin{solution}
\begin{enumerate}
\item
\[
\dfrac{- 2}{6} - \dfrac{2}{6}=\dfrac{- 2}{6} - \dfrac{2}{6}=\dfrac{- 2 - 2}{6}=\dfrac{- 2 - 2}{6}=\dfrac{- 4}{6}
\]
\item
\[
\dfrac{9}{4} - \dfrac{2}{20}=\dfrac{9}{4} - \dfrac{2}{20}=\dfrac{9 \times 5}{4 \times 5} - \dfrac{2}{20}=\dfrac{45}{20} - \dfrac{2}{20}=\dfrac{45 - 2}{20}=\dfrac{45 - 2}{20}=\dfrac{43}{20}
\]
\item
\[
\dfrac{- 4}{8} + \dfrac{- 6}{7}=\dfrac{- 4 \times 7}{8 \times 7} + \dfrac{- 6 \times 8}{7 \times 8}=\dfrac{- 28}{56} + \dfrac{- 48}{56}=\dfrac{- 28 - 48}{56}=\dfrac{- 76}{56}
\]
\item
\[
\dfrac{- 7}{6} - 1=\dfrac{- 7}{6} + \dfrac{- 1}{1}=\dfrac{- 7}{6} + \dfrac{- 1 \times 6}{1 \times 6}=\dfrac{- 7}{6} + \dfrac{- 6}{6}=\dfrac{- 7 - 6}{6}=\dfrac{- 13}{6}
\]
\item
\[
\dfrac{5}{2} \times \dfrac{5}{1}=\dfrac{5 \times 5}{2 \times 1}=\dfrac{25}{2}
\]
\item
\[
\dfrac{1}{9} \times 4=\dfrac{1 \times 4}{9}=\dfrac{4}{9}
\]
\end{enumerate}
\end{solution}
\begin{exercise}[subtitle={Développer réduire}]
Développer puis réduire les expressions suivantes
\begin{multicols}{2}
\begin{enumerate}
\item $A = (8x + 2)(- 8x + 2)$
\item $B = (- 5x - 8)(9x - 8)$
\item $C = (- 1x - 1)^{2}$
\item $D = - 7 + x(7x + 7)$
\item $E = 5x^{2} + x(- 6x + 8)$
\item $F = - 10(x + 5)(x - 10)$
\end{enumerate}
\end{multicols}
\end{exercise}
\begin{solution}
\begin{enumerate}
\item
\begin{align*}
A &= (8x + 2)(- 8x + 2)\\&= 8x \times - 8x + 8x \times 2 + 2 \times - 8x + 2 \times 2\\&= 8 \times - 8 \times x^{1 + 1} + 2 \times 8 \times x + 2 \times - 8 \times x + 4\\&= 16x - 16x - 64x^{2} + 4\\&= (16 - 16) \times x - 64x^{2} + 4\\&= 0x - 64x^{2} + 4\\&= - 64x^{2} + 4
\end{align*}
\item
\begin{align*}
B &= (- 5x - 8)(9x - 8)\\&= - 5x \times 9x - 5x \times - 8 - 8 \times 9x - 8 \times - 8\\&= - 5 \times 9 \times x^{1 + 1} - 8 \times - 5 \times x - 8 \times 9 \times x + 64\\&= 40x - 72x - 45x^{2} + 64\\&= (40 - 72) \times x - 45x^{2} + 64\\&= - 45x^{2} - 32x + 64
\end{align*}
\item
\begin{align*}
C &= (- 1x - 1)^{2}\\&= (- x - 1)(- x - 1)\\&= - x \times - x - x \times - 1 - 1 \times - x - 1 \times - 1\\&= - 1 \times - 1 \times x^{1 + 1} - 1 \times - 1 \times x - 1 \times - 1 \times x + 1\\&= x^{2} + 2x + 1
\end{align*}
\item
\begin{align*}
D &= - 7 + x(7x + 7)\\&= - 7 + x \times 7x + x \times 7\\&= 7x^{2} + 7x - 7
\end{align*}
\item
\begin{align*}
E &= 5x^{2} + x(- 6x + 8)\\&= 5x^{2} + x \times - 6x + x \times 8\\&= 5x^{2} - 6x^{2} + 8x\\&= 5x^{2} - 6x^{2} + 8x\\&= (5 - 6) \times x^{2} + 8x\\&= - x^{2} + 8x
\end{align*}
\item
\begin{align*}
F &= - 10(x + 5)(x - 10)\\&= (- 10x - 10 \times 5)(x - 10)\\&= (- 10x - 50)(x - 10)\\&= - 10x \times x - 10x \times - 10 - 50x - 50 \times - 10\\&= - 10 \times - 10 \times x + 500 - 10x^{2} - 50x\\&= 100x + 500 - 10x^{2} - 50x\\&= - 10x^{2} + 100x - 50x + 500\\&= - 10x^{2} + (100 - 50) \times x + 500\\&= - 10x^{2} + 50x + 500
\end{align*}
\end{enumerate}
\end{solution}
\begin{exercise}[subtitle={Étude de fonctions}]
Soit $f(x) = - 3x^{2} - 45x - 168$ une fonction définie sur $\R$.
\begin{enumerate}
\item Calculer les valeurs suivantes
\[
f(1) \qquad f(-2)
\]
\item Dériver la fonction $f$
\item Étudier le signe de $f'$ puis en déduire les variations de $f$.
\item Est-ce que $f$ admet un maximum? un minimum? Calculer sa valeur.
\end{enumerate}
\end{exercise}
\begin{solution}
\begin{enumerate}
\item On remplace $x$ par les valeurs demandées
\[
f(1) = - 3 \times 1^{2} - 45 \times 1 - 168=- 3 \times 1 - 45 - 168=- 3 - 213=- 216
\]
\[
f(-1) = - 3 \times - 1^{2} - 45 \times - 1 - 168=- 3 \times 1 + 45 - 168=- 3 - 123=- 126
\]
\item Pas de solutions automatiques.
\item Pas de solutions automatiques.
\end{enumerate}
\end{solution}
%\printsolutionstype{exercise}
\end{document}
%%% Local Variables:
%%% mode: latex
%%% TeX-master: "master"
%%% End:

View File

@ -0,0 +1,141 @@
\documentclass[a5paper,10pt]{article}
\usepackage{myXsim}
\usepackage{tasks}
% Title Page
\title{DM1 \hfill FOIGNY Romain}
\tribe{TST}
\date{Toussain 2020}
\begin{document}
\maketitle
\begin{exercise}[subtitle={Fractions}]
Faire les calculs avec les fraction suivants
\begin{multicols}{3}
\begin{enumerate}
\item $A = \dfrac{- 6}{8} - \dfrac{2}{8}$
\item $B = \dfrac{- 4}{6} - \dfrac{10}{60}$
\item $C = \dfrac{8}{10} + \dfrac{- 3}{9}$
\item $D = \dfrac{6}{7} - 1$
\item $E = \dfrac{7}{4} \times \dfrac{6}{3}$
\item $F = \dfrac{- 3}{5} \times - 8$
\end{enumerate}
\end{multicols}
\end{exercise}
\begin{solution}
\begin{enumerate}
\item
\[
\dfrac{- 6}{8} - \dfrac{2}{8}=\dfrac{- 6}{8} - \dfrac{2}{8}=\dfrac{- 6 - 2}{8}=\dfrac{- 6 - 2}{8}=\dfrac{- 8}{8}
\]
\item
\[
\dfrac{- 4}{6} - \dfrac{10}{60}=\dfrac{- 4}{6} - \dfrac{10}{60}=\dfrac{- 4 \times 10}{6 \times 10} - \dfrac{10}{60}=\dfrac{- 40}{60} - \dfrac{10}{60}=\dfrac{- 40 - 10}{60}=\dfrac{- 40 - 10}{60}=\dfrac{- 50}{60}
\]
\item
\[
\dfrac{8}{10} + \dfrac{- 3}{9}=\dfrac{8 \times 9}{10 \times 9} + \dfrac{- 3 \times 10}{9 \times 10}=\dfrac{72}{90} + \dfrac{- 30}{90}=\dfrac{72 - 30}{90}=\dfrac{42}{90}
\]
\item
\[
\dfrac{6}{7} - 1=\dfrac{6}{7} + \dfrac{- 1}{1}=\dfrac{6}{7} + \dfrac{- 1 \times 7}{1 \times 7}=\dfrac{6}{7} + \dfrac{- 7}{7}=\dfrac{6 - 7}{7}=\dfrac{- 1}{7}
\]
\item
\[
\dfrac{7}{4} \times \dfrac{6}{3}=\dfrac{7 \times 6}{4 \times 3}=\dfrac{42}{12}
\]
\item
\[
\dfrac{- 3}{5} \times - 8=\dfrac{- 3 \times - 8}{5}=\dfrac{24}{5}
\]
\end{enumerate}
\end{solution}
\begin{exercise}[subtitle={Développer réduire}]
Développer puis réduire les expressions suivantes
\begin{multicols}{2}
\begin{enumerate}
\item $A = (5x + 10)(- 1x + 10)$
\item $B = (- 7x + 9)(5x + 9)$
\item $C = (- 7x - 10)^{2}$
\item $D = - 9 + x(- 1x - 2)$
\item $E = - 4x^{2} + x(- 5x + 5)$
\item $F = 5(x + 10)(x + 9)$
\end{enumerate}
\end{multicols}
\end{exercise}
\begin{solution}
\begin{enumerate}
\item
\begin{align*}
A &= (5x + 10)(- 1x + 10)\\&= 5x \times - x + 5x \times 10 + 10 \times - x + 10 \times 10\\&= 5 \times - 1 \times x^{1 + 1} + 10 \times 5 \times x + 10 \times - 1 \times x + 100\\&= 50x - 10x - 5x^{2} + 100\\&= (50 - 10) \times x - 5x^{2} + 100\\&= - 5x^{2} + 40x + 100
\end{align*}
\item
\begin{align*}
B &= (- 7x + 9)(5x + 9)\\&= - 7x \times 5x - 7x \times 9 + 9 \times 5x + 9 \times 9\\&= - 7 \times 5 \times x^{1 + 1} + 9 \times - 7 \times x + 9 \times 5 \times x + 81\\&= - 63x + 45x - 35x^{2} + 81\\&= (- 63 + 45) \times x - 35x^{2} + 81\\&= - 35x^{2} - 18x + 81
\end{align*}
\item
\begin{align*}
C &= (- 7x - 10)^{2}\\&= (- 7x - 10)(- 7x - 10)\\&= - 7x \times - 7x - 7x \times - 10 - 10 \times - 7x - 10 \times - 10\\&= - 7 \times - 7 \times x^{1 + 1} - 10 \times - 7 \times x - 10 \times - 7 \times x + 100\\&= 70x + 70x + 49x^{2} + 100\\&= (70 + 70) \times x + 49x^{2} + 100\\&= 49x^{2} + 140x + 100
\end{align*}
\item
\begin{align*}
D &= - 9 + x(- 1x - 2)\\&= - 9 + x \times - x + x \times - 2\\&= - x^{2} - 2x - 9
\end{align*}
\item
\begin{align*}
E &= - 4x^{2} + x(- 5x + 5)\\&= - 4x^{2} + x \times - 5x + x \times 5\\&= - 4x^{2} - 5x^{2} + 5x\\&= - 4x^{2} - 5x^{2} + 5x\\&= (- 4 - 5) \times x^{2} + 5x\\&= - 9x^{2} + 5x
\end{align*}
\item
\begin{align*}
F &= 5(x + 10)(x + 9)\\&= (5x + 5 \times 10)(x + 9)\\&= (5x + 50)(x + 9)\\&= 5x \times x + 5x \times 9 + 50x + 50 \times 9\\&= 9 \times 5 \times x + 450 + 5x^{2} + 50x\\&= 45x + 450 + 5x^{2} + 50x\\&= 5x^{2} + 45x + 50x + 450\\&= 5x^{2} + (45 + 50) \times x + 450\\&= 5x^{2} + 95x + 450
\end{align*}
\end{enumerate}
\end{solution}
\begin{exercise}[subtitle={Étude de fonctions}]
Soit $f(x) = 7x^{2} - 56x - 63$ une fonction définie sur $\R$.
\begin{enumerate}
\item Calculer les valeurs suivantes
\[
f(1) \qquad f(-2)
\]
\item Dériver la fonction $f$
\item Étudier le signe de $f'$ puis en déduire les variations de $f$.
\item Est-ce que $f$ admet un maximum? un minimum? Calculer sa valeur.
\end{enumerate}
\end{exercise}
\begin{solution}
\begin{enumerate}
\item On remplace $x$ par les valeurs demandées
\[
f(1) = 7 \times 1^{2} - 56 \times 1 - 63=7 \times 1 - 56 - 63=7 - 119=- 112
\]
\[
f(-1) = 7 \times - 1^{2} - 56 \times - 1 - 63=7 \times 1 + 56 - 63=7 - 7=0
\]
\item Pas de solutions automatiques.
\item Pas de solutions automatiques.
\end{enumerate}
\end{solution}
%\printsolutionstype{exercise}
\end{document}
%%% Local Variables:
%%% mode: latex
%%% TeX-master: "master"
%%% End:

View File

@ -0,0 +1,141 @@
\documentclass[a5paper,10pt]{article}
\usepackage{myXsim}
\usepackage{tasks}
% Title Page
\title{DM1 \hfill HIPOLITO DA SILVA Andréa}
\tribe{TST}
\date{Toussain 2020}
\begin{document}
\maketitle
\begin{exercise}[subtitle={Fractions}]
Faire les calculs avec les fraction suivants
\begin{multicols}{3}
\begin{enumerate}
\item $A = \dfrac{6}{10} - \dfrac{9}{10}$
\item $B = \dfrac{9}{3} - \dfrac{- 7}{6}$
\item $C = \dfrac{7}{2} + \dfrac{- 1}{1}$
\item $D = \dfrac{- 8}{4} - 1$
\item $E = \dfrac{4}{5} \times \dfrac{6}{4}$
\item $F = \dfrac{- 9}{5} \times - 7$
\end{enumerate}
\end{multicols}
\end{exercise}
\begin{solution}
\begin{enumerate}
\item
\[
\dfrac{6}{10} - \dfrac{9}{10}=\dfrac{6}{10} - \dfrac{9}{10}=\dfrac{6 - 9}{10}=\dfrac{6 - 9}{10}=\dfrac{- 3}{10}
\]
\item
\[
\dfrac{9}{3} - \dfrac{- 7}{6}=\dfrac{9}{3} + \dfrac{7}{6}=\dfrac{9 \times 2}{3 \times 2} + \dfrac{7}{6}=\dfrac{18}{6} + \dfrac{7}{6}=\dfrac{18 + 7}{6}=\dfrac{25}{6}
\]
\item
\[
\dfrac{7}{2} + \dfrac{- 1}{1}=\dfrac{7}{2} + \dfrac{- 1 \times 2}{1 \times 2}=\dfrac{7}{2} + \dfrac{- 2}{2}=\dfrac{7 - 2}{2}=\dfrac{5}{2}
\]
\item
\[
\dfrac{- 8}{4} - 1=\dfrac{- 8}{4} + \dfrac{- 1}{1}=\dfrac{- 8}{4} + \dfrac{- 1 \times 4}{1 \times 4}=\dfrac{- 8}{4} + \dfrac{- 4}{4}=\dfrac{- 8 - 4}{4}=\dfrac{- 12}{4}
\]
\item
\[
\dfrac{4}{5} \times \dfrac{6}{4}=\dfrac{4 \times 6}{5 \times 4}=\dfrac{24}{20}
\]
\item
\[
\dfrac{- 9}{5} \times - 7=\dfrac{- 9 \times - 7}{5}=\dfrac{63}{5}
\]
\end{enumerate}
\end{solution}
\begin{exercise}[subtitle={Développer réduire}]
Développer puis réduire les expressions suivantes
\begin{multicols}{2}
\begin{enumerate}
\item $A = (10x + 4)(- 3x + 4)$
\item $B = (- 8x - 9)(3x - 9)$
\item $C = (- 10x + 1)^{2}$
\item $D = - 2 + x(2x + 6)$
\item $E = 10x^{2} + x(- 1x + 7)$
\item $F = - 7(x + 9)(x - 5)$
\end{enumerate}
\end{multicols}
\end{exercise}
\begin{solution}
\begin{enumerate}
\item
\begin{align*}
A &= (10x + 4)(- 3x + 4)\\&= 10x \times - 3x + 10x \times 4 + 4 \times - 3x + 4 \times 4\\&= 10 \times - 3 \times x^{1 + 1} + 4 \times 10 \times x + 4 \times - 3 \times x + 16\\&= 40x - 12x - 30x^{2} + 16\\&= (40 - 12) \times x - 30x^{2} + 16\\&= - 30x^{2} + 28x + 16
\end{align*}
\item
\begin{align*}
B &= (- 8x - 9)(3x - 9)\\&= - 8x \times 3x - 8x \times - 9 - 9 \times 3x - 9 \times - 9\\&= - 8 \times 3 \times x^{1 + 1} - 9 \times - 8 \times x - 9 \times 3 \times x + 81\\&= 72x - 27x - 24x^{2} + 81\\&= (72 - 27) \times x - 24x^{2} + 81\\&= - 24x^{2} + 45x + 81
\end{align*}
\item
\begin{align*}
C &= (- 10x + 1)^{2}\\&= (- 10x + 1)(- 10x + 1)\\&= - 10x \times - 10x - 10x \times 1 + 1 \times - 10x + 1 \times 1\\&= - 10 \times - 10 \times x^{1 + 1} - 10x - 10x + 1\\&= 100x^{2} - 10x - 10x + 1\\&= 100x^{2} + (- 10 - 10) \times x + 1\\&= 100x^{2} - 20x + 1
\end{align*}
\item
\begin{align*}
D &= - 2 + x(2x + 6)\\&= - 2 + x \times 2x + x \times 6\\&= 2x^{2} + 6x - 2
\end{align*}
\item
\begin{align*}
E &= 10x^{2} + x(- 1x + 7)\\&= 10x^{2} + x \times - x + x \times 7\\&= 10x^{2} - x^{2} + 7x\\&= 10x^{2} - x^{2} + 7x\\&= (10 - 1) \times x^{2} + 7x\\&= 9x^{2} + 7x
\end{align*}
\item
\begin{align*}
F &= - 7(x + 9)(x - 5)\\&= (- 7x - 7 \times 9)(x - 5)\\&= (- 7x - 63)(x - 5)\\&= - 7x \times x - 7x \times - 5 - 63x - 63 \times - 5\\&= - 5 \times - 7 \times x + 315 - 7x^{2} - 63x\\&= 35x + 315 - 7x^{2} - 63x\\&= - 7x^{2} + 35x - 63x + 315\\&= - 7x^{2} + (35 - 63) \times x + 315\\&= - 7x^{2} - 28x + 315
\end{align*}
\end{enumerate}
\end{solution}
\begin{exercise}[subtitle={Étude de fonctions}]
Soit $f(x) = x^{2} + 5x - 6$ une fonction définie sur $\R$.
\begin{enumerate}
\item Calculer les valeurs suivantes
\[
f(1) \qquad f(-2)
\]
\item Dériver la fonction $f$
\item Étudier le signe de $f'$ puis en déduire les variations de $f$.
\item Est-ce que $f$ admet un maximum? un minimum? Calculer sa valeur.
\end{enumerate}
\end{exercise}
\begin{solution}
\begin{enumerate}
\item On remplace $x$ par les valeurs demandées
\[
f(1) = 1^{2} + 5 \times 1 - 6=1 + 5 - 6=1 - 1=0
\]
\[
f(-1) = - 1^{2} + 5 \times - 1 - 6=1 - 5 - 6=1 - 11=- 10
\]
\item Pas de solutions automatiques.
\item Pas de solutions automatiques.
\end{enumerate}
\end{solution}
%\printsolutionstype{exercise}
\end{document}
%%% Local Variables:
%%% mode: latex
%%% TeX-master: "master"
%%% End:

View File

@ -0,0 +1,141 @@
\documentclass[a5paper,10pt]{article}
\usepackage{myXsim}
\usepackage{tasks}
% Title Page
\title{DM1 \hfill HUMBERT Rayan}
\tribe{TST}
\date{Toussain 2020}
\begin{document}
\maketitle
\begin{exercise}[subtitle={Fractions}]
Faire les calculs avec les fraction suivants
\begin{multicols}{3}
\begin{enumerate}
\item $A = \dfrac{- 9}{5} - \dfrac{7}{5}$
\item $B = \dfrac{- 1}{6} - \dfrac{- 6}{54}$
\item $C = \dfrac{4}{10} + \dfrac{10}{9}$
\item $D = \dfrac{9}{5} + 8$
\item $E = \dfrac{- 2}{4} \times \dfrac{8}{3}$
\item $F = \dfrac{- 7}{4} \times 10$
\end{enumerate}
\end{multicols}
\end{exercise}
\begin{solution}
\begin{enumerate}
\item
\[
\dfrac{- 9}{5} - \dfrac{7}{5}=\dfrac{- 9}{5} - \dfrac{7}{5}=\dfrac{- 9 - 7}{5}=\dfrac{- 9 - 7}{5}=\dfrac{- 16}{5}
\]
\item
\[
\dfrac{- 1}{6} - \dfrac{- 6}{54}=\dfrac{- 1}{6} + \dfrac{6}{54}=\dfrac{- 1 \times 9}{6 \times 9} + \dfrac{6}{54}=\dfrac{- 9}{54} + \dfrac{6}{54}=\dfrac{- 9 + 6}{54}=\dfrac{- 3}{54}
\]
\item
\[
\dfrac{4}{10} + \dfrac{10}{9}=\dfrac{4 \times 9}{10 \times 9} + \dfrac{10 \times 10}{9 \times 10}=\dfrac{36}{90} + \dfrac{100}{90}=\dfrac{36 + 100}{90}=\dfrac{136}{90}
\]
\item
\[
\dfrac{9}{5} + 8=\dfrac{9}{5} + \dfrac{8}{1}=\dfrac{9}{5} + \dfrac{8 \times 5}{1 \times 5}=\dfrac{9}{5} + \dfrac{40}{5}=\dfrac{9 + 40}{5}=\dfrac{49}{5}
\]
\item
\[
\dfrac{- 2}{4} \times \dfrac{8}{3}=\dfrac{- 2 \times 8}{4 \times 3}=\dfrac{- 16}{12}
\]
\item
\[
\dfrac{- 7}{4} \times 10=\dfrac{- 7 \times 10}{4}=\dfrac{- 70}{4}
\]
\end{enumerate}
\end{solution}
\begin{exercise}[subtitle={Développer réduire}]
Développer puis réduire les expressions suivantes
\begin{multicols}{2}
\begin{enumerate}
\item $A = (3x - 5)(- 3x - 5)$
\item $B = (3x + 4)(- 10x + 4)$
\item $C = (3x - 3)^{2}$
\item $D = 8 + x(- 6x + 5)$
\item $E = - 3x^{2} + x(2x - 3)$
\item $F = 5(x - 9)(x + 4)$
\end{enumerate}
\end{multicols}
\end{exercise}
\begin{solution}
\begin{enumerate}
\item
\begin{align*}
A &= (3x - 5)(- 3x - 5)\\&= 3x \times - 3x + 3x \times - 5 - 5 \times - 3x - 5 \times - 5\\&= 3 \times - 3 \times x^{1 + 1} - 5 \times 3 \times x - 5 \times - 3 \times x + 25\\&= - 15x + 15x - 9x^{2} + 25\\&= (- 15 + 15) \times x - 9x^{2} + 25\\&= 0x - 9x^{2} + 25\\&= - 9x^{2} + 25
\end{align*}
\item
\begin{align*}
B &= (3x + 4)(- 10x + 4)\\&= 3x \times - 10x + 3x \times 4 + 4 \times - 10x + 4 \times 4\\&= 3 \times - 10 \times x^{1 + 1} + 4 \times 3 \times x + 4 \times - 10 \times x + 16\\&= 12x - 40x - 30x^{2} + 16\\&= (12 - 40) \times x - 30x^{2} + 16\\&= - 30x^{2} - 28x + 16
\end{align*}
\item
\begin{align*}
C &= (3x - 3)^{2}\\&= (3x - 3)(3x - 3)\\&= 3x \times 3x + 3x \times - 3 - 3 \times 3x - 3 \times - 3\\&= 3 \times 3 \times x^{1 + 1} - 3 \times 3 \times x - 3 \times 3 \times x + 9\\&= - 9x - 9x + 9x^{2} + 9\\&= (- 9 - 9) \times x + 9x^{2} + 9\\&= 9x^{2} - 18x + 9
\end{align*}
\item
\begin{align*}
D &= 8 + x(- 6x + 5)\\&= 8 + x \times - 6x + x \times 5\\&= - 6x^{2} + 5x + 8
\end{align*}
\item
\begin{align*}
E &= - 3x^{2} + x(2x - 3)\\&= - 3x^{2} + x \times 2x + x \times - 3\\&= - 3x^{2} + 2x^{2} - 3x\\&= - 3x^{2} + 2x^{2} - 3x\\&= (- 3 + 2) \times x^{2} - 3x\\&= - x^{2} - 3x
\end{align*}
\item
\begin{align*}
F &= 5(x - 9)(x + 4)\\&= (5x + 5 \times - 9)(x + 4)\\&= (5x - 45)(x + 4)\\&= 5x \times x + 5x \times 4 - 45x - 45 \times 4\\&= 4 \times 5 \times x - 180 + 5x^{2} - 45x\\&= 20x - 180 + 5x^{2} - 45x\\&= 5x^{2} + 20x - 45x - 180\\&= 5x^{2} + (20 - 45) \times x - 180\\&= 5x^{2} - 25x - 180
\end{align*}
\end{enumerate}
\end{solution}
\begin{exercise}[subtitle={Étude de fonctions}]
Soit $f(x) = 9x^{2} + 153x + 648$ une fonction définie sur $\R$.
\begin{enumerate}
\item Calculer les valeurs suivantes
\[
f(1) \qquad f(-2)
\]
\item Dériver la fonction $f$
\item Étudier le signe de $f'$ puis en déduire les variations de $f$.
\item Est-ce que $f$ admet un maximum? un minimum? Calculer sa valeur.
\end{enumerate}
\end{exercise}
\begin{solution}
\begin{enumerate}
\item On remplace $x$ par les valeurs demandées
\[
f(1) = 9 \times 1^{2} + 153 \times 1 + 648=9 \times 1 + 153 + 648=9 + 801=810
\]
\[
f(-1) = 9 \times - 1^{2} + 153 \times - 1 + 648=9 \times 1 - 153 + 648=9 + 495=504
\]
\item Pas de solutions automatiques.
\item Pas de solutions automatiques.
\end{enumerate}
\end{solution}
%\printsolutionstype{exercise}
\end{document}
%%% Local Variables:
%%% mode: latex
%%% TeX-master: "master"
%%% End:

View File

@ -0,0 +1,141 @@
\documentclass[a5paper,10pt]{article}
\usepackage{myXsim}
\usepackage{tasks}
% Title Page
\title{DM1 \hfill MASSON Grace}
\tribe{TST}
\date{Toussain 2020}
\begin{document}
\maketitle
\begin{exercise}[subtitle={Fractions}]
Faire les calculs avec les fraction suivants
\begin{multicols}{3}
\begin{enumerate}
\item $A = \dfrac{- 7}{8} - \dfrac{- 7}{8}$
\item $B = \dfrac{- 6}{7} - \dfrac{- 5}{70}$
\item $C = \dfrac{- 2}{2} + \dfrac{4}{1}$
\item $D = \dfrac{3}{10} + 5$
\item $E = \dfrac{8}{3} \times \dfrac{- 10}{2}$
\item $F = \dfrac{4}{9} \times - 4$
\end{enumerate}
\end{multicols}
\end{exercise}
\begin{solution}
\begin{enumerate}
\item
\[
\dfrac{- 7}{8} - \dfrac{- 7}{8}=\dfrac{- 7}{8} + \dfrac{7}{8}=\dfrac{- 7 + 7}{8}=\dfrac{0}{8}
\]
\item
\[
\dfrac{- 6}{7} - \dfrac{- 5}{70}=\dfrac{- 6}{7} + \dfrac{5}{70}=\dfrac{- 6 \times 10}{7 \times 10} + \dfrac{5}{70}=\dfrac{- 60}{70} + \dfrac{5}{70}=\dfrac{- 60 + 5}{70}=\dfrac{- 55}{70}
\]
\item
\[
\dfrac{- 2}{2} + \dfrac{4}{1}=\dfrac{- 2}{2} + \dfrac{4 \times 2}{1 \times 2}=\dfrac{- 2}{2} + \dfrac{8}{2}=\dfrac{- 2 + 8}{2}=\dfrac{6}{2}
\]
\item
\[
\dfrac{3}{10} + 5=\dfrac{3}{10} + \dfrac{5}{1}=\dfrac{3}{10} + \dfrac{5 \times 10}{1 \times 10}=\dfrac{3}{10} + \dfrac{50}{10}=\dfrac{3 + 50}{10}=\dfrac{53}{10}
\]
\item
\[
\dfrac{8}{3} \times \dfrac{- 10}{2}=\dfrac{8 \times - 10}{3 \times 2}=\dfrac{- 80}{6}
\]
\item
\[
\dfrac{4}{9} \times - 4=\dfrac{4 \times - 4}{9}=\dfrac{- 16}{9}
\]
\end{enumerate}
\end{solution}
\begin{exercise}[subtitle={Développer réduire}]
Développer puis réduire les expressions suivantes
\begin{multicols}{2}
\begin{enumerate}
\item $A = (- 10x + 7)(- 1x + 7)$
\item $B = (2x - 10)(- 4x - 10)$
\item $C = (- 2x + 1)^{2}$
\item $D = - 6 + x(- 8x - 9)$
\item $E = - 1x^{2} + x(- 6x + 6)$
\item $F = - 2(x + 5)(x - 5)$
\end{enumerate}
\end{multicols}
\end{exercise}
\begin{solution}
\begin{enumerate}
\item
\begin{align*}
A &= (- 10x + 7)(- 1x + 7)\\&= - 10x \times - x - 10x \times 7 + 7 \times - x + 7 \times 7\\&= - 10 \times - 1 \times x^{1 + 1} + 7 \times - 10 \times x + 7 \times - 1 \times x + 49\\&= - 70x - 7x + 10x^{2} + 49\\&= (- 70 - 7) \times x + 10x^{2} + 49\\&= 10x^{2} - 77x + 49
\end{align*}
\item
\begin{align*}
B &= (2x - 10)(- 4x - 10)\\&= 2x \times - 4x + 2x \times - 10 - 10 \times - 4x - 10 \times - 10\\&= 2 \times - 4 \times x^{1 + 1} - 10 \times 2 \times x - 10 \times - 4 \times x + 100\\&= - 20x + 40x - 8x^{2} + 100\\&= (- 20 + 40) \times x - 8x^{2} + 100\\&= - 8x^{2} + 20x + 100
\end{align*}
\item
\begin{align*}
C &= (- 2x + 1)^{2}\\&= (- 2x + 1)(- 2x + 1)\\&= - 2x \times - 2x - 2x \times 1 + 1 \times - 2x + 1 \times 1\\&= - 2 \times - 2 \times x^{1 + 1} - 2x - 2x + 1\\&= 4x^{2} - 2x - 2x + 1\\&= 4x^{2} + (- 2 - 2) \times x + 1\\&= 4x^{2} - 4x + 1
\end{align*}
\item
\begin{align*}
D &= - 6 + x(- 8x - 9)\\&= - 6 + x \times - 8x + x \times - 9\\&= - 8x^{2} - 9x - 6
\end{align*}
\item
\begin{align*}
E &= - 1x^{2} + x(- 6x + 6)\\&= - x^{2} + x \times - 6x + x \times 6\\&= - x^{2} - 6x^{2} + 6x\\&= - x^{2} - 6x^{2} + 6x\\&= (- 1 - 6) \times x^{2} + 6x\\&= - 7x^{2} + 6x
\end{align*}
\item
\begin{align*}
F &= - 2(x + 5)(x - 5)\\&= (- 2x - 2 \times 5)(x - 5)\\&= (- 2x - 10)(x - 5)\\&= - 2x \times x - 2x \times - 5 - 10x - 10 \times - 5\\&= - 5 \times - 2 \times x + 50 - 2x^{2} - 10x\\&= 10x + 50 - 2x^{2} - 10x\\&= - 2x^{2} + 10x - 10x + 50\\&= - 2x^{2} + (10 - 10) \times x + 50\\&= - 2x^{2} + 50 + 0x\\&= - 2x^{2} + 50
\end{align*}
\end{enumerate}
\end{solution}
\begin{exercise}[subtitle={Étude de fonctions}]
Soit $f(x) = - 6x^{2} - 66x - 144$ une fonction définie sur $\R$.
\begin{enumerate}
\item Calculer les valeurs suivantes
\[
f(1) \qquad f(-2)
\]
\item Dériver la fonction $f$
\item Étudier le signe de $f'$ puis en déduire les variations de $f$.
\item Est-ce que $f$ admet un maximum? un minimum? Calculer sa valeur.
\end{enumerate}
\end{exercise}
\begin{solution}
\begin{enumerate}
\item On remplace $x$ par les valeurs demandées
\[
f(1) = - 6 \times 1^{2} - 66 \times 1 - 144=- 6 \times 1 - 66 - 144=- 6 - 210=- 216
\]
\[
f(-1) = - 6 \times - 1^{2} - 66 \times - 1 - 144=- 6 \times 1 + 66 - 144=- 6 - 78=- 84
\]
\item Pas de solutions automatiques.
\item Pas de solutions automatiques.
\end{enumerate}
\end{solution}
%\printsolutionstype{exercise}
\end{document}
%%% Local Variables:
%%% mode: latex
%%% TeX-master: "master"
%%% End:

View File

@ -0,0 +1,141 @@
\documentclass[a5paper,10pt]{article}
\usepackage{myXsim}
\usepackage{tasks}
% Title Page
\title{DM1 \hfill MOKHTARI Nissrine}
\tribe{TST}
\date{Toussain 2020}
\begin{document}
\maketitle
\begin{exercise}[subtitle={Fractions}]
Faire les calculs avec les fraction suivants
\begin{multicols}{3}
\begin{enumerate}
\item $A = \dfrac{7}{9} - \dfrac{- 6}{9}$
\item $B = \dfrac{6}{3} - \dfrac{3}{27}$
\item $C = \dfrac{- 9}{7} + \dfrac{4}{6}$
\item $D = \dfrac{- 4}{5} + 7$
\item $E = \dfrac{2}{4} \times \dfrac{6}{3}$
\item $F = \dfrac{1}{2} \times 5$
\end{enumerate}
\end{multicols}
\end{exercise}
\begin{solution}
\begin{enumerate}
\item
\[
\dfrac{7}{9} - \dfrac{- 6}{9}=\dfrac{7}{9} + \dfrac{6}{9}=\dfrac{7 + 6}{9}=\dfrac{13}{9}
\]
\item
\[
\dfrac{6}{3} - \dfrac{3}{27}=\dfrac{6}{3} - \dfrac{3}{27}=\dfrac{6 \times 9}{3 \times 9} - \dfrac{3}{27}=\dfrac{54}{27} - \dfrac{3}{27}=\dfrac{54 - 3}{27}=\dfrac{54 - 3}{27}=\dfrac{51}{27}
\]
\item
\[
\dfrac{- 9}{7} + \dfrac{4}{6}=\dfrac{- 9 \times 6}{7 \times 6} + \dfrac{4 \times 7}{6 \times 7}=\dfrac{- 54}{42} + \dfrac{28}{42}=\dfrac{- 54 + 28}{42}=\dfrac{- 26}{42}
\]
\item
\[
\dfrac{- 4}{5} + 7=\dfrac{- 4}{5} + \dfrac{7}{1}=\dfrac{- 4}{5} + \dfrac{7 \times 5}{1 \times 5}=\dfrac{- 4}{5} + \dfrac{35}{5}=\dfrac{- 4 + 35}{5}=\dfrac{31}{5}
\]
\item
\[
\dfrac{2}{4} \times \dfrac{6}{3}=\dfrac{2 \times 6}{4 \times 3}=\dfrac{12}{12}
\]
\item
\[
\dfrac{1}{2} \times 5=\dfrac{1 \times 5}{2}=\dfrac{5}{2}
\]
\end{enumerate}
\end{solution}
\begin{exercise}[subtitle={Développer réduire}]
Développer puis réduire les expressions suivantes
\begin{multicols}{2}
\begin{enumerate}
\item $A = (8x + 8)(9x + 8)$
\item $B = (- 9x - 7)(3x - 7)$
\item $C = (8x - 10)^{2}$
\item $D = - 1 + x(- 3x - 7)$
\item $E = - 7x^{2} + x(3x + 1)$
\item $F = - 6(x + 9)(x - 4)$
\end{enumerate}
\end{multicols}
\end{exercise}
\begin{solution}
\begin{enumerate}
\item
\begin{align*}
A &= (8x + 8)(9x + 8)\\&= 8x \times 9x + 8x \times 8 + 8 \times 9x + 8 \times 8\\&= 8 \times 9 \times x^{1 + 1} + 8 \times 8 \times x + 8 \times 9 \times x + 64\\&= 64x + 72x + 72x^{2} + 64\\&= (64 + 72) \times x + 72x^{2} + 64\\&= 72x^{2} + 136x + 64
\end{align*}
\item
\begin{align*}
B &= (- 9x - 7)(3x - 7)\\&= - 9x \times 3x - 9x \times - 7 - 7 \times 3x - 7 \times - 7\\&= - 9 \times 3 \times x^{1 + 1} - 7 \times - 9 \times x - 7 \times 3 \times x + 49\\&= 63x - 21x - 27x^{2} + 49\\&= (63 - 21) \times x - 27x^{2} + 49\\&= - 27x^{2} + 42x + 49
\end{align*}
\item
\begin{align*}
C &= (8x - 10)^{2}\\&= (8x - 10)(8x - 10)\\&= 8x \times 8x + 8x \times - 10 - 10 \times 8x - 10 \times - 10\\&= 8 \times 8 \times x^{1 + 1} - 10 \times 8 \times x - 10 \times 8 \times x + 100\\&= - 80x - 80x + 64x^{2} + 100\\&= (- 80 - 80) \times x + 64x^{2} + 100\\&= 64x^{2} - 160x + 100
\end{align*}
\item
\begin{align*}
D &= - 1 + x(- 3x - 7)\\&= - 1 + x \times - 3x + x \times - 7\\&= - 3x^{2} - 7x - 1
\end{align*}
\item
\begin{align*}
E &= - 7x^{2} + x(3x + 1)\\&= - 7x^{2} + x \times 3x + x \times 1\\&= - 7x^{2} + 3x^{2} + x\\&= - 7x^{2} + 3x^{2} + x\\&= (- 7 + 3) \times x^{2} + x\\&= - 4x^{2} + x
\end{align*}
\item
\begin{align*}
F &= - 6(x + 9)(x - 4)\\&= (- 6x - 6 \times 9)(x - 4)\\&= (- 6x - 54)(x - 4)\\&= - 6x \times x - 6x \times - 4 - 54x - 54 \times - 4\\&= - 4 \times - 6 \times x + 216 - 6x^{2} - 54x\\&= 24x + 216 - 6x^{2} - 54x\\&= - 6x^{2} + 24x - 54x + 216\\&= - 6x^{2} + (24 - 54) \times x + 216\\&= - 6x^{2} - 30x + 216
\end{align*}
\end{enumerate}
\end{solution}
\begin{exercise}[subtitle={Étude de fonctions}]
Soit $f(x) = x^{2} + 6x - 7$ une fonction définie sur $\R$.
\begin{enumerate}
\item Calculer les valeurs suivantes
\[
f(1) \qquad f(-2)
\]
\item Dériver la fonction $f$
\item Étudier le signe de $f'$ puis en déduire les variations de $f$.
\item Est-ce que $f$ admet un maximum? un minimum? Calculer sa valeur.
\end{enumerate}
\end{exercise}
\begin{solution}
\begin{enumerate}
\item On remplace $x$ par les valeurs demandées
\[
f(1) = 1^{2} + 6 \times 1 - 7=1 + 6 - 7=1 - 1=0
\]
\[
f(-1) = - 1^{2} + 6 \times - 1 - 7=1 - 6 - 7=1 - 13=- 12
\]
\item Pas de solutions automatiques.
\item Pas de solutions automatiques.
\end{enumerate}
\end{solution}
%\printsolutionstype{exercise}
\end{document}
%%% Local Variables:
%%% mode: latex
%%% TeX-master: "master"
%%% End:

View File

@ -0,0 +1,141 @@
\documentclass[a5paper,10pt]{article}
\usepackage{myXsim}
\usepackage{tasks}
% Title Page
\title{DM1 \hfill MOUFAQ Amine}
\tribe{TST}
\date{Toussain 2020}
\begin{document}
\maketitle
\begin{exercise}[subtitle={Fractions}]
Faire les calculs avec les fraction suivants
\begin{multicols}{3}
\begin{enumerate}
\item $A = \dfrac{10}{2} - \dfrac{4}{2}$
\item $B = \dfrac{- 7}{3} - \dfrac{5}{15}$
\item $C = \dfrac{- 10}{6} + \dfrac{5}{5}$
\item $D = \dfrac{- 10}{9} + 10$
\item $E = \dfrac{4}{2} \times \dfrac{- 3}{1}$
\item $F = \dfrac{- 10}{3} \times 10$
\end{enumerate}
\end{multicols}
\end{exercise}
\begin{solution}
\begin{enumerate}
\item
\[
\dfrac{10}{2} - \dfrac{4}{2}=\dfrac{10}{2} - \dfrac{4}{2}=\dfrac{10 - 4}{2}=\dfrac{10 - 4}{2}=\dfrac{6}{2}
\]
\item
\[
\dfrac{- 7}{3} - \dfrac{5}{15}=\dfrac{- 7}{3} - \dfrac{5}{15}=\dfrac{- 7 \times 5}{3 \times 5} - \dfrac{5}{15}=\dfrac{- 35}{15} - \dfrac{5}{15}=\dfrac{- 35 - 5}{15}=\dfrac{- 35 - 5}{15}=\dfrac{- 40}{15}
\]
\item
\[
\dfrac{- 10}{6} + \dfrac{5}{5}=\dfrac{- 10 \times 5}{6 \times 5} + \dfrac{5 \times 6}{5 \times 6}=\dfrac{- 50}{30} + \dfrac{30}{30}=\dfrac{- 50 + 30}{30}=\dfrac{- 20}{30}
\]
\item
\[
\dfrac{- 10}{9} + 10=\dfrac{- 10}{9} + \dfrac{10}{1}=\dfrac{- 10}{9} + \dfrac{10 \times 9}{1 \times 9}=\dfrac{- 10}{9} + \dfrac{90}{9}=\dfrac{- 10 + 90}{9}=\dfrac{80}{9}
\]
\item
\[
\dfrac{4}{2} \times \dfrac{- 3}{1}=\dfrac{4 \times - 3}{2 \times 1}=\dfrac{- 12}{2}
\]
\item
\[
\dfrac{- 10}{3} \times 10=\dfrac{- 10 \times 10}{3}=\dfrac{- 100}{3}
\]
\end{enumerate}
\end{solution}
\begin{exercise}[subtitle={Développer réduire}]
Développer puis réduire les expressions suivantes
\begin{multicols}{2}
\begin{enumerate}
\item $A = (- 9x + 2)(- 8x + 2)$
\item $B = (- 7x - 7)(- 8x - 7)$
\item $C = (2x - 8)^{2}$
\item $D = - 6 + x(5x - 3)$
\item $E = 8x^{2} + x(- 4x - 7)$
\item $F = 5(x - 4)(x + 2)$
\end{enumerate}
\end{multicols}
\end{exercise}
\begin{solution}
\begin{enumerate}
\item
\begin{align*}
A &= (- 9x + 2)(- 8x + 2)\\&= - 9x \times - 8x - 9x \times 2 + 2 \times - 8x + 2 \times 2\\&= - 9 \times - 8 \times x^{1 + 1} + 2 \times - 9 \times x + 2 \times - 8 \times x + 4\\&= - 18x - 16x + 72x^{2} + 4\\&= (- 18 - 16) \times x + 72x^{2} + 4\\&= 72x^{2} - 34x + 4
\end{align*}
\item
\begin{align*}
B &= (- 7x - 7)(- 8x - 7)\\&= - 7x \times - 8x - 7x \times - 7 - 7 \times - 8x - 7 \times - 7\\&= - 7 \times - 8 \times x^{1 + 1} - 7 \times - 7 \times x - 7 \times - 8 \times x + 49\\&= 49x + 56x + 56x^{2} + 49\\&= (49 + 56) \times x + 56x^{2} + 49\\&= 56x^{2} + 105x + 49
\end{align*}
\item
\begin{align*}
C &= (2x - 8)^{2}\\&= (2x - 8)(2x - 8)\\&= 2x \times 2x + 2x \times - 8 - 8 \times 2x - 8 \times - 8\\&= 2 \times 2 \times x^{1 + 1} - 8 \times 2 \times x - 8 \times 2 \times x + 64\\&= - 16x - 16x + 4x^{2} + 64\\&= (- 16 - 16) \times x + 4x^{2} + 64\\&= 4x^{2} - 32x + 64
\end{align*}
\item
\begin{align*}
D &= - 6 + x(5x - 3)\\&= - 6 + x \times 5x + x \times - 3\\&= 5x^{2} - 3x - 6
\end{align*}
\item
\begin{align*}
E &= 8x^{2} + x(- 4x - 7)\\&= 8x^{2} + x \times - 4x + x \times - 7\\&= 8x^{2} - 4x^{2} - 7x\\&= 8x^{2} - 4x^{2} - 7x\\&= (8 - 4) \times x^{2} - 7x\\&= 4x^{2} - 7x
\end{align*}
\item
\begin{align*}
F &= 5(x - 4)(x + 2)\\&= (5x + 5 \times - 4)(x + 2)\\&= (5x - 20)(x + 2)\\&= 5x \times x + 5x \times 2 - 20x - 20 \times 2\\&= 2 \times 5 \times x - 40 + 5x^{2} - 20x\\&= 10x - 40 + 5x^{2} - 20x\\&= 5x^{2} + 10x - 20x - 40\\&= 5x^{2} + (10 - 20) \times x - 40\\&= 5x^{2} - 10x - 40
\end{align*}
\end{enumerate}
\end{solution}
\begin{exercise}[subtitle={Étude de fonctions}]
Soit $f(x) = - 9x^{2} + 90x - 216$ une fonction définie sur $\R$.
\begin{enumerate}
\item Calculer les valeurs suivantes
\[
f(1) \qquad f(-2)
\]
\item Dériver la fonction $f$
\item Étudier le signe de $f'$ puis en déduire les variations de $f$.
\item Est-ce que $f$ admet un maximum? un minimum? Calculer sa valeur.
\end{enumerate}
\end{exercise}
\begin{solution}
\begin{enumerate}
\item On remplace $x$ par les valeurs demandées
\[
f(1) = - 9 \times 1^{2} + 90 \times 1 - 216=- 9 \times 1 + 90 - 216=- 9 - 126=- 135
\]
\[
f(-1) = - 9 \times - 1^{2} + 90 \times - 1 - 216=- 9 \times 1 - 90 - 216=- 9 - 306=- 315
\]
\item Pas de solutions automatiques.
\item Pas de solutions automatiques.
\end{enumerate}
\end{solution}
%\printsolutionstype{exercise}
\end{document}
%%% Local Variables:
%%% mode: latex
%%% TeX-master: "master"
%%% End:

View File

@ -0,0 +1,141 @@
\documentclass[a5paper,10pt]{article}
\usepackage{myXsim}
\usepackage{tasks}
% Title Page
\title{DM1 \hfill ONAL Yakub}
\tribe{TST}
\date{Toussain 2020}
\begin{document}
\maketitle
\begin{exercise}[subtitle={Fractions}]
Faire les calculs avec les fraction suivants
\begin{multicols}{3}
\begin{enumerate}
\item $A = \dfrac{9}{6} - \dfrac{9}{6}$
\item $B = \dfrac{3}{4} - \dfrac{1}{32}$
\item $C = \dfrac{- 3}{7} + \dfrac{- 10}{6}$
\item $D = \dfrac{- 7}{2} - 1$
\item $E = \dfrac{5}{10} \times \dfrac{- 7}{9}$
\item $F = \dfrac{10}{7} \times 9$
\end{enumerate}
\end{multicols}
\end{exercise}
\begin{solution}
\begin{enumerate}
\item
\[
\dfrac{9}{6} - \dfrac{9}{6}=\dfrac{9}{6} - \dfrac{9}{6}=\dfrac{9 - 9}{6}=\dfrac{9 - 9}{6}=\dfrac{0}{6}
\]
\item
\[
\dfrac{3}{4} - \dfrac{1}{32}=\dfrac{3}{4} - \dfrac{1}{32}=\dfrac{3 \times 8}{4 \times 8} - \dfrac{1}{32}=\dfrac{24}{32} - \dfrac{1}{32}=\dfrac{24 - 1}{32}=\dfrac{24 - 1}{32}=\dfrac{23}{32}
\]
\item
\[
\dfrac{- 3}{7} + \dfrac{- 10}{6}=\dfrac{- 3 \times 6}{7 \times 6} + \dfrac{- 10 \times 7}{6 \times 7}=\dfrac{- 18}{42} + \dfrac{- 70}{42}=\dfrac{- 18 - 70}{42}=\dfrac{- 88}{42}
\]
\item
\[
\dfrac{- 7}{2} - 1=\dfrac{- 7}{2} + \dfrac{- 1}{1}=\dfrac{- 7}{2} + \dfrac{- 1 \times 2}{1 \times 2}=\dfrac{- 7}{2} + \dfrac{- 2}{2}=\dfrac{- 7 - 2}{2}=\dfrac{- 9}{2}
\]
\item
\[
\dfrac{5}{10} \times \dfrac{- 7}{9}=\dfrac{5 \times - 7}{10 \times 9}=\dfrac{- 35}{90}
\]
\item
\[
\dfrac{10}{7} \times 9=\dfrac{10 \times 9}{7}=\dfrac{90}{7}
\]
\end{enumerate}
\end{solution}
\begin{exercise}[subtitle={Développer réduire}]
Développer puis réduire les expressions suivantes
\begin{multicols}{2}
\begin{enumerate}
\item $A = (1x + 7)(3x + 7)$
\item $B = (- 7x - 3)(- 9x - 3)$
\item $C = (- 2x + 9)^{2}$
\item $D = - 3 + x(6x + 1)$
\item $E = - 7x^{2} + x(- 10x - 10)$
\item $F = - 7(x - 6)(x - 6)$
\end{enumerate}
\end{multicols}
\end{exercise}
\begin{solution}
\begin{enumerate}
\item
\begin{align*}
A &= (1x + 7)(3x + 7)\\&= x \times 3x + x \times 7 + 7 \times 3x + 7 \times 7\\&= 7 \times 3 \times x + 49 + 3x^{2} + 7x\\&= 21x + 49 + 3x^{2} + 7x\\&= 3x^{2} + 21x + 7x + 49\\&= 3x^{2} + (21 + 7) \times x + 49\\&= 3x^{2} + 28x + 49
\end{align*}
\item
\begin{align*}
B &= (- 7x - 3)(- 9x - 3)\\&= - 7x \times - 9x - 7x \times - 3 - 3 \times - 9x - 3 \times - 3\\&= - 7 \times - 9 \times x^{1 + 1} - 3 \times - 7 \times x - 3 \times - 9 \times x + 9\\&= 21x + 27x + 63x^{2} + 9\\&= (21 + 27) \times x + 63x^{2} + 9\\&= 63x^{2} + 48x + 9
\end{align*}
\item
\begin{align*}
C &= (- 2x + 9)^{2}\\&= (- 2x + 9)(- 2x + 9)\\&= - 2x \times - 2x - 2x \times 9 + 9 \times - 2x + 9 \times 9\\&= - 2 \times - 2 \times x^{1 + 1} + 9 \times - 2 \times x + 9 \times - 2 \times x + 81\\&= - 18x - 18x + 4x^{2} + 81\\&= (- 18 - 18) \times x + 4x^{2} + 81\\&= 4x^{2} - 36x + 81
\end{align*}
\item
\begin{align*}
D &= - 3 + x(6x + 1)\\&= - 3 + x \times 6x + x \times 1\\&= 6x^{2} + x - 3
\end{align*}
\item
\begin{align*}
E &= - 7x^{2} + x(- 10x - 10)\\&= - 7x^{2} + x \times - 10x + x \times - 10\\&= - 7x^{2} - 10x^{2} - 10x\\&= - 7x^{2} - 10x^{2} - 10x\\&= (- 7 - 10) \times x^{2} - 10x\\&= - 17x^{2} - 10x
\end{align*}
\item
\begin{align*}
F &= - 7(x - 6)(x - 6)\\&= (- 7x - 7 \times - 6)(x - 6)\\&= (- 7x + 42)(x - 6)\\&= - 7x \times x - 7x \times - 6 + 42x + 42 \times - 6\\&= - 6 \times - 7 \times x - 252 - 7x^{2} + 42x\\&= 42x - 252 - 7x^{2} + 42x\\&= - 7x^{2} + 42x + 42x - 252\\&= - 7x^{2} + (42 + 42) \times x - 252\\&= - 7x^{2} + 84x - 252
\end{align*}
\end{enumerate}
\end{solution}
\begin{exercise}[subtitle={Étude de fonctions}]
Soit $f(x) = - x^{2} + 4x + 60$ une fonction définie sur $\R$.
\begin{enumerate}
\item Calculer les valeurs suivantes
\[
f(1) \qquad f(-2)
\]
\item Dériver la fonction $f$
\item Étudier le signe de $f'$ puis en déduire les variations de $f$.
\item Est-ce que $f$ admet un maximum? un minimum? Calculer sa valeur.
\end{enumerate}
\end{exercise}
\begin{solution}
\begin{enumerate}
\item On remplace $x$ par les valeurs demandées
\[
f(1) = - 1 \times 1^{2} + 4 \times 1 + 60=- 1 \times 1 + 4 + 60=- 1 + 64=63
\]
\[
f(-1) = - 1 \times - 1^{2} + 4 \times - 1 + 60=- 1 \times 1 - 4 + 60=- 1 + 56=55
\]
\item Pas de solutions automatiques.
\item Pas de solutions automatiques.
\end{enumerate}
\end{solution}
%\printsolutionstype{exercise}
\end{document}
%%% Local Variables:
%%% mode: latex
%%% TeX-master: "master"
%%% End:

View File

@ -0,0 +1,141 @@
\documentclass[a5paper,10pt]{article}
\usepackage{myXsim}
\usepackage{tasks}
% Title Page
\title{DM1 \hfill SORIANO Laura}
\tribe{TST}
\date{Toussain 2020}
\begin{document}
\maketitle
\begin{exercise}[subtitle={Fractions}]
Faire les calculs avec les fraction suivants
\begin{multicols}{3}
\begin{enumerate}
\item $A = \dfrac{- 7}{6} - \dfrac{9}{6}$
\item $B = \dfrac{2}{4} - \dfrac{4}{20}$
\item $C = \dfrac{7}{10} + \dfrac{- 7}{9}$
\item $D = \dfrac{6}{4} - 3$
\item $E = \dfrac{- 6}{8} \times \dfrac{7}{7}$
\item $F = \dfrac{- 5}{7} \times - 2$
\end{enumerate}
\end{multicols}
\end{exercise}
\begin{solution}
\begin{enumerate}
\item
\[
\dfrac{- 7}{6} - \dfrac{9}{6}=\dfrac{- 7}{6} - \dfrac{9}{6}=\dfrac{- 7 - 9}{6}=\dfrac{- 7 - 9}{6}=\dfrac{- 16}{6}
\]
\item
\[
\dfrac{2}{4} - \dfrac{4}{20}=\dfrac{2}{4} - \dfrac{4}{20}=\dfrac{2 \times 5}{4 \times 5} - \dfrac{4}{20}=\dfrac{10}{20} - \dfrac{4}{20}=\dfrac{10 - 4}{20}=\dfrac{10 - 4}{20}=\dfrac{6}{20}
\]
\item
\[
\dfrac{7}{10} + \dfrac{- 7}{9}=\dfrac{7 \times 9}{10 \times 9} + \dfrac{- 7 \times 10}{9 \times 10}=\dfrac{63}{90} + \dfrac{- 70}{90}=\dfrac{63 - 70}{90}=\dfrac{- 7}{90}
\]
\item
\[
\dfrac{6}{4} - 3=\dfrac{6}{4} + \dfrac{- 3}{1}=\dfrac{6}{4} + \dfrac{- 3 \times 4}{1 \times 4}=\dfrac{6}{4} + \dfrac{- 12}{4}=\dfrac{6 - 12}{4}=\dfrac{- 6}{4}
\]
\item
\[
\dfrac{- 6}{8} \times \dfrac{7}{7}=\dfrac{- 6 \times 7}{8 \times 7}=\dfrac{- 42}{56}
\]
\item
\[
\dfrac{- 5}{7} \times - 2=\dfrac{- 5 \times - 2}{7}=\dfrac{10}{7}
\]
\end{enumerate}
\end{solution}
\begin{exercise}[subtitle={Développer réduire}]
Développer puis réduire les expressions suivantes
\begin{multicols}{2}
\begin{enumerate}
\item $A = (2x + 4)(- 2x + 4)$
\item $B = (- 1x + 6)(- 7x + 6)$
\item $C = (7x - 7)^{2}$
\item $D = - 5 + x(- 9x - 9)$
\item $E = 7x^{2} + x(- 5x - 4)$
\item $F = - 10(x - 8)(x + 9)$
\end{enumerate}
\end{multicols}
\end{exercise}
\begin{solution}
\begin{enumerate}
\item
\begin{align*}
A &= (2x + 4)(- 2x + 4)\\&= 2x \times - 2x + 2x \times 4 + 4 \times - 2x + 4 \times 4\\&= 2 \times - 2 \times x^{1 + 1} + 4 \times 2 \times x + 4 \times - 2 \times x + 16\\&= 8x - 8x - 4x^{2} + 16\\&= (8 - 8) \times x - 4x^{2} + 16\\&= 0x - 4x^{2} + 16\\&= - 4x^{2} + 16
\end{align*}
\item
\begin{align*}
B &= (- 1x + 6)(- 7x + 6)\\&= - x \times - 7x - x \times 6 + 6 \times - 7x + 6 \times 6\\&= - 1 \times - 7 \times x^{1 + 1} + 6 \times - 1 \times x + 6 \times - 7 \times x + 36\\&= - 6x - 42x + 7x^{2} + 36\\&= (- 6 - 42) \times x + 7x^{2} + 36\\&= 7x^{2} - 48x + 36
\end{align*}
\item
\begin{align*}
C &= (7x - 7)^{2}\\&= (7x - 7)(7x - 7)\\&= 7x \times 7x + 7x \times - 7 - 7 \times 7x - 7 \times - 7\\&= 7 \times 7 \times x^{1 + 1} - 7 \times 7 \times x - 7 \times 7 \times x + 49\\&= - 49x - 49x + 49x^{2} + 49\\&= (- 49 - 49) \times x + 49x^{2} + 49\\&= 49x^{2} - 98x + 49
\end{align*}
\item
\begin{align*}
D &= - 5 + x(- 9x - 9)\\&= - 5 + x \times - 9x + x \times - 9\\&= - 9x^{2} - 9x - 5
\end{align*}
\item
\begin{align*}
E &= 7x^{2} + x(- 5x - 4)\\&= 7x^{2} + x \times - 5x + x \times - 4\\&= 7x^{2} - 5x^{2} - 4x\\&= 7x^{2} - 5x^{2} - 4x\\&= (7 - 5) \times x^{2} - 4x\\&= 2x^{2} - 4x
\end{align*}
\item
\begin{align*}
F &= - 10(x - 8)(x + 9)\\&= (- 10x - 10 \times - 8)(x + 9)\\&= (- 10x + 80)(x + 9)\\&= - 10x \times x - 10x \times 9 + 80x + 80 \times 9\\&= 9 \times - 10 \times x + 720 - 10x^{2} + 80x\\&= - 90x + 720 - 10x^{2} + 80x\\&= - 10x^{2} - 90x + 80x + 720\\&= - 10x^{2} + (- 90 + 80) \times x + 720\\&= - 10x^{2} - 10x + 720
\end{align*}
\end{enumerate}
\end{solution}
\begin{exercise}[subtitle={Étude de fonctions}]
Soit $f(x) = 6x^{2} - 48x - 54$ une fonction définie sur $\R$.
\begin{enumerate}
\item Calculer les valeurs suivantes
\[
f(1) \qquad f(-2)
\]
\item Dériver la fonction $f$
\item Étudier le signe de $f'$ puis en déduire les variations de $f$.
\item Est-ce que $f$ admet un maximum? un minimum? Calculer sa valeur.
\end{enumerate}
\end{exercise}
\begin{solution}
\begin{enumerate}
\item On remplace $x$ par les valeurs demandées
\[
f(1) = 6 \times 1^{2} - 48 \times 1 - 54=6 \times 1 - 48 - 54=6 - 102=- 96
\]
\[
f(-1) = 6 \times - 1^{2} - 48 \times - 1 - 54=6 \times 1 + 48 - 54=6 - 6=0
\]
\item Pas de solutions automatiques.
\item Pas de solutions automatiques.
\end{enumerate}
\end{solution}
%\printsolutionstype{exercise}
\end{document}
%%% Local Variables:
%%% mode: latex
%%% TeX-master: "master"
%%% End:

View File

@ -0,0 +1,141 @@
\documentclass[a5paper,10pt]{article}
\usepackage{myXsim}
\usepackage{tasks}
% Title Page
\title{DM1 \hfill VECCHIO Léa}
\tribe{TST}
\date{Toussain 2020}
\begin{document}
\maketitle
\begin{exercise}[subtitle={Fractions}]
Faire les calculs avec les fraction suivants
\begin{multicols}{3}
\begin{enumerate}
\item $A = \dfrac{5}{9} - \dfrac{9}{9}$
\item $B = \dfrac{7}{4} - \dfrac{8}{36}$
\item $C = \dfrac{- 10}{10} + \dfrac{2}{9}$
\item $D = \dfrac{5}{4} + 6$
\item $E = \dfrac{- 7}{7} \times \dfrac{1}{6}$
\item $F = \dfrac{7}{10} \times - 8$
\end{enumerate}
\end{multicols}
\end{exercise}
\begin{solution}
\begin{enumerate}
\item
\[
\dfrac{5}{9} - \dfrac{9}{9}=\dfrac{5}{9} - \dfrac{9}{9}=\dfrac{5 - 9}{9}=\dfrac{5 - 9}{9}=\dfrac{- 4}{9}
\]
\item
\[
\dfrac{7}{4} - \dfrac{8}{36}=\dfrac{7}{4} - \dfrac{8}{36}=\dfrac{7 \times 9}{4 \times 9} - \dfrac{8}{36}=\dfrac{63}{36} - \dfrac{8}{36}=\dfrac{63 - 8}{36}=\dfrac{63 - 8}{36}=\dfrac{55}{36}
\]
\item
\[
\dfrac{- 10}{10} + \dfrac{2}{9}=\dfrac{- 10 \times 9}{10 \times 9} + \dfrac{2 \times 10}{9 \times 10}=\dfrac{- 90}{90} + \dfrac{20}{90}=\dfrac{- 90 + 20}{90}=\dfrac{- 70}{90}
\]
\item
\[
\dfrac{5}{4} + 6=\dfrac{5}{4} + \dfrac{6}{1}=\dfrac{5}{4} + \dfrac{6 \times 4}{1 \times 4}=\dfrac{5}{4} + \dfrac{24}{4}=\dfrac{5 + 24}{4}=\dfrac{29}{4}
\]
\item
\[
\dfrac{- 7}{7} \times \dfrac{1}{6}=\dfrac{- 7 \times 1}{7 \times 6}=\dfrac{- 7}{42}
\]
\item
\[
\dfrac{7}{10} \times - 8=\dfrac{7 \times - 8}{10}=\dfrac{- 56}{10}
\]
\end{enumerate}
\end{solution}
\begin{exercise}[subtitle={Développer réduire}]
Développer puis réduire les expressions suivantes
\begin{multicols}{2}
\begin{enumerate}
\item $A = (- 5x + 5)(- 8x + 5)$
\item $B = (1x + 7)(- 10x + 7)$
\item $C = (2x + 4)^{2}$
\item $D = 1 + x(- 7x + 1)$
\item $E = - 3x^{2} + x(- 2x + 6)$
\item $F = 5(x - 6)(x + 8)$
\end{enumerate}
\end{multicols}
\end{exercise}
\begin{solution}
\begin{enumerate}
\item
\begin{align*}
A &= (- 5x + 5)(- 8x + 5)\\&= - 5x \times - 8x - 5x \times 5 + 5 \times - 8x + 5 \times 5\\&= - 5 \times - 8 \times x^{1 + 1} + 5 \times - 5 \times x + 5 \times - 8 \times x + 25\\&= - 25x - 40x + 40x^{2} + 25\\&= (- 25 - 40) \times x + 40x^{2} + 25\\&= 40x^{2} - 65x + 25
\end{align*}
\item
\begin{align*}
B &= (1x + 7)(- 10x + 7)\\&= x \times - 10x + x \times 7 + 7 \times - 10x + 7 \times 7\\&= 7 \times - 10 \times x + 49 - 10x^{2} + 7x\\&= - 70x + 49 - 10x^{2} + 7x\\&= - 10x^{2} - 70x + 7x + 49\\&= - 10x^{2} + (- 70 + 7) \times x + 49\\&= - 10x^{2} - 63x + 49
\end{align*}
\item
\begin{align*}
C &= (2x + 4)^{2}\\&= (2x + 4)(2x + 4)\\&= 2x \times 2x + 2x \times 4 + 4 \times 2x + 4 \times 4\\&= 2 \times 2 \times x^{1 + 1} + 4 \times 2 \times x + 4 \times 2 \times x + 16\\&= 8x + 8x + 4x^{2} + 16\\&= (8 + 8) \times x + 4x^{2} + 16\\&= 4x^{2} + 16x + 16
\end{align*}
\item
\begin{align*}
D &= 1 + x(- 7x + 1)\\&= 1 + x \times - 7x + x \times 1\\&= - 7x^{2} + x + 1
\end{align*}
\item
\begin{align*}
E &= - 3x^{2} + x(- 2x + 6)\\&= - 3x^{2} + x \times - 2x + x \times 6\\&= - 3x^{2} - 2x^{2} + 6x\\&= - 3x^{2} - 2x^{2} + 6x\\&= (- 3 - 2) \times x^{2} + 6x\\&= - 5x^{2} + 6x
\end{align*}
\item
\begin{align*}
F &= 5(x - 6)(x + 8)\\&= (5x + 5 \times - 6)(x + 8)\\&= (5x - 30)(x + 8)\\&= 5x \times x + 5x \times 8 - 30x - 30 \times 8\\&= 8 \times 5 \times x - 240 + 5x^{2} - 30x\\&= 40x - 240 + 5x^{2} - 30x\\&= 5x^{2} + 40x - 30x - 240\\&= 5x^{2} + (40 - 30) \times x - 240\\&= 5x^{2} + 10x - 240
\end{align*}
\end{enumerate}
\end{solution}
\begin{exercise}[subtitle={Étude de fonctions}]
Soit $f(x) = - 6x^{2} - 108x - 480$ une fonction définie sur $\R$.
\begin{enumerate}
\item Calculer les valeurs suivantes
\[
f(1) \qquad f(-2)
\]
\item Dériver la fonction $f$
\item Étudier le signe de $f'$ puis en déduire les variations de $f$.
\item Est-ce que $f$ admet un maximum? un minimum? Calculer sa valeur.
\end{enumerate}
\end{exercise}
\begin{solution}
\begin{enumerate}
\item On remplace $x$ par les valeurs demandées
\[
f(1) = - 6 \times 1^{2} - 108 \times 1 - 480=- 6 \times 1 - 108 - 480=- 6 - 588=- 594
\]
\[
f(-1) = - 6 \times - 1^{2} - 108 \times - 1 - 480=- 6 \times 1 + 108 - 480=- 6 - 372=- 378
\]
\item Pas de solutions automatiques.
\item Pas de solutions automatiques.
\end{enumerate}
\end{solution}
%\printsolutionstype{exercise}
\end{document}
%%% Local Variables:
%%% mode: latex
%%% TeX-master: "master"
%%% End:

Binary file not shown.

View File

@ -0,0 +1,155 @@
\documentclass[a5paper,10pt]{article}
\usepackage{myXsim}
\usepackage{tasks}
% Title Page
\title{DM1 \hfill \Var{Nom}}
\tribe{TST}
\date{Toussain 2020}
\begin{document}
\maketitle
\begin{exercise}[subtitle={Fractions}]
Faire les calculs avec les fraction suivants
\begin{multicols}{3}
\begin{enumerate}
%- set a = Expression.random("{a} / {b} - {c} / {b}", ["b > 1"])
\item $A = \Var{a}$
%- set b = Expression.random("{a} / {b} - {c} / {k*b}", ["b > 1", "k>1"])
\item $B = \Var{b}$
%- set c = Expression.random("{a} / {b} + {c} / {b-1}", ["b > 1"])
\item $C = \Var{c}$
%- set d = Expression.random("{a} / {b} + {c}", ["b > 1"])
\item $D = \Var{d}$
%- set e = Expression.random("{a} / {b} * {c} / {b-1}", ["b > 1"])
\item $E = \Var{e}$
%- set f = Expression.random("{a} / {b} * {c}", ["b > 1"])
\item $F = \Var{f}$
\end{enumerate}
\end{multicols}
\end{exercise}
\begin{solution}
\begin{enumerate}
\item
\[
\Var{a.simplify().explain() | join('=')}
\]
\item
\[
\Var{b.simplify().explain() | join('=')}
\]
\item
\[
\Var{c.simplify().explain() | join('=')}
\]
\item
\[
\Var{d.simplify().explain() | join('=')}
\]
\item
\[
\Var{e.simplify().explain() | join('=')}
\]
\item
\[
\Var{f.simplify().explain() | join('=')}
\]
\end{enumerate}
\end{solution}
\begin{exercise}[subtitle={Développer réduire}]
Développer puis réduire les expressions suivantes
\begin{multicols}{2}
\begin{enumerate}
%- set a = Expression.random("({a}x + {b})*({c}x + {b})")
\item $A = \Var{a}$
%- set b = Expression.random("({a}x + {b})*({c}x + {b})")
\item $B = \Var{b}$
%- set c = Expression.random("({a}x + {b})^2")
\item $C = \Var{c}$
%- set d = Expression.random("{c} + x*({a}x + {b})")
\item $D = \Var{d}$
%- set e = Expression.random("{c}*x^2 + x*({a}x + {b})")
\item $E = \Var{e}$
%- set f = Expression.random("{a}(x+{b})(x+{c})")
\item $F = \Var{f}$
\end{enumerate}
\end{multicols}
\end{exercise}
\begin{solution}
\begin{enumerate}
\item
\begin{align*}
A &= \Var{a.simplify().explain() | join('\\\\&= ')}
\end{align*}
\item
\begin{align*}
B &= \Var{b.simplify().explain() | join('\\\\&= ')}
\end{align*}
\item
\begin{align*}
C &= \Var{c.simplify().explain() | join('\\\\&= ')}
\end{align*}
\item
\begin{align*}
D &= \Var{d.simplify().explain() | join('\\\\&= ')}
\end{align*}
\item
\begin{align*}
E &= \Var{e.simplify().explain() | join('\\\\&= ')}
\end{align*}
\item
\begin{align*}
F &= \Var{f.simplify().explain() | join('\\\\&= ')}
\end{align*}
\end{enumerate}
\end{solution}
\begin{exercise}[subtitle={Étude de fonctions}]
%- set f = Expression.random("{a}(x-{b})(x-{c})")
Soit $f(x) = \Var{f.simplify()}$ une fonction définie sur $\R$.
\begin{enumerate}
\item Calculer les valeurs suivantes
\[
f(1) \qquad f(-2)
\]
\item Dériver la fonction $f$
\item Étudier le signe de $f'$ puis en déduire les variations de $f$.
\item Est-ce que $f$ admet un maximum? un minimum? Calculer sa valeur.
\end{enumerate}
\end{exercise}
\begin{solution}
\begin{enumerate}
\item On remplace $x$ par les valeurs demandées
\[
f(1) = \Var{f.simplify()(1).explain() | join('=')}
\]
\[
f(-1) = \Var{f.simplify()(-1).explain() | join('=')}
\]
\item Pas de solutions automatiques.
\item Pas de solutions automatiques.
\end{enumerate}
\end{solution}
%\printsolutionstype{exercise}
\end{document}
%%% Local Variables:
%%% mode: latex
%%% TeX-master: "master"
%%% End:

View File

@ -0,0 +1,141 @@
\documentclass[a5paper,10pt]{article}
\usepackage{myXsim}
\usepackage{tasks}
% Title Page
\title{DM1 \hfill ASAIDI Sophian}
\tribe{TST}
\date{Toussain 2020}
\begin{document}
\maketitle
\begin{exercise}[subtitle={Fractions}]
Faire les calculs avec les fraction suivants
\begin{multicols}{3}
\begin{enumerate}
\item $A = \dfrac{- 4}{10} - \dfrac{- 2}{10}$
\item $B = \dfrac{- 10}{8} - \dfrac{6}{40}$
\item $C = \dfrac{3}{8} + \dfrac{- 5}{7}$
\item $D = \dfrac{- 6}{9} - 1$
\item $E = \dfrac{9}{10} \times \dfrac{10}{9}$
\item $F = \dfrac{7}{2} \times - 10$
\end{enumerate}
\end{multicols}
\end{exercise}
\begin{solution}
\begin{enumerate}
\item
\[
\dfrac{- 4}{10} - \dfrac{- 2}{10}=\dfrac{- 4}{10} + \dfrac{2}{10}=\dfrac{- 4 + 2}{10}=\dfrac{- 2}{10}
\]
\item
\[
\dfrac{- 10}{8} - \dfrac{6}{40}=\dfrac{- 10}{8} - \dfrac{6}{40}=\dfrac{- 10 \times 5}{8 \times 5} - \dfrac{6}{40}=\dfrac{- 50}{40} - \dfrac{6}{40}=\dfrac{- 50 - 6}{40}=\dfrac{- 50 - 6}{40}=\dfrac{- 56}{40}
\]
\item
\[
\dfrac{3}{8} + \dfrac{- 5}{7}=\dfrac{3 \times 7}{8 \times 7} + \dfrac{- 5 \times 8}{7 \times 8}=\dfrac{21}{56} + \dfrac{- 40}{56}=\dfrac{21 - 40}{56}=\dfrac{- 19}{56}
\]
\item
\[
\dfrac{- 6}{9} - 1=\dfrac{- 6}{9} + \dfrac{- 1}{1}=\dfrac{- 6}{9} + \dfrac{- 1 \times 9}{1 \times 9}=\dfrac{- 6}{9} + \dfrac{- 9}{9}=\dfrac{- 6 - 9}{9}=\dfrac{- 15}{9}
\]
\item
\[
\dfrac{9}{10} \times \dfrac{10}{9}=\dfrac{9 \times 10}{10 \times 9}=\dfrac{90}{90}
\]
\item
\[
\dfrac{7}{2} \times - 10=\dfrac{7 \times - 10}{2}=\dfrac{- 70}{2}
\]
\end{enumerate}
\end{solution}
\begin{exercise}[subtitle={Développer réduire}]
Développer puis réduire les expressions suivantes
\begin{multicols}{2}
\begin{enumerate}
\item $A = (4x + 9)(3x + 9)$
\item $B = (- 6x + 2)(10x + 2)$
\item $C = (2x + 1)^{2}$
\item $D = - 3 + x(9x - 5)$
\item $E = 8x^{2} + x(5x + 6)$
\item $F = - 6(x - 10)(x + 4)$
\end{enumerate}
\end{multicols}
\end{exercise}
\begin{solution}
\begin{enumerate}
\item
\begin{align*}
A &= (4x + 9)(3x + 9)\\&= 4x \times 3x + 4x \times 9 + 9 \times 3x + 9 \times 9\\&= 4 \times 3 \times x^{1 + 1} + 9 \times 4 \times x + 9 \times 3 \times x + 81\\&= 36x + 27x + 12x^{2} + 81\\&= (36 + 27) \times x + 12x^{2} + 81\\&= 12x^{2} + 63x + 81
\end{align*}
\item
\begin{align*}
B &= (- 6x + 2)(10x + 2)\\&= - 6x \times 10x - 6x \times 2 + 2 \times 10x + 2 \times 2\\&= - 6 \times 10 \times x^{1 + 1} + 2 \times - 6 \times x + 2 \times 10 \times x + 4\\&= - 12x + 20x - 60x^{2} + 4\\&= (- 12 + 20) \times x - 60x^{2} + 4\\&= - 60x^{2} + 8x + 4
\end{align*}
\item
\begin{align*}
C &= (2x + 1)^{2}\\&= (2x + 1)(2x + 1)\\&= 2x \times 2x + 2x \times 1 + 1 \times 2x + 1 \times 1\\&= 2 \times 2 \times x^{1 + 1} + 2x + 2x + 1\\&= 4x^{2} + 2x + 2x + 1\\&= 4x^{2} + (2 + 2) \times x + 1\\&= 4x^{2} + 4x + 1
\end{align*}
\item
\begin{align*}
D &= - 3 + x(9x - 5)\\&= - 3 + x \times 9x + x \times - 5\\&= 9x^{2} - 5x - 3
\end{align*}
\item
\begin{align*}
E &= 8x^{2} + x(5x + 6)\\&= 8x^{2} + x \times 5x + x \times 6\\&= 8x^{2} + 5x^{2} + 6x\\&= 8x^{2} + 5x^{2} + 6x\\&= (8 + 5) \times x^{2} + 6x\\&= 13x^{2} + 6x
\end{align*}
\item
\begin{align*}
F &= - 6(x - 10)(x + 4)\\&= (- 6x - 6 \times - 10)(x + 4)\\&= (- 6x + 60)(x + 4)\\&= - 6x \times x - 6x \times 4 + 60x + 60 \times 4\\&= 4 \times - 6 \times x + 240 - 6x^{2} + 60x\\&= - 24x + 240 - 6x^{2} + 60x\\&= - 6x^{2} - 24x + 60x + 240\\&= - 6x^{2} + (- 24 + 60) \times x + 240\\&= - 6x^{2} + 36x + 240
\end{align*}
\end{enumerate}
\end{solution}
\begin{exercise}[subtitle={Étude de fonctions}]
Soit $f(x) = - 9x^{2} + 27x + 162$ une fonction définie sur $\R$.
\begin{enumerate}
\item Calculer les valeurs suivantes
\[
f(1) \qquad f(-2)
\]
\item Dériver la fonction $f$
\item Étudier le signe de $f'$ puis en déduire les variations de $f$.
\item Est-ce que $f$ admet un maximum? un minimum? Calculer sa valeur.
\end{enumerate}
\end{exercise}
\begin{solution}
\begin{enumerate}
\item On remplace $x$ par les valeurs demandées
\[
f(1) = - 9 \times 1^{2} + 27 \times 1 + 162=- 9 \times 1 + 27 + 162=- 9 + 189=180
\]
\[
f(-1) = - 9 \times - 1^{2} + 27 \times - 1 + 162=- 9 \times 1 - 27 + 162=- 9 + 135=126
\]
\item Pas de solutions automatiques.
\item Pas de solutions automatiques.
\end{enumerate}
\end{solution}
%\printsolutionstype{exercise}
\end{document}
%%% Local Variables:
%%% mode: latex
%%% TeX-master: "master"
%%% End:

View File

@ -0,0 +1,141 @@
\documentclass[a5paper,10pt]{article}
\usepackage{myXsim}
\usepackage{tasks}
% Title Page
\title{DM1 \hfill BELARBI Samira}
\tribe{TST}
\date{Toussain 2020}
\begin{document}
\maketitle
\begin{exercise}[subtitle={Fractions}]
Faire les calculs avec les fraction suivants
\begin{multicols}{3}
\begin{enumerate}
\item $A = \dfrac{- 8}{9} - \dfrac{- 3}{9}$
\item $B = \dfrac{3}{6} - \dfrac{5}{54}$
\item $C = \dfrac{- 6}{10} + \dfrac{- 4}{9}$
\item $D = \dfrac{9}{4} + 1$
\item $E = \dfrac{5}{2} \times \dfrac{5}{1}$
\item $F = \dfrac{- 8}{5} \times 9$
\end{enumerate}
\end{multicols}
\end{exercise}
\begin{solution}
\begin{enumerate}
\item
\[
\dfrac{- 8}{9} - \dfrac{- 3}{9}=\dfrac{- 8}{9} + \dfrac{3}{9}=\dfrac{- 8 + 3}{9}=\dfrac{- 5}{9}
\]
\item
\[
\dfrac{3}{6} - \dfrac{5}{54}=\dfrac{3}{6} - \dfrac{5}{54}=\dfrac{3 \times 9}{6 \times 9} - \dfrac{5}{54}=\dfrac{27}{54} - \dfrac{5}{54}=\dfrac{27 - 5}{54}=\dfrac{27 - 5}{54}=\dfrac{22}{54}
\]
\item
\[
\dfrac{- 6}{10} + \dfrac{- 4}{9}=\dfrac{- 6 \times 9}{10 \times 9} + \dfrac{- 4 \times 10}{9 \times 10}=\dfrac{- 54}{90} + \dfrac{- 40}{90}=\dfrac{- 54 - 40}{90}=\dfrac{- 94}{90}
\]
\item
\[
\dfrac{9}{4} + 1=\dfrac{9}{4} + \dfrac{1}{1}=\dfrac{9}{4} + \dfrac{1 \times 4}{1 \times 4}=\dfrac{9}{4} + \dfrac{4}{4}=\dfrac{9 + 4}{4}=\dfrac{13}{4}
\]
\item
\[
\dfrac{5}{2} \times \dfrac{5}{1}=\dfrac{5 \times 5}{2 \times 1}=\dfrac{25}{2}
\]
\item
\[
\dfrac{- 8}{5} \times 9=\dfrac{- 8 \times 9}{5}=\dfrac{- 72}{5}
\]
\end{enumerate}
\end{solution}
\begin{exercise}[subtitle={Développer réduire}]
Développer puis réduire les expressions suivantes
\begin{multicols}{2}
\begin{enumerate}
\item $A = (- 2x + 6)(- 6x + 6)$
\item $B = (- 8x + 4)(- 1x + 4)$
\item $C = (- 6x + 9)^{2}$
\item $D = 4 + x(7x - 4)$
\item $E = - 5x^{2} + x(- 5x - 7)$
\item $F = - 8(x - 1)(x + 5)$
\end{enumerate}
\end{multicols}
\end{exercise}
\begin{solution}
\begin{enumerate}
\item
\begin{align*}
A &= (- 2x + 6)(- 6x + 6)\\&= - 2x \times - 6x - 2x \times 6 + 6 \times - 6x + 6 \times 6\\&= - 2 \times - 6 \times x^{1 + 1} + 6 \times - 2 \times x + 6 \times - 6 \times x + 36\\&= - 12x - 36x + 12x^{2} + 36\\&= (- 12 - 36) \times x + 12x^{2} + 36\\&= 12x^{2} - 48x + 36
\end{align*}
\item
\begin{align*}
B &= (- 8x + 4)(- 1x + 4)\\&= - 8x \times - x - 8x \times 4 + 4 \times - x + 4 \times 4\\&= - 8 \times - 1 \times x^{1 + 1} + 4 \times - 8 \times x + 4 \times - 1 \times x + 16\\&= - 32x - 4x + 8x^{2} + 16\\&= (- 32 - 4) \times x + 8x^{2} + 16\\&= 8x^{2} - 36x + 16
\end{align*}
\item
\begin{align*}
C &= (- 6x + 9)^{2}\\&= (- 6x + 9)(- 6x + 9)\\&= - 6x \times - 6x - 6x \times 9 + 9 \times - 6x + 9 \times 9\\&= - 6 \times - 6 \times x^{1 + 1} + 9 \times - 6 \times x + 9 \times - 6 \times x + 81\\&= - 54x - 54x + 36x^{2} + 81\\&= (- 54 - 54) \times x + 36x^{2} + 81\\&= 36x^{2} - 108x + 81
\end{align*}
\item
\begin{align*}
D &= 4 + x(7x - 4)\\&= 4 + x \times 7x + x \times - 4\\&= 7x^{2} - 4x + 4
\end{align*}
\item
\begin{align*}
E &= - 5x^{2} + x(- 5x - 7)\\&= - 5x^{2} + x \times - 5x + x \times - 7\\&= - 5x^{2} - 5x^{2} - 7x\\&= - 5x^{2} - 5x^{2} - 7x\\&= (- 5 - 5) \times x^{2} - 7x\\&= - 10x^{2} - 7x
\end{align*}
\item
\begin{align*}
F &= - 8(x - 1)(x + 5)\\&= (- 8x - 8 \times - 1)(x + 5)\\&= (- 8x + 8)(x + 5)\\&= - 8x \times x - 8x \times 5 + 8x + 8 \times 5\\&= 5 \times - 8 \times x + 40 - 8x^{2} + 8x\\&= - 40x + 40 - 8x^{2} + 8x\\&= - 8x^{2} - 40x + 8x + 40\\&= - 8x^{2} + (- 40 + 8) \times x + 40\\&= - 8x^{2} - 32x + 40
\end{align*}
\end{enumerate}
\end{solution}
\begin{exercise}[subtitle={Étude de fonctions}]
Soit $f(x) = 5x^{2} - 35x + 60$ une fonction définie sur $\R$.
\begin{enumerate}
\item Calculer les valeurs suivantes
\[
f(1) \qquad f(-2)
\]
\item Dériver la fonction $f$
\item Étudier le signe de $f'$ puis en déduire les variations de $f$.
\item Est-ce que $f$ admet un maximum? un minimum? Calculer sa valeur.
\end{enumerate}
\end{exercise}
\begin{solution}
\begin{enumerate}
\item On remplace $x$ par les valeurs demandées
\[
f(1) = 5 \times 1^{2} - 35 \times 1 + 60=5 \times 1 - 35 + 60=5 + 25=30
\]
\[
f(-1) = 5 \times - 1^{2} - 35 \times - 1 + 60=5 \times 1 + 35 + 60=5 + 95=100
\]
\item Pas de solutions automatiques.
\item Pas de solutions automatiques.
\end{enumerate}
\end{solution}
%\printsolutionstype{exercise}
\end{document}
%%% Local Variables:
%%% mode: latex
%%% TeX-master: "master"
%%% End:

View File

@ -0,0 +1,141 @@
\documentclass[a5paper,10pt]{article}
\usepackage{myXsim}
\usepackage{tasks}
% Title Page
\title{DM1 \hfill BERTAN Ufuk}
\tribe{TST}
\date{Toussain 2020}
\begin{document}
\maketitle
\begin{exercise}[subtitle={Fractions}]
Faire les calculs avec les fraction suivants
\begin{multicols}{3}
\begin{enumerate}
\item $A = \dfrac{2}{3} - \dfrac{7}{3}$
\item $B = \dfrac{3}{5} - \dfrac{- 2}{40}$
\item $C = \dfrac{8}{4} + \dfrac{- 6}{3}$
\item $D = \dfrac{- 3}{6} + 9$
\item $E = \dfrac{1}{3} \times \dfrac{- 6}{2}$
\item $F = \dfrac{- 5}{8} \times 4$
\end{enumerate}
\end{multicols}
\end{exercise}
\begin{solution}
\begin{enumerate}
\item
\[
\dfrac{2}{3} - \dfrac{7}{3}=\dfrac{2}{3} - \dfrac{7}{3}=\dfrac{2 - 7}{3}=\dfrac{2 - 7}{3}=\dfrac{- 5}{3}
\]
\item
\[
\dfrac{3}{5} - \dfrac{- 2}{40}=\dfrac{3}{5} + \dfrac{2}{40}=\dfrac{3 \times 8}{5 \times 8} + \dfrac{2}{40}=\dfrac{24}{40} + \dfrac{2}{40}=\dfrac{24 + 2}{40}=\dfrac{26}{40}
\]
\item
\[
\dfrac{8}{4} + \dfrac{- 6}{3}=\dfrac{8 \times 3}{4 \times 3} + \dfrac{- 6 \times 4}{3 \times 4}=\dfrac{24}{12} + \dfrac{- 24}{12}=\dfrac{24 - 24}{12}=\dfrac{0}{12}
\]
\item
\[
\dfrac{- 3}{6} + 9=\dfrac{- 3}{6} + \dfrac{9}{1}=\dfrac{- 3}{6} + \dfrac{9 \times 6}{1 \times 6}=\dfrac{- 3}{6} + \dfrac{54}{6}=\dfrac{- 3 + 54}{6}=\dfrac{51}{6}
\]
\item
\[
\dfrac{1}{3} \times \dfrac{- 6}{2}=\dfrac{1 \times - 6}{3 \times 2}=\dfrac{- 6}{6}
\]
\item
\[
\dfrac{- 5}{8} \times 4=\dfrac{- 5 \times 4}{8}=\dfrac{- 20}{8}
\]
\end{enumerate}
\end{solution}
\begin{exercise}[subtitle={Développer réduire}]
Développer puis réduire les expressions suivantes
\begin{multicols}{2}
\begin{enumerate}
\item $A = (- 10x - 7)(- 8x - 7)$
\item $B = (5x - 3)(- 2x - 3)$
\item $C = (- 3x + 8)^{2}$
\item $D = - 3 + x(- 9x + 7)$
\item $E = 9x^{2} + x(- 5x - 3)$
\item $F = 3(x + 4)(x - 7)$
\end{enumerate}
\end{multicols}
\end{exercise}
\begin{solution}
\begin{enumerate}
\item
\begin{align*}
A &= (- 10x - 7)(- 8x - 7)\\&= - 10x \times - 8x - 10x \times - 7 - 7 \times - 8x - 7 \times - 7\\&= - 10 \times - 8 \times x^{1 + 1} - 7 \times - 10 \times x - 7 \times - 8 \times x + 49\\&= 70x + 56x + 80x^{2} + 49\\&= (70 + 56) \times x + 80x^{2} + 49\\&= 80x^{2} + 126x + 49
\end{align*}
\item
\begin{align*}
B &= (5x - 3)(- 2x - 3)\\&= 5x \times - 2x + 5x \times - 3 - 3 \times - 2x - 3 \times - 3\\&= 5 \times - 2 \times x^{1 + 1} - 3 \times 5 \times x - 3 \times - 2 \times x + 9\\&= - 15x + 6x - 10x^{2} + 9\\&= (- 15 + 6) \times x - 10x^{2} + 9\\&= - 10x^{2} - 9x + 9
\end{align*}
\item
\begin{align*}
C &= (- 3x + 8)^{2}\\&= (- 3x + 8)(- 3x + 8)\\&= - 3x \times - 3x - 3x \times 8 + 8 \times - 3x + 8 \times 8\\&= - 3 \times - 3 \times x^{1 + 1} + 8 \times - 3 \times x + 8 \times - 3 \times x + 64\\&= - 24x - 24x + 9x^{2} + 64\\&= (- 24 - 24) \times x + 9x^{2} + 64\\&= 9x^{2} - 48x + 64
\end{align*}
\item
\begin{align*}
D &= - 3 + x(- 9x + 7)\\&= - 3 + x \times - 9x + x \times 7\\&= - 9x^{2} + 7x - 3
\end{align*}
\item
\begin{align*}
E &= 9x^{2} + x(- 5x - 3)\\&= 9x^{2} + x \times - 5x + x \times - 3\\&= 9x^{2} - 5x^{2} - 3x\\&= 9x^{2} - 5x^{2} - 3x\\&= (9 - 5) \times x^{2} - 3x\\&= 4x^{2} - 3x
\end{align*}
\item
\begin{align*}
F &= 3(x + 4)(x - 7)\\&= (3x + 3 \times 4)(x - 7)\\&= (3x + 12)(x - 7)\\&= 3x \times x + 3x \times - 7 + 12x + 12 \times - 7\\&= - 7 \times 3 \times x - 84 + 3x^{2} + 12x\\&= - 21x - 84 + 3x^{2} + 12x\\&= 3x^{2} - 21x + 12x - 84\\&= 3x^{2} + (- 21 + 12) \times x - 84\\&= 3x^{2} - 9x - 84
\end{align*}
\end{enumerate}
\end{solution}
\begin{exercise}[subtitle={Étude de fonctions}]
Soit $f(x) = 3x^{2} - 21x - 54$ une fonction définie sur $\R$.
\begin{enumerate}
\item Calculer les valeurs suivantes
\[
f(1) \qquad f(-2)
\]
\item Dériver la fonction $f$
\item Étudier le signe de $f'$ puis en déduire les variations de $f$.
\item Est-ce que $f$ admet un maximum? un minimum? Calculer sa valeur.
\end{enumerate}
\end{exercise}
\begin{solution}
\begin{enumerate}
\item On remplace $x$ par les valeurs demandées
\[
f(1) = 3 \times 1^{2} - 21 \times 1 - 54=3 \times 1 - 21 - 54=3 - 75=- 72
\]
\[
f(-1) = 3 \times - 1^{2} - 21 \times - 1 - 54=3 \times 1 + 21 - 54=3 - 33=- 30
\]
\item Pas de solutions automatiques.
\item Pas de solutions automatiques.
\end{enumerate}
\end{solution}
%\printsolutionstype{exercise}
\end{document}
%%% Local Variables:
%%% mode: latex
%%% TeX-master: "master"
%%% End:

View File

@ -0,0 +1,141 @@
\documentclass[a5paper,10pt]{article}
\usepackage{myXsim}
\usepackage{tasks}
% Title Page
\title{DM1 \hfill BOUALIA Bilel}
\tribe{TST}
\date{Toussain 2020}
\begin{document}
\maketitle
\begin{exercise}[subtitle={Fractions}]
Faire les calculs avec les fraction suivants
\begin{multicols}{3}
\begin{enumerate}
\item $A = \dfrac{1}{4} - \dfrac{10}{4}$
\item $B = \dfrac{- 5}{6} - \dfrac{- 6}{30}$
\item $C = \dfrac{- 8}{2} + \dfrac{- 6}{1}$
\item $D = \dfrac{2}{10} - 10$
\item $E = \dfrac{1}{3} \times \dfrac{- 5}{2}$
\item $F = \dfrac{10}{2} \times - 10$
\end{enumerate}
\end{multicols}
\end{exercise}
\begin{solution}
\begin{enumerate}
\item
\[
\dfrac{1}{4} - \dfrac{10}{4}=\dfrac{1}{4} - \dfrac{10}{4}=\dfrac{1 - 10}{4}=\dfrac{1 - 10}{4}=\dfrac{- 9}{4}
\]
\item
\[
\dfrac{- 5}{6} - \dfrac{- 6}{30}=\dfrac{- 5}{6} + \dfrac{6}{30}=\dfrac{- 5 \times 5}{6 \times 5} + \dfrac{6}{30}=\dfrac{- 25}{30} + \dfrac{6}{30}=\dfrac{- 25 + 6}{30}=\dfrac{- 19}{30}
\]
\item
\[
\dfrac{- 8}{2} + \dfrac{- 6}{1}=\dfrac{- 8}{2} + \dfrac{- 6 \times 2}{1 \times 2}=\dfrac{- 8}{2} + \dfrac{- 12}{2}=\dfrac{- 8 - 12}{2}=\dfrac{- 20}{2}
\]
\item
\[
\dfrac{2}{10} - 10=\dfrac{2}{10} + \dfrac{- 10}{1}=\dfrac{2}{10} + \dfrac{- 10 \times 10}{1 \times 10}=\dfrac{2}{10} + \dfrac{- 100}{10}=\dfrac{2 - 100}{10}=\dfrac{- 98}{10}
\]
\item
\[
\dfrac{1}{3} \times \dfrac{- 5}{2}=\dfrac{1 \times - 5}{3 \times 2}=\dfrac{- 5}{6}
\]
\item
\[
\dfrac{10}{2} \times - 10=\dfrac{10 \times - 10}{2}=\dfrac{- 100}{2}
\]
\end{enumerate}
\end{solution}
\begin{exercise}[subtitle={Développer réduire}]
Développer puis réduire les expressions suivantes
\begin{multicols}{2}
\begin{enumerate}
\item $A = (4x - 7)(8x - 7)$
\item $B = (- 2x + 4)(7x + 4)$
\item $C = (- 4x - 5)^{2}$
\item $D = - 3 + x(- 4x - 4)$
\item $E = 10x^{2} + x(9x - 10)$
\item $F = 7(x + 8)(x + 9)$
\end{enumerate}
\end{multicols}
\end{exercise}
\begin{solution}
\begin{enumerate}
\item
\begin{align*}
A &= (4x - 7)(8x - 7)\\&= 4x \times 8x + 4x \times - 7 - 7 \times 8x - 7 \times - 7\\&= 4 \times 8 \times x^{1 + 1} - 7 \times 4 \times x - 7 \times 8 \times x + 49\\&= - 28x - 56x + 32x^{2} + 49\\&= (- 28 - 56) \times x + 32x^{2} + 49\\&= 32x^{2} - 84x + 49
\end{align*}
\item
\begin{align*}
B &= (- 2x + 4)(7x + 4)\\&= - 2x \times 7x - 2x \times 4 + 4 \times 7x + 4 \times 4\\&= - 2 \times 7 \times x^{1 + 1} + 4 \times - 2 \times x + 4 \times 7 \times x + 16\\&= - 8x + 28x - 14x^{2} + 16\\&= (- 8 + 28) \times x - 14x^{2} + 16\\&= - 14x^{2} + 20x + 16
\end{align*}
\item
\begin{align*}
C &= (- 4x - 5)^{2}\\&= (- 4x - 5)(- 4x - 5)\\&= - 4x \times - 4x - 4x \times - 5 - 5 \times - 4x - 5 \times - 5\\&= - 4 \times - 4 \times x^{1 + 1} - 5 \times - 4 \times x - 5 \times - 4 \times x + 25\\&= 20x + 20x + 16x^{2} + 25\\&= (20 + 20) \times x + 16x^{2} + 25\\&= 16x^{2} + 40x + 25
\end{align*}
\item
\begin{align*}
D &= - 3 + x(- 4x - 4)\\&= - 3 + x \times - 4x + x \times - 4\\&= - 4x^{2} - 4x - 3
\end{align*}
\item
\begin{align*}
E &= 10x^{2} + x(9x - 10)\\&= 10x^{2} + x \times 9x + x \times - 10\\&= 10x^{2} + 9x^{2} - 10x\\&= 10x^{2} + 9x^{2} - 10x\\&= (10 + 9) \times x^{2} - 10x\\&= 19x^{2} - 10x
\end{align*}
\item
\begin{align*}
F &= 7(x + 8)(x + 9)\\&= (7x + 7 \times 8)(x + 9)\\&= (7x + 56)(x + 9)\\&= 7x \times x + 7x \times 9 + 56x + 56 \times 9\\&= 9 \times 7 \times x + 504 + 7x^{2} + 56x\\&= 63x + 504 + 7x^{2} + 56x\\&= 7x^{2} + 63x + 56x + 504\\&= 7x^{2} + (63 + 56) \times x + 504\\&= 7x^{2} + 119x + 504
\end{align*}
\end{enumerate}
\end{solution}
\begin{exercise}[subtitle={Étude de fonctions}]
Soit $f(x) = 4x^{2} - 20x - 24$ une fonction définie sur $\R$.
\begin{enumerate}
\item Calculer les valeurs suivantes
\[
f(1) \qquad f(-2)
\]
\item Dériver la fonction $f$
\item Étudier le signe de $f'$ puis en déduire les variations de $f$.
\item Est-ce que $f$ admet un maximum? un minimum? Calculer sa valeur.
\end{enumerate}
\end{exercise}
\begin{solution}
\begin{enumerate}
\item On remplace $x$ par les valeurs demandées
\[
f(1) = 4 \times 1^{2} - 20 \times 1 - 24=4 \times 1 - 20 - 24=4 - 44=- 40
\]
\[
f(-1) = 4 \times - 1^{2} - 20 \times - 1 - 24=4 \times 1 + 20 - 24=4 - 4=0
\]
\item Pas de solutions automatiques.
\item Pas de solutions automatiques.
\end{enumerate}
\end{solution}
%\printsolutionstype{exercise}
\end{document}
%%% Local Variables:
%%% mode: latex
%%% TeX-master: "master"
%%% End:

View File

@ -0,0 +1,141 @@
\documentclass[a5paper,10pt]{article}
\usepackage{myXsim}
\usepackage{tasks}
% Title Page
\title{DM1 \hfill BOUCHOUX Kevin}
\tribe{TST}
\date{Toussain 2020}
\begin{document}
\maketitle
\begin{exercise}[subtitle={Fractions}]
Faire les calculs avec les fraction suivants
\begin{multicols}{3}
\begin{enumerate}
\item $A = \dfrac{- 2}{8} - \dfrac{- 8}{8}$
\item $B = \dfrac{8}{7} - \dfrac{5}{56}$
\item $C = \dfrac{2}{8} + \dfrac{8}{7}$
\item $D = \dfrac{4}{7} + 9$
\item $E = \dfrac{- 10}{3} \times \dfrac{7}{2}$
\item $F = \dfrac{4}{4} \times 5$
\end{enumerate}
\end{multicols}
\end{exercise}
\begin{solution}
\begin{enumerate}
\item
\[
\dfrac{- 2}{8} - \dfrac{- 8}{8}=\dfrac{- 2}{8} + \dfrac{8}{8}=\dfrac{- 2 + 8}{8}=\dfrac{6}{8}
\]
\item
\[
\dfrac{8}{7} - \dfrac{5}{56}=\dfrac{8}{7} - \dfrac{5}{56}=\dfrac{8 \times 8}{7 \times 8} - \dfrac{5}{56}=\dfrac{64}{56} - \dfrac{5}{56}=\dfrac{64 - 5}{56}=\dfrac{64 - 5}{56}=\dfrac{59}{56}
\]
\item
\[
\dfrac{2}{8} + \dfrac{8}{7}=\dfrac{2 \times 7}{8 \times 7} + \dfrac{8 \times 8}{7 \times 8}=\dfrac{14}{56} + \dfrac{64}{56}=\dfrac{14 + 64}{56}=\dfrac{78}{56}
\]
\item
\[
\dfrac{4}{7} + 9=\dfrac{4}{7} + \dfrac{9}{1}=\dfrac{4}{7} + \dfrac{9 \times 7}{1 \times 7}=\dfrac{4}{7} + \dfrac{63}{7}=\dfrac{4 + 63}{7}=\dfrac{67}{7}
\]
\item
\[
\dfrac{- 10}{3} \times \dfrac{7}{2}=\dfrac{- 10 \times 7}{3 \times 2}=\dfrac{- 70}{6}
\]
\item
\[
\dfrac{4}{4} \times 5=\dfrac{4 \times 5}{4}=\dfrac{20}{4}
\]
\end{enumerate}
\end{solution}
\begin{exercise}[subtitle={Développer réduire}]
Développer puis réduire les expressions suivantes
\begin{multicols}{2}
\begin{enumerate}
\item $A = (5x + 10)(5x + 10)$
\item $B = (- 10x + 9)(9x + 9)$
\item $C = (- 9x - 6)^{2}$
\item $D = - 8 + x(3x + 9)$
\item $E = 10x^{2} + x(7x - 2)$
\item $F = - 6(x + 10)(x + 8)$
\end{enumerate}
\end{multicols}
\end{exercise}
\begin{solution}
\begin{enumerate}
\item
\begin{align*}
A &= (5x + 10)(5x + 10)\\&= 5x \times 5x + 5x \times 10 + 10 \times 5x + 10 \times 10\\&= 5 \times 5 \times x^{1 + 1} + 10 \times 5 \times x + 10 \times 5 \times x + 100\\&= 50x + 50x + 25x^{2} + 100\\&= (50 + 50) \times x + 25x^{2} + 100\\&= 25x^{2} + 100x + 100
\end{align*}
\item
\begin{align*}
B &= (- 10x + 9)(9x + 9)\\&= - 10x \times 9x - 10x \times 9 + 9 \times 9x + 9 \times 9\\&= - 10 \times 9 \times x^{1 + 1} + 9 \times - 10 \times x + 9 \times 9 \times x + 81\\&= - 90x + 81x - 90x^{2} + 81\\&= (- 90 + 81) \times x - 90x^{2} + 81\\&= - 90x^{2} - 9x + 81
\end{align*}
\item
\begin{align*}
C &= (- 9x - 6)^{2}\\&= (- 9x - 6)(- 9x - 6)\\&= - 9x \times - 9x - 9x \times - 6 - 6 \times - 9x - 6 \times - 6\\&= - 9 \times - 9 \times x^{1 + 1} - 6 \times - 9 \times x - 6 \times - 9 \times x + 36\\&= 54x + 54x + 81x^{2} + 36\\&= (54 + 54) \times x + 81x^{2} + 36\\&= 81x^{2} + 108x + 36
\end{align*}
\item
\begin{align*}
D &= - 8 + x(3x + 9)\\&= - 8 + x \times 3x + x \times 9\\&= 3x^{2} + 9x - 8
\end{align*}
\item
\begin{align*}
E &= 10x^{2} + x(7x - 2)\\&= 10x^{2} + x \times 7x + x \times - 2\\&= 10x^{2} + 7x^{2} - 2x\\&= 10x^{2} + 7x^{2} - 2x\\&= (10 + 7) \times x^{2} - 2x\\&= 17x^{2} - 2x
\end{align*}
\item
\begin{align*}
F &= - 6(x + 10)(x + 8)\\&= (- 6x - 6 \times 10)(x + 8)\\&= (- 6x - 60)(x + 8)\\&= - 6x \times x - 6x \times 8 - 60x - 60 \times 8\\&= 8 \times - 6 \times x - 480 - 6x^{2} - 60x\\&= - 48x - 480 - 6x^{2} - 60x\\&= - 6x^{2} - 48x - 60x - 480\\&= - 6x^{2} + (- 48 - 60) \times x - 480\\&= - 6x^{2} - 108x - 480
\end{align*}
\end{enumerate}
\end{solution}
\begin{exercise}[subtitle={Étude de fonctions}]
Soit $f(x) = 4x^{2} + 20x - 200$ une fonction définie sur $\R$.
\begin{enumerate}
\item Calculer les valeurs suivantes
\[
f(1) \qquad f(-2)
\]
\item Dériver la fonction $f$
\item Étudier le signe de $f'$ puis en déduire les variations de $f$.
\item Est-ce que $f$ admet un maximum? un minimum? Calculer sa valeur.
\end{enumerate}
\end{exercise}
\begin{solution}
\begin{enumerate}
\item On remplace $x$ par les valeurs demandées
\[
f(1) = 4 \times 1^{2} + 20 \times 1 - 200=4 \times 1 + 20 - 200=4 - 180=- 176
\]
\[
f(-1) = 4 \times - 1^{2} + 20 \times - 1 - 200=4 \times 1 - 20 - 200=4 - 220=- 216
\]
\item Pas de solutions automatiques.
\item Pas de solutions automatiques.
\end{enumerate}
\end{solution}
%\printsolutionstype{exercise}
\end{document}
%%% Local Variables:
%%% mode: latex
%%% TeX-master: "master"
%%% End:

View File

@ -0,0 +1,141 @@
\documentclass[a5paper,10pt]{article}
\usepackage{myXsim}
\usepackage{tasks}
% Title Page
\title{DM1 \hfill BUDIN Nathan}
\tribe{TST}
\date{Toussain 2020}
\begin{document}
\maketitle
\begin{exercise}[subtitle={Fractions}]
Faire les calculs avec les fraction suivants
\begin{multicols}{3}
\begin{enumerate}
\item $A = \dfrac{- 1}{6} - \dfrac{1}{6}$
\item $B = \dfrac{9}{5} - \dfrac{1}{40}$
\item $C = \dfrac{6}{7} + \dfrac{5}{6}$
\item $D = \dfrac{2}{10} + 6$
\item $E = \dfrac{- 5}{10} \times \dfrac{- 9}{9}$
\item $F = \dfrac{- 5}{5} \times 2$
\end{enumerate}
\end{multicols}
\end{exercise}
\begin{solution}
\begin{enumerate}
\item
\[
\dfrac{- 1}{6} - \dfrac{1}{6}=\dfrac{- 1}{6} - \dfrac{1}{6}=\dfrac{- 1 - 1}{6}=\dfrac{- 1 - 1}{6}=\dfrac{- 2}{6}
\]
\item
\[
\dfrac{9}{5} - \dfrac{1}{40}=\dfrac{9}{5} - \dfrac{1}{40}=\dfrac{9 \times 8}{5 \times 8} - \dfrac{1}{40}=\dfrac{72}{40} - \dfrac{1}{40}=\dfrac{72 - 1}{40}=\dfrac{72 - 1}{40}=\dfrac{71}{40}
\]
\item
\[
\dfrac{6}{7} + \dfrac{5}{6}=\dfrac{6 \times 6}{7 \times 6} + \dfrac{5 \times 7}{6 \times 7}=\dfrac{36}{42} + \dfrac{35}{42}=\dfrac{36 + 35}{42}=\dfrac{71}{42}
\]
\item
\[
\dfrac{2}{10} + 6=\dfrac{2}{10} + \dfrac{6}{1}=\dfrac{2}{10} + \dfrac{6 \times 10}{1 \times 10}=\dfrac{2}{10} + \dfrac{60}{10}=\dfrac{2 + 60}{10}=\dfrac{62}{10}
\]
\item
\[
\dfrac{- 5}{10} \times \dfrac{- 9}{9}=\dfrac{- 5 \times - 9}{10 \times 9}=\dfrac{45}{90}
\]
\item
\[
\dfrac{- 5}{5} \times 2=\dfrac{- 5 \times 2}{5}=\dfrac{- 10}{5}
\]
\end{enumerate}
\end{solution}
\begin{exercise}[subtitle={Développer réduire}]
Développer puis réduire les expressions suivantes
\begin{multicols}{2}
\begin{enumerate}
\item $A = (- 4x + 2)(- 8x + 2)$
\item $B = (10x - 5)(5x - 5)$
\item $C = (2x - 6)^{2}$
\item $D = 1 + x(- 3x - 2)$
\item $E = 5x^{2} + x(7x + 3)$
\item $F = 2(x - 2)(x - 8)$
\end{enumerate}
\end{multicols}
\end{exercise}
\begin{solution}
\begin{enumerate}
\item
\begin{align*}
A &= (- 4x + 2)(- 8x + 2)\\&= - 4x \times - 8x - 4x \times 2 + 2 \times - 8x + 2 \times 2\\&= - 4 \times - 8 \times x^{1 + 1} + 2 \times - 4 \times x + 2 \times - 8 \times x + 4\\&= - 8x - 16x + 32x^{2} + 4\\&= (- 8 - 16) \times x + 32x^{2} + 4\\&= 32x^{2} - 24x + 4
\end{align*}
\item
\begin{align*}
B &= (10x - 5)(5x - 5)\\&= 10x \times 5x + 10x \times - 5 - 5 \times 5x - 5 \times - 5\\&= 10 \times 5 \times x^{1 + 1} - 5 \times 10 \times x - 5 \times 5 \times x + 25\\&= - 50x - 25x + 50x^{2} + 25\\&= (- 50 - 25) \times x + 50x^{2} + 25\\&= 50x^{2} - 75x + 25
\end{align*}
\item
\begin{align*}
C &= (2x - 6)^{2}\\&= (2x - 6)(2x - 6)\\&= 2x \times 2x + 2x \times - 6 - 6 \times 2x - 6 \times - 6\\&= 2 \times 2 \times x^{1 + 1} - 6 \times 2 \times x - 6 \times 2 \times x + 36\\&= - 12x - 12x + 4x^{2} + 36\\&= (- 12 - 12) \times x + 4x^{2} + 36\\&= 4x^{2} - 24x + 36
\end{align*}
\item
\begin{align*}
D &= 1 + x(- 3x - 2)\\&= 1 + x \times - 3x + x \times - 2\\&= - 3x^{2} - 2x + 1
\end{align*}
\item
\begin{align*}
E &= 5x^{2} + x(7x + 3)\\&= 5x^{2} + x \times 7x + x \times 3\\&= 5x^{2} + 7x^{2} + 3x\\&= 5x^{2} + 7x^{2} + 3x\\&= (5 + 7) \times x^{2} + 3x\\&= 12x^{2} + 3x
\end{align*}
\item
\begin{align*}
F &= 2(x - 2)(x - 8)\\&= (2x + 2 \times - 2)(x - 8)\\&= (2x - 4)(x - 8)\\&= 2x \times x + 2x \times - 8 - 4x - 4 \times - 8\\&= - 8 \times 2 \times x + 32 + 2x^{2} - 4x\\&= - 16x + 32 + 2x^{2} - 4x\\&= 2x^{2} - 16x - 4x + 32\\&= 2x^{2} + (- 16 - 4) \times x + 32\\&= 2x^{2} - 20x + 32
\end{align*}
\end{enumerate}
\end{solution}
\begin{exercise}[subtitle={Étude de fonctions}]
Soit $f(x) = - 4x^{2} + 32x + 36$ une fonction définie sur $\R$.
\begin{enumerate}
\item Calculer les valeurs suivantes
\[
f(1) \qquad f(-2)
\]
\item Dériver la fonction $f$
\item Étudier le signe de $f'$ puis en déduire les variations de $f$.
\item Est-ce que $f$ admet un maximum? un minimum? Calculer sa valeur.
\end{enumerate}
\end{exercise}
\begin{solution}
\begin{enumerate}
\item On remplace $x$ par les valeurs demandées
\[
f(1) = - 4 \times 1^{2} + 32 \times 1 + 36=- 4 \times 1 + 32 + 36=- 4 + 68=64
\]
\[
f(-1) = - 4 \times - 1^{2} + 32 \times - 1 + 36=- 4 \times 1 - 32 + 36=- 4 + 4=0
\]
\item Pas de solutions automatiques.
\item Pas de solutions automatiques.
\end{enumerate}
\end{solution}
%\printsolutionstype{exercise}
\end{document}
%%% Local Variables:
%%% mode: latex
%%% TeX-master: "master"
%%% End:

View File

@ -0,0 +1,141 @@
\documentclass[a5paper,10pt]{article}
\usepackage{myXsim}
\usepackage{tasks}
% Title Page
\title{DM1 \hfill CAGLAR Rojin}
\tribe{TST}
\date{Toussain 2020}
\begin{document}
\maketitle
\begin{exercise}[subtitle={Fractions}]
Faire les calculs avec les fraction suivants
\begin{multicols}{3}
\begin{enumerate}
\item $A = \dfrac{- 9}{4} - \dfrac{9}{4}$
\item $B = \dfrac{4}{5} - \dfrac{- 10}{45}$
\item $C = \dfrac{- 6}{3} + \dfrac{- 5}{2}$
\item $D = \dfrac{8}{5} + 7$
\item $E = \dfrac{6}{8} \times \dfrac{- 5}{7}$
\item $F = \dfrac{7}{6} \times 4$
\end{enumerate}
\end{multicols}
\end{exercise}
\begin{solution}
\begin{enumerate}
\item
\[
\dfrac{- 9}{4} - \dfrac{9}{4}=\dfrac{- 9}{4} - \dfrac{9}{4}=\dfrac{- 9 - 9}{4}=\dfrac{- 9 - 9}{4}=\dfrac{- 18}{4}
\]
\item
\[
\dfrac{4}{5} - \dfrac{- 10}{45}=\dfrac{4}{5} + \dfrac{10}{45}=\dfrac{4 \times 9}{5 \times 9} + \dfrac{10}{45}=\dfrac{36}{45} + \dfrac{10}{45}=\dfrac{36 + 10}{45}=\dfrac{46}{45}
\]
\item
\[
\dfrac{- 6}{3} + \dfrac{- 5}{2}=\dfrac{- 6 \times 2}{3 \times 2} + \dfrac{- 5 \times 3}{2 \times 3}=\dfrac{- 12}{6} + \dfrac{- 15}{6}=\dfrac{- 12 - 15}{6}=\dfrac{- 27}{6}
\]
\item
\[
\dfrac{8}{5} + 7=\dfrac{8}{5} + \dfrac{7}{1}=\dfrac{8}{5} + \dfrac{7 \times 5}{1 \times 5}=\dfrac{8}{5} + \dfrac{35}{5}=\dfrac{8 + 35}{5}=\dfrac{43}{5}
\]
\item
\[
\dfrac{6}{8} \times \dfrac{- 5}{7}=\dfrac{6 \times - 5}{8 \times 7}=\dfrac{- 30}{56}
\]
\item
\[
\dfrac{7}{6} \times 4=\dfrac{7 \times 4}{6}=\dfrac{28}{6}
\]
\end{enumerate}
\end{solution}
\begin{exercise}[subtitle={Développer réduire}]
Développer puis réduire les expressions suivantes
\begin{multicols}{2}
\begin{enumerate}
\item $A = (4x + 8)(- 4x + 8)$
\item $B = (6x - 9)(- 7x - 9)$
\item $C = (- 9x + 7)^{2}$
\item $D = - 7 + x(3x - 5)$
\item $E = 8x^{2} + x(6x - 7)$
\item $F = 3(x - 9)(x - 2)$
\end{enumerate}
\end{multicols}
\end{exercise}
\begin{solution}
\begin{enumerate}
\item
\begin{align*}
A &= (4x + 8)(- 4x + 8)\\&= 4x \times - 4x + 4x \times 8 + 8 \times - 4x + 8 \times 8\\&= 4 \times - 4 \times x^{1 + 1} + 8 \times 4 \times x + 8 \times - 4 \times x + 64\\&= 32x - 32x - 16x^{2} + 64\\&= (32 - 32) \times x - 16x^{2} + 64\\&= 0x - 16x^{2} + 64\\&= - 16x^{2} + 64
\end{align*}
\item
\begin{align*}
B &= (6x - 9)(- 7x - 9)\\&= 6x \times - 7x + 6x \times - 9 - 9 \times - 7x - 9 \times - 9\\&= 6 \times - 7 \times x^{1 + 1} - 9 \times 6 \times x - 9 \times - 7 \times x + 81\\&= - 54x + 63x - 42x^{2} + 81\\&= (- 54 + 63) \times x - 42x^{2} + 81\\&= - 42x^{2} + 9x + 81
\end{align*}
\item
\begin{align*}
C &= (- 9x + 7)^{2}\\&= (- 9x + 7)(- 9x + 7)\\&= - 9x \times - 9x - 9x \times 7 + 7 \times - 9x + 7 \times 7\\&= - 9 \times - 9 \times x^{1 + 1} + 7 \times - 9 \times x + 7 \times - 9 \times x + 49\\&= - 63x - 63x + 81x^{2} + 49\\&= (- 63 - 63) \times x + 81x^{2} + 49\\&= 81x^{2} - 126x + 49
\end{align*}
\item
\begin{align*}
D &= - 7 + x(3x - 5)\\&= - 7 + x \times 3x + x \times - 5\\&= 3x^{2} - 5x - 7
\end{align*}
\item
\begin{align*}
E &= 8x^{2} + x(6x - 7)\\&= 8x^{2} + x \times 6x + x \times - 7\\&= 8x^{2} + 6x^{2} - 7x\\&= 8x^{2} + 6x^{2} - 7x\\&= (8 + 6) \times x^{2} - 7x\\&= 14x^{2} - 7x
\end{align*}
\item
\begin{align*}
F &= 3(x - 9)(x - 2)\\&= (3x + 3 \times - 9)(x - 2)\\&= (3x - 27)(x - 2)\\&= 3x \times x + 3x \times - 2 - 27x - 27 \times - 2\\&= - 2 \times 3 \times x + 54 + 3x^{2} - 27x\\&= - 6x + 54 + 3x^{2} - 27x\\&= 3x^{2} - 6x - 27x + 54\\&= 3x^{2} + (- 6 - 27) \times x + 54\\&= 3x^{2} - 33x + 54
\end{align*}
\end{enumerate}
\end{solution}
\begin{exercise}[subtitle={Étude de fonctions}]
Soit $f(x) = - 5x^{2} + 25x + 250$ une fonction définie sur $\R$.
\begin{enumerate}
\item Calculer les valeurs suivantes
\[
f(1) \qquad f(-2)
\]
\item Dériver la fonction $f$
\item Étudier le signe de $f'$ puis en déduire les variations de $f$.
\item Est-ce que $f$ admet un maximum? un minimum? Calculer sa valeur.
\end{enumerate}
\end{exercise}
\begin{solution}
\begin{enumerate}
\item On remplace $x$ par les valeurs demandées
\[
f(1) = - 5 \times 1^{2} + 25 \times 1 + 250=- 5 \times 1 + 25 + 250=- 5 + 275=270
\]
\[
f(-1) = - 5 \times - 1^{2} + 25 \times - 1 + 250=- 5 \times 1 - 25 + 250=- 5 + 225=220
\]
\item Pas de solutions automatiques.
\item Pas de solutions automatiques.
\end{enumerate}
\end{solution}
%\printsolutionstype{exercise}
\end{document}
%%% Local Variables:
%%% mode: latex
%%% TeX-master: "master"
%%% End:

View File

@ -0,0 +1,141 @@
\documentclass[a5paper,10pt]{article}
\usepackage{myXsim}
\usepackage{tasks}
% Title Page
\title{DM1 \hfill DARICHE Kaïs}
\tribe{TST}
\date{Toussain 2020}
\begin{document}
\maketitle
\begin{exercise}[subtitle={Fractions}]
Faire les calculs avec les fraction suivants
\begin{multicols}{3}
\begin{enumerate}
\item $A = \dfrac{9}{3} - \dfrac{3}{3}$
\item $B = \dfrac{6}{2} - \dfrac{- 1}{14}$
\item $C = \dfrac{4}{7} + \dfrac{10}{6}$
\item $D = \dfrac{- 3}{4} + 4$
\item $E = \dfrac{2}{7} \times \dfrac{- 9}{6}$
\item $F = \dfrac{- 9}{6} \times - 10$
\end{enumerate}
\end{multicols}
\end{exercise}
\begin{solution}
\begin{enumerate}
\item
\[
\dfrac{9}{3} - \dfrac{3}{3}=\dfrac{9}{3} - \dfrac{3}{3}=\dfrac{9 - 3}{3}=\dfrac{9 - 3}{3}=\dfrac{6}{3}
\]
\item
\[
\dfrac{6}{2} - \dfrac{- 1}{14}=\dfrac{6}{2} + \dfrac{1}{14}=\dfrac{6 \times 7}{2 \times 7} + \dfrac{1}{14}=\dfrac{42}{14} + \dfrac{1}{14}=\dfrac{42 + 1}{14}=\dfrac{43}{14}
\]
\item
\[
\dfrac{4}{7} + \dfrac{10}{6}=\dfrac{4 \times 6}{7 \times 6} + \dfrac{10 \times 7}{6 \times 7}=\dfrac{24}{42} + \dfrac{70}{42}=\dfrac{24 + 70}{42}=\dfrac{94}{42}
\]
\item
\[
\dfrac{- 3}{4} + 4=\dfrac{- 3}{4} + \dfrac{4}{1}=\dfrac{- 3}{4} + \dfrac{4 \times 4}{1 \times 4}=\dfrac{- 3}{4} + \dfrac{16}{4}=\dfrac{- 3 + 16}{4}=\dfrac{13}{4}
\]
\item
\[
\dfrac{2}{7} \times \dfrac{- 9}{6}=\dfrac{2 \times - 9}{7 \times 6}=\dfrac{- 18}{42}
\]
\item
\[
\dfrac{- 9}{6} \times - 10=\dfrac{- 9 \times - 10}{6}=\dfrac{90}{6}
\]
\end{enumerate}
\end{solution}
\begin{exercise}[subtitle={Développer réduire}]
Développer puis réduire les expressions suivantes
\begin{multicols}{2}
\begin{enumerate}
\item $A = (- 6x - 6)(- 1x - 6)$
\item $B = (- 2x - 1)(- 8x - 1)$
\item $C = (- 9x + 2)^{2}$
\item $D = - 6 + x(- 10x - 6)$
\item $E = 8x^{2} + x(- 9x + 2)$
\item $F = 6(x - 7)(x - 9)$
\end{enumerate}
\end{multicols}
\end{exercise}
\begin{solution}
\begin{enumerate}
\item
\begin{align*}
A &= (- 6x - 6)(- 1x - 6)\\&= - 6x \times - x - 6x \times - 6 - 6 \times - x - 6 \times - 6\\&= - 6 \times - 1 \times x^{1 + 1} - 6 \times - 6 \times x - 6 \times - 1 \times x + 36\\&= 36x + 6x + 6x^{2} + 36\\&= (36 + 6) \times x + 6x^{2} + 36\\&= 6x^{2} + 42x + 36
\end{align*}
\item
\begin{align*}
B &= (- 2x - 1)(- 8x - 1)\\&= - 2x \times - 8x - 2x \times - 1 - 1 \times - 8x - 1 \times - 1\\&= - 2 \times - 8 \times x^{1 + 1} - 1 \times - 2 \times x - 1 \times - 8 \times x + 1\\&= 2x + 8x + 16x^{2} + 1\\&= (2 + 8) \times x + 16x^{2} + 1\\&= 16x^{2} + 10x + 1
\end{align*}
\item
\begin{align*}
C &= (- 9x + 2)^{2}\\&= (- 9x + 2)(- 9x + 2)\\&= - 9x \times - 9x - 9x \times 2 + 2 \times - 9x + 2 \times 2\\&= - 9 \times - 9 \times x^{1 + 1} + 2 \times - 9 \times x + 2 \times - 9 \times x + 4\\&= - 18x - 18x + 81x^{2} + 4\\&= (- 18 - 18) \times x + 81x^{2} + 4\\&= 81x^{2} - 36x + 4
\end{align*}
\item
\begin{align*}
D &= - 6 + x(- 10x - 6)\\&= - 6 + x \times - 10x + x \times - 6\\&= - 10x^{2} - 6x - 6
\end{align*}
\item
\begin{align*}
E &= 8x^{2} + x(- 9x + 2)\\&= 8x^{2} + x \times - 9x + x \times 2\\&= 8x^{2} - 9x^{2} + 2x\\&= 8x^{2} - 9x^{2} + 2x\\&= (8 - 9) \times x^{2} + 2x\\&= - x^{2} + 2x
\end{align*}
\item
\begin{align*}
F &= 6(x - 7)(x - 9)\\&= (6x + 6 \times - 7)(x - 9)\\&= (6x - 42)(x - 9)\\&= 6x \times x + 6x \times - 9 - 42x - 42 \times - 9\\&= - 9 \times 6 \times x + 378 + 6x^{2} - 42x\\&= - 54x + 378 + 6x^{2} - 42x\\&= 6x^{2} - 54x - 42x + 378\\&= 6x^{2} + (- 54 - 42) \times x + 378\\&= 6x^{2} - 96x + 378
\end{align*}
\end{enumerate}
\end{solution}
\begin{exercise}[subtitle={Étude de fonctions}]
Soit $f(x) = - 5x^{2} - 15x + 140$ une fonction définie sur $\R$.
\begin{enumerate}
\item Calculer les valeurs suivantes
\[
f(1) \qquad f(-2)
\]
\item Dériver la fonction $f$
\item Étudier le signe de $f'$ puis en déduire les variations de $f$.
\item Est-ce que $f$ admet un maximum? un minimum? Calculer sa valeur.
\end{enumerate}
\end{exercise}
\begin{solution}
\begin{enumerate}
\item On remplace $x$ par les valeurs demandées
\[
f(1) = - 5 \times 1^{2} - 15 \times 1 + 140=- 5 \times 1 - 15 + 140=- 5 + 125=120
\]
\[
f(-1) = - 5 \times - 1^{2} - 15 \times - 1 + 140=- 5 \times 1 + 15 + 140=- 5 + 155=150
\]
\item Pas de solutions automatiques.
\item Pas de solutions automatiques.
\end{enumerate}
\end{solution}
%\printsolutionstype{exercise}
\end{document}
%%% Local Variables:
%%% mode: latex
%%% TeX-master: "master"
%%% End:

View File

@ -0,0 +1,141 @@
\documentclass[a5paper,10pt]{article}
\usepackage{myXsim}
\usepackage{tasks}
% Title Page
\title{DM1 \hfill DEBRAS Noémie}
\tribe{TST}
\date{Toussain 2020}
\begin{document}
\maketitle
\begin{exercise}[subtitle={Fractions}]
Faire les calculs avec les fraction suivants
\begin{multicols}{3}
\begin{enumerate}
\item $A = \dfrac{- 4}{10} - \dfrac{- 6}{10}$
\item $B = \dfrac{4}{10} - \dfrac{10}{100}$
\item $C = \dfrac{8}{8} + \dfrac{- 5}{7}$
\item $D = \dfrac{2}{8} - 1$
\item $E = \dfrac{- 8}{9} \times \dfrac{- 8}{8}$
\item $F = \dfrac{- 6}{5} \times 7$
\end{enumerate}
\end{multicols}
\end{exercise}
\begin{solution}
\begin{enumerate}
\item
\[
\dfrac{- 4}{10} - \dfrac{- 6}{10}=\dfrac{- 4}{10} + \dfrac{6}{10}=\dfrac{- 4 + 6}{10}=\dfrac{2}{10}
\]
\item
\[
\dfrac{4}{10} - \dfrac{10}{100}=\dfrac{4}{10} - \dfrac{10}{100}=\dfrac{4 \times 10}{10 \times 10} - \dfrac{10}{100}=\dfrac{40}{100} - \dfrac{10}{100}=\dfrac{40 - 10}{100}=\dfrac{40 - 10}{100}=\dfrac{30}{100}
\]
\item
\[
\dfrac{8}{8} + \dfrac{- 5}{7}=\dfrac{8 \times 7}{8 \times 7} + \dfrac{- 5 \times 8}{7 \times 8}=\dfrac{56}{56} + \dfrac{- 40}{56}=\dfrac{56 - 40}{56}=\dfrac{16}{56}
\]
\item
\[
\dfrac{2}{8} - 1=\dfrac{2}{8} + \dfrac{- 1}{1}=\dfrac{2}{8} + \dfrac{- 1 \times 8}{1 \times 8}=\dfrac{2}{8} + \dfrac{- 8}{8}=\dfrac{2 - 8}{8}=\dfrac{- 6}{8}
\]
\item
\[
\dfrac{- 8}{9} \times \dfrac{- 8}{8}=\dfrac{- 8 \times - 8}{9 \times 8}=\dfrac{64}{72}
\]
\item
\[
\dfrac{- 6}{5} \times 7=\dfrac{- 6 \times 7}{5}=\dfrac{- 42}{5}
\]
\end{enumerate}
\end{solution}
\begin{exercise}[subtitle={Développer réduire}]
Développer puis réduire les expressions suivantes
\begin{multicols}{2}
\begin{enumerate}
\item $A = (- 6x + 1)(- 2x + 1)$
\item $B = (- 5x + 3)(- 3x + 3)$
\item $C = (1x + 8)^{2}$
\item $D = - 8 + x(- 5x - 1)$
\item $E = 5x^{2} + x(9x + 8)$
\item $F = - 2(x + 5)(x - 5)$
\end{enumerate}
\end{multicols}
\end{exercise}
\begin{solution}
\begin{enumerate}
\item
\begin{align*}
A &= (- 6x + 1)(- 2x + 1)\\&= - 6x \times - 2x - 6x \times 1 + 1 \times - 2x + 1 \times 1\\&= - 6 \times - 2 \times x^{1 + 1} - 6x - 2x + 1\\&= 12x^{2} - 6x - 2x + 1\\&= 12x^{2} + (- 6 - 2) \times x + 1\\&= 12x^{2} - 8x + 1
\end{align*}
\item
\begin{align*}
B &= (- 5x + 3)(- 3x + 3)\\&= - 5x \times - 3x - 5x \times 3 + 3 \times - 3x + 3 \times 3\\&= - 5 \times - 3 \times x^{1 + 1} + 3 \times - 5 \times x + 3 \times - 3 \times x + 9\\&= - 15x - 9x + 15x^{2} + 9\\&= (- 15 - 9) \times x + 15x^{2} + 9\\&= 15x^{2} - 24x + 9
\end{align*}
\item
\begin{align*}
C &= (1x + 8)^{2}\\&= (x + 8)(x + 8)\\&= x \times x + x \times 8 + 8x + 8 \times 8\\&= x^{2} + 64 + (8 + 8) \times x\\&= x^{2} + 16x + 64
\end{align*}
\item
\begin{align*}
D &= - 8 + x(- 5x - 1)\\&= - 8 + x \times - 5x + x \times - 1\\&= - 5x^{2} - x - 8
\end{align*}
\item
\begin{align*}
E &= 5x^{2} + x(9x + 8)\\&= 5x^{2} + x \times 9x + x \times 8\\&= 5x^{2} + 9x^{2} + 8x\\&= 5x^{2} + 9x^{2} + 8x\\&= (5 + 9) \times x^{2} + 8x\\&= 14x^{2} + 8x
\end{align*}
\item
\begin{align*}
F &= - 2(x + 5)(x - 5)\\&= (- 2x - 2 \times 5)(x - 5)\\&= (- 2x - 10)(x - 5)\\&= - 2x \times x - 2x \times - 5 - 10x - 10 \times - 5\\&= - 5 \times - 2 \times x + 50 - 2x^{2} - 10x\\&= 10x + 50 - 2x^{2} - 10x\\&= - 2x^{2} + 10x - 10x + 50\\&= - 2x^{2} + (10 - 10) \times x + 50\\&= - 2x^{2} + 50 + 0x\\&= - 2x^{2} + 50
\end{align*}
\end{enumerate}
\end{solution}
\begin{exercise}[subtitle={Étude de fonctions}]
Soit $f(x) = - 8x^{2} + 48x + 56$ une fonction définie sur $\R$.
\begin{enumerate}
\item Calculer les valeurs suivantes
\[
f(1) \qquad f(-2)
\]
\item Dériver la fonction $f$
\item Étudier le signe de $f'$ puis en déduire les variations de $f$.
\item Est-ce que $f$ admet un maximum? un minimum? Calculer sa valeur.
\end{enumerate}
\end{exercise}
\begin{solution}
\begin{enumerate}
\item On remplace $x$ par les valeurs demandées
\[
f(1) = - 8 \times 1^{2} + 48 \times 1 + 56=- 8 \times 1 + 48 + 56=- 8 + 104=96
\]
\[
f(-1) = - 8 \times - 1^{2} + 48 \times - 1 + 56=- 8 \times 1 - 48 + 56=- 8 + 8=0
\]
\item Pas de solutions automatiques.
\item Pas de solutions automatiques.
\end{enumerate}
\end{solution}
%\printsolutionstype{exercise}
\end{document}
%%% Local Variables:
%%% mode: latex
%%% TeX-master: "master"
%%% End:

View File

@ -0,0 +1,141 @@
\documentclass[a5paper,10pt]{article}
\usepackage{myXsim}
\usepackage{tasks}
% Title Page
\title{DM1 \hfill GERMAIN Anaïs}
\tribe{TST}
\date{Toussain 2020}
\begin{document}
\maketitle
\begin{exercise}[subtitle={Fractions}]
Faire les calculs avec les fraction suivants
\begin{multicols}{3}
\begin{enumerate}
\item $A = \dfrac{- 3}{10} - \dfrac{- 1}{10}$
\item $B = \dfrac{7}{2} - \dfrac{5}{16}$
\item $C = \dfrac{- 7}{6} + \dfrac{8}{5}$
\item $D = \dfrac{9}{2} - 2$
\item $E = \dfrac{4}{8} \times \dfrac{7}{7}$
\item $F = \dfrac{3}{10} \times 9$
\end{enumerate}
\end{multicols}
\end{exercise}
\begin{solution}
\begin{enumerate}
\item
\[
\dfrac{- 3}{10} - \dfrac{- 1}{10}=\dfrac{- 3}{10} + \dfrac{1}{10}=\dfrac{- 3 + 1}{10}=\dfrac{- 2}{10}
\]
\item
\[
\dfrac{7}{2} - \dfrac{5}{16}=\dfrac{7}{2} - \dfrac{5}{16}=\dfrac{7 \times 8}{2 \times 8} - \dfrac{5}{16}=\dfrac{56}{16} - \dfrac{5}{16}=\dfrac{56 - 5}{16}=\dfrac{56 - 5}{16}=\dfrac{51}{16}
\]
\item
\[
\dfrac{- 7}{6} + \dfrac{8}{5}=\dfrac{- 7 \times 5}{6 \times 5} + \dfrac{8 \times 6}{5 \times 6}=\dfrac{- 35}{30} + \dfrac{48}{30}=\dfrac{- 35 + 48}{30}=\dfrac{13}{30}
\]
\item
\[
\dfrac{9}{2} - 2=\dfrac{9}{2} + \dfrac{- 2}{1}=\dfrac{9}{2} + \dfrac{- 2 \times 2}{1 \times 2}=\dfrac{9}{2} + \dfrac{- 4}{2}=\dfrac{9 - 4}{2}=\dfrac{5}{2}
\]
\item
\[
\dfrac{4}{8} \times \dfrac{7}{7}=\dfrac{4 \times 7}{8 \times 7}=\dfrac{28}{56}
\]
\item
\[
\dfrac{3}{10} \times 9=\dfrac{3 \times 9}{10}=\dfrac{27}{10}
\]
\end{enumerate}
\end{solution}
\begin{exercise}[subtitle={Développer réduire}]
Développer puis réduire les expressions suivantes
\begin{multicols}{2}
\begin{enumerate}
\item $A = (6x - 2)(- 9x - 2)$
\item $B = (- 3x + 2)(6x + 2)$
\item $C = (- 5x - 7)^{2}$
\item $D = 1 + x(5x - 7)$
\item $E = 7x^{2} + x(5x - 4)$
\item $F = - 6(x + 3)(x - 6)$
\end{enumerate}
\end{multicols}
\end{exercise}
\begin{solution}
\begin{enumerate}
\item
\begin{align*}
A &= (6x - 2)(- 9x - 2)\\&= 6x \times - 9x + 6x \times - 2 - 2 \times - 9x - 2 \times - 2\\&= 6 \times - 9 \times x^{1 + 1} - 2 \times 6 \times x - 2 \times - 9 \times x + 4\\&= - 12x + 18x - 54x^{2} + 4\\&= (- 12 + 18) \times x - 54x^{2} + 4\\&= - 54x^{2} + 6x + 4
\end{align*}
\item
\begin{align*}
B &= (- 3x + 2)(6x + 2)\\&= - 3x \times 6x - 3x \times 2 + 2 \times 6x + 2 \times 2\\&= - 3 \times 6 \times x^{1 + 1} + 2 \times - 3 \times x + 2 \times 6 \times x + 4\\&= - 6x + 12x - 18x^{2} + 4\\&= (- 6 + 12) \times x - 18x^{2} + 4\\&= - 18x^{2} + 6x + 4
\end{align*}
\item
\begin{align*}
C &= (- 5x - 7)^{2}\\&= (- 5x - 7)(- 5x - 7)\\&= - 5x \times - 5x - 5x \times - 7 - 7 \times - 5x - 7 \times - 7\\&= - 5 \times - 5 \times x^{1 + 1} - 7 \times - 5 \times x - 7 \times - 5 \times x + 49\\&= 35x + 35x + 25x^{2} + 49\\&= (35 + 35) \times x + 25x^{2} + 49\\&= 25x^{2} + 70x + 49
\end{align*}
\item
\begin{align*}
D &= 1 + x(5x - 7)\\&= 1 + x \times 5x + x \times - 7\\&= 5x^{2} - 7x + 1
\end{align*}
\item
\begin{align*}
E &= 7x^{2} + x(5x - 4)\\&= 7x^{2} + x \times 5x + x \times - 4\\&= 7x^{2} + 5x^{2} - 4x\\&= 7x^{2} + 5x^{2} - 4x\\&= (7 + 5) \times x^{2} - 4x\\&= 12x^{2} - 4x
\end{align*}
\item
\begin{align*}
F &= - 6(x + 3)(x - 6)\\&= (- 6x - 6 \times 3)(x - 6)\\&= (- 6x - 18)(x - 6)\\&= - 6x \times x - 6x \times - 6 - 18x - 18 \times - 6\\&= - 6 \times - 6 \times x + 108 - 6x^{2} - 18x\\&= 36x + 108 - 6x^{2} - 18x\\&= - 6x^{2} + 36x - 18x + 108\\&= - 6x^{2} + (36 - 18) \times x + 108\\&= - 6x^{2} + 18x + 108
\end{align*}
\end{enumerate}
\end{solution}
\begin{exercise}[subtitle={Étude de fonctions}]
Soit $f(x) = 3x^{2} - 51x + 210$ une fonction définie sur $\R$.
\begin{enumerate}
\item Calculer les valeurs suivantes
\[
f(1) \qquad f(-2)
\]
\item Dériver la fonction $f$
\item Étudier le signe de $f'$ puis en déduire les variations de $f$.
\item Est-ce que $f$ admet un maximum? un minimum? Calculer sa valeur.
\end{enumerate}
\end{exercise}
\begin{solution}
\begin{enumerate}
\item On remplace $x$ par les valeurs demandées
\[
f(1) = 3 \times 1^{2} - 51 \times 1 + 210=3 \times 1 - 51 + 210=3 + 159=162
\]
\[
f(-1) = 3 \times - 1^{2} - 51 \times - 1 + 210=3 \times 1 + 51 + 210=3 + 261=264
\]
\item Pas de solutions automatiques.
\item Pas de solutions automatiques.
\end{enumerate}
\end{solution}
%\printsolutionstype{exercise}
\end{document}
%%% Local Variables:
%%% mode: latex
%%% TeX-master: "master"
%%% End:

View File

@ -0,0 +1,141 @@
\documentclass[a5paper,10pt]{article}
\usepackage{myXsim}
\usepackage{tasks}
% Title Page
\title{DM1 \hfill HADJRAS Mohcine}
\tribe{TST}
\date{Toussain 2020}
\begin{document}
\maketitle
\begin{exercise}[subtitle={Fractions}]
Faire les calculs avec les fraction suivants
\begin{multicols}{3}
\begin{enumerate}
\item $A = \dfrac{5}{8} - \dfrac{9}{8}$
\item $B = \dfrac{- 7}{4} - \dfrac{- 10}{20}$
\item $C = \dfrac{- 8}{9} + \dfrac{- 8}{8}$
\item $D = \dfrac{9}{4} - 9$
\item $E = \dfrac{- 2}{6} \times \dfrac{7}{5}$
\item $F = \dfrac{7}{4} \times - 7$
\end{enumerate}
\end{multicols}
\end{exercise}
\begin{solution}
\begin{enumerate}
\item
\[
\dfrac{5}{8} - \dfrac{9}{8}=\dfrac{5}{8} - \dfrac{9}{8}=\dfrac{5 - 9}{8}=\dfrac{5 - 9}{8}=\dfrac{- 4}{8}
\]
\item
\[
\dfrac{- 7}{4} - \dfrac{- 10}{20}=\dfrac{- 7}{4} + \dfrac{10}{20}=\dfrac{- 7 \times 5}{4 \times 5} + \dfrac{10}{20}=\dfrac{- 35}{20} + \dfrac{10}{20}=\dfrac{- 35 + 10}{20}=\dfrac{- 25}{20}
\]
\item
\[
\dfrac{- 8}{9} + \dfrac{- 8}{8}=\dfrac{- 8 \times 8}{9 \times 8} + \dfrac{- 8 \times 9}{8 \times 9}=\dfrac{- 64}{72} + \dfrac{- 72}{72}=\dfrac{- 64 - 72}{72}=\dfrac{- 136}{72}
\]
\item
\[
\dfrac{9}{4} - 9=\dfrac{9}{4} + \dfrac{- 9}{1}=\dfrac{9}{4} + \dfrac{- 9 \times 4}{1 \times 4}=\dfrac{9}{4} + \dfrac{- 36}{4}=\dfrac{9 - 36}{4}=\dfrac{- 27}{4}
\]
\item
\[
\dfrac{- 2}{6} \times \dfrac{7}{5}=\dfrac{- 2 \times 7}{6 \times 5}=\dfrac{- 14}{30}
\]
\item
\[
\dfrac{7}{4} \times - 7=\dfrac{7 \times - 7}{4}=\dfrac{- 49}{4}
\]
\end{enumerate}
\end{solution}
\begin{exercise}[subtitle={Développer réduire}]
Développer puis réduire les expressions suivantes
\begin{multicols}{2}
\begin{enumerate}
\item $A = (7x + 8)(- 10x + 8)$
\item $B = (- 1x - 4)(6x - 4)$
\item $C = (8x + 10)^{2}$
\item $D = - 4 + x(5x - 6)$
\item $E = 2x^{2} + x(- 3x - 1)$
\item $F = - 9(x - 3)(x + 1)$
\end{enumerate}
\end{multicols}
\end{exercise}
\begin{solution}
\begin{enumerate}
\item
\begin{align*}
A &= (7x + 8)(- 10x + 8)\\&= 7x \times - 10x + 7x \times 8 + 8 \times - 10x + 8 \times 8\\&= 7 \times - 10 \times x^{1 + 1} + 8 \times 7 \times x + 8 \times - 10 \times x + 64\\&= 56x - 80x - 70x^{2} + 64\\&= (56 - 80) \times x - 70x^{2} + 64\\&= - 70x^{2} - 24x + 64
\end{align*}
\item
\begin{align*}
B &= (- 1x - 4)(6x - 4)\\&= - x \times 6x - x \times - 4 - 4 \times 6x - 4 \times - 4\\&= - 1 \times 6 \times x^{1 + 1} - 4 \times - 1 \times x - 4 \times 6 \times x + 16\\&= 4x - 24x - 6x^{2} + 16\\&= (4 - 24) \times x - 6x^{2} + 16\\&= - 6x^{2} - 20x + 16
\end{align*}
\item
\begin{align*}
C &= (8x + 10)^{2}\\&= (8x + 10)(8x + 10)\\&= 8x \times 8x + 8x \times 10 + 10 \times 8x + 10 \times 10\\&= 8 \times 8 \times x^{1 + 1} + 10 \times 8 \times x + 10 \times 8 \times x + 100\\&= 80x + 80x + 64x^{2} + 100\\&= (80 + 80) \times x + 64x^{2} + 100\\&= 64x^{2} + 160x + 100
\end{align*}
\item
\begin{align*}
D &= - 4 + x(5x - 6)\\&= - 4 + x \times 5x + x \times - 6\\&= 5x^{2} - 6x - 4
\end{align*}
\item
\begin{align*}
E &= 2x^{2} + x(- 3x - 1)\\&= 2x^{2} + x \times - 3x + x \times - 1\\&= 2x^{2} - 3x^{2} - x\\&= 2x^{2} - 3x^{2} - x\\&= (2 - 3) \times x^{2} - x\\&= - x^{2} - x
\end{align*}
\item
\begin{align*}
F &= - 9(x - 3)(x + 1)\\&= (- 9x - 9 \times - 3)(x + 1)\\&= (- 9x + 27)(x + 1)\\&= - 9x \times x - 9x \times 1 + 27x + 27 \times 1\\&= - 9x + 27 - 9x^{2} + 27x\\&= - 9x^{2} - 9x + 27x + 27\\&= - 9x^{2} + (- 9 + 27) \times x + 27\\&= - 9x^{2} + 18x + 27
\end{align*}
\end{enumerate}
\end{solution}
\begin{exercise}[subtitle={Étude de fonctions}]
Soit $f(x) = - 3x^{2} - 36x - 81$ une fonction définie sur $\R$.
\begin{enumerate}
\item Calculer les valeurs suivantes
\[
f(1) \qquad f(-2)
\]
\item Dériver la fonction $f$
\item Étudier le signe de $f'$ puis en déduire les variations de $f$.
\item Est-ce que $f$ admet un maximum? un minimum? Calculer sa valeur.
\end{enumerate}
\end{exercise}
\begin{solution}
\begin{enumerate}
\item On remplace $x$ par les valeurs demandées
\[
f(1) = - 3 \times 1^{2} - 36 \times 1 - 81=- 3 \times 1 - 36 - 81=- 3 - 117=- 120
\]
\[
f(-1) = - 3 \times - 1^{2} - 36 \times - 1 - 81=- 3 \times 1 + 36 - 81=- 3 - 45=- 48
\]
\item Pas de solutions automatiques.
\item Pas de solutions automatiques.
\end{enumerate}
\end{solution}
%\printsolutionstype{exercise}
\end{document}
%%% Local Variables:
%%% mode: latex
%%% TeX-master: "master"
%%% End:

View File

@ -0,0 +1,141 @@
\documentclass[a5paper,10pt]{article}
\usepackage{myXsim}
\usepackage{tasks}
% Title Page
\title{DM1 \hfill HENRIST Maxime}
\tribe{TST}
\date{Toussain 2020}
\begin{document}
\maketitle
\begin{exercise}[subtitle={Fractions}]
Faire les calculs avec les fraction suivants
\begin{multicols}{3}
\begin{enumerate}
\item $A = \dfrac{- 6}{7} - \dfrac{- 5}{7}$
\item $B = \dfrac{- 3}{2} - \dfrac{3}{12}$
\item $C = \dfrac{- 7}{4} + \dfrac{- 6}{3}$
\item $D = \dfrac{5}{5} + 9$
\item $E = \dfrac{- 9}{3} \times \dfrac{8}{2}$
\item $F = \dfrac{- 6}{10} \times - 1$
\end{enumerate}
\end{multicols}
\end{exercise}
\begin{solution}
\begin{enumerate}
\item
\[
\dfrac{- 6}{7} - \dfrac{- 5}{7}=\dfrac{- 6}{7} + \dfrac{5}{7}=\dfrac{- 6 + 5}{7}=\dfrac{- 1}{7}
\]
\item
\[
\dfrac{- 3}{2} - \dfrac{3}{12}=\dfrac{- 3}{2} - \dfrac{3}{12}=\dfrac{- 3 \times 6}{2 \times 6} - \dfrac{3}{12}=\dfrac{- 18}{12} - \dfrac{3}{12}=\dfrac{- 18 - 3}{12}=\dfrac{- 18 - 3}{12}=\dfrac{- 21}{12}
\]
\item
\[
\dfrac{- 7}{4} + \dfrac{- 6}{3}=\dfrac{- 7 \times 3}{4 \times 3} + \dfrac{- 6 \times 4}{3 \times 4}=\dfrac{- 21}{12} + \dfrac{- 24}{12}=\dfrac{- 21 - 24}{12}=\dfrac{- 45}{12}
\]
\item
\[
\dfrac{5}{5} + 9=\dfrac{5}{5} + \dfrac{9}{1}=\dfrac{5}{5} + \dfrac{9 \times 5}{1 \times 5}=\dfrac{5}{5} + \dfrac{45}{5}=\dfrac{5 + 45}{5}=\dfrac{50}{5}
\]
\item
\[
\dfrac{- 9}{3} \times \dfrac{8}{2}=\dfrac{- 9 \times 8}{3 \times 2}=\dfrac{- 72}{6}
\]
\item
\[
\dfrac{- 6}{10} \times - 1=\dfrac{- 6 \times - 1}{10}=\dfrac{6}{10}
\]
\end{enumerate}
\end{solution}
\begin{exercise}[subtitle={Développer réduire}]
Développer puis réduire les expressions suivantes
\begin{multicols}{2}
\begin{enumerate}
\item $A = (5x + 1)(2x + 1)$
\item $B = (- 4x + 8)(- 7x + 8)$
\item $C = (- 9x + 1)^{2}$
\item $D = 8 + x(- 8x + 7)$
\item $E = 3x^{2} + x(4x + 1)$
\item $F = - 8(x - 2)(x - 9)$
\end{enumerate}
\end{multicols}
\end{exercise}
\begin{solution}
\begin{enumerate}
\item
\begin{align*}
A &= (5x + 1)(2x + 1)\\&= 5x \times 2x + 5x \times 1 + 1 \times 2x + 1 \times 1\\&= 5 \times 2 \times x^{1 + 1} + 5x + 2x + 1\\&= 10x^{2} + 5x + 2x + 1\\&= 10x^{2} + (5 + 2) \times x + 1\\&= 10x^{2} + 7x + 1
\end{align*}
\item
\begin{align*}
B &= (- 4x + 8)(- 7x + 8)\\&= - 4x \times - 7x - 4x \times 8 + 8 \times - 7x + 8 \times 8\\&= - 4 \times - 7 \times x^{1 + 1} + 8 \times - 4 \times x + 8 \times - 7 \times x + 64\\&= - 32x - 56x + 28x^{2} + 64\\&= (- 32 - 56) \times x + 28x^{2} + 64\\&= 28x^{2} - 88x + 64
\end{align*}
\item
\begin{align*}
C &= (- 9x + 1)^{2}\\&= (- 9x + 1)(- 9x + 1)\\&= - 9x \times - 9x - 9x \times 1 + 1 \times - 9x + 1 \times 1\\&= - 9 \times - 9 \times x^{1 + 1} - 9x - 9x + 1\\&= 81x^{2} - 9x - 9x + 1\\&= 81x^{2} + (- 9 - 9) \times x + 1\\&= 81x^{2} - 18x + 1
\end{align*}
\item
\begin{align*}
D &= 8 + x(- 8x + 7)\\&= 8 + x \times - 8x + x \times 7\\&= - 8x^{2} + 7x + 8
\end{align*}
\item
\begin{align*}
E &= 3x^{2} + x(4x + 1)\\&= 3x^{2} + x \times 4x + x \times 1\\&= 3x^{2} + 4x^{2} + x\\&= 3x^{2} + 4x^{2} + x\\&= (3 + 4) \times x^{2} + x\\&= 7x^{2} + x
\end{align*}
\item
\begin{align*}
F &= - 8(x - 2)(x - 9)\\&= (- 8x - 8 \times - 2)(x - 9)\\&= (- 8x + 16)(x - 9)\\&= - 8x \times x - 8x \times - 9 + 16x + 16 \times - 9\\&= - 9 \times - 8 \times x - 144 - 8x^{2} + 16x\\&= 72x - 144 - 8x^{2} + 16x\\&= - 8x^{2} + 72x + 16x - 144\\&= - 8x^{2} + (72 + 16) \times x - 144\\&= - 8x^{2} + 88x - 144
\end{align*}
\end{enumerate}
\end{solution}
\begin{exercise}[subtitle={Étude de fonctions}]
Soit $f(x) = - 8x^{2} - 48x + 216$ une fonction définie sur $\R$.
\begin{enumerate}
\item Calculer les valeurs suivantes
\[
f(1) \qquad f(-2)
\]
\item Dériver la fonction $f$
\item Étudier le signe de $f'$ puis en déduire les variations de $f$.
\item Est-ce que $f$ admet un maximum? un minimum? Calculer sa valeur.
\end{enumerate}
\end{exercise}
\begin{solution}
\begin{enumerate}
\item On remplace $x$ par les valeurs demandées
\[
f(1) = - 8 \times 1^{2} - 48 \times 1 + 216=- 8 \times 1 - 48 + 216=- 8 + 168=160
\]
\[
f(-1) = - 8 \times - 1^{2} - 48 \times - 1 + 216=- 8 \times 1 + 48 + 216=- 8 + 264=256
\]
\item Pas de solutions automatiques.
\item Pas de solutions automatiques.
\end{enumerate}
\end{solution}
%\printsolutionstype{exercise}
\end{document}
%%% Local Variables:
%%% mode: latex
%%% TeX-master: "master"
%%% End:

View File

@ -0,0 +1,141 @@
\documentclass[a5paper,10pt]{article}
\usepackage{myXsim}
\usepackage{tasks}
% Title Page
\title{DM1 \hfill INFANTES Antoine}
\tribe{TST}
\date{Toussain 2020}
\begin{document}
\maketitle
\begin{exercise}[subtitle={Fractions}]
Faire les calculs avec les fraction suivants
\begin{multicols}{3}
\begin{enumerate}
\item $A = \dfrac{10}{4} - \dfrac{- 6}{4}$
\item $B = \dfrac{3}{6} - \dfrac{3}{54}$
\item $C = \dfrac{- 10}{10} + \dfrac{2}{9}$
\item $D = \dfrac{- 8}{8} + 1$
\item $E = \dfrac{- 3}{4} \times \dfrac{10}{3}$
\item $F = \dfrac{- 5}{7} \times - 6$
\end{enumerate}
\end{multicols}
\end{exercise}
\begin{solution}
\begin{enumerate}
\item
\[
\dfrac{10}{4} - \dfrac{- 6}{4}=\dfrac{10}{4} + \dfrac{6}{4}=\dfrac{10 + 6}{4}=\dfrac{16}{4}
\]
\item
\[
\dfrac{3}{6} - \dfrac{3}{54}=\dfrac{3}{6} - \dfrac{3}{54}=\dfrac{3 \times 9}{6 \times 9} - \dfrac{3}{54}=\dfrac{27}{54} - \dfrac{3}{54}=\dfrac{27 - 3}{54}=\dfrac{27 - 3}{54}=\dfrac{24}{54}
\]
\item
\[
\dfrac{- 10}{10} + \dfrac{2}{9}=\dfrac{- 10 \times 9}{10 \times 9} + \dfrac{2 \times 10}{9 \times 10}=\dfrac{- 90}{90} + \dfrac{20}{90}=\dfrac{- 90 + 20}{90}=\dfrac{- 70}{90}
\]
\item
\[
\dfrac{- 8}{8} + 1=\dfrac{- 8}{8} + \dfrac{1}{1}=\dfrac{- 8}{8} + \dfrac{1 \times 8}{1 \times 8}=\dfrac{- 8}{8} + \dfrac{8}{8}=\dfrac{- 8 + 8}{8}=\dfrac{0}{8}
\]
\item
\[
\dfrac{- 3}{4} \times \dfrac{10}{3}=\dfrac{- 3 \times 10}{4 \times 3}=\dfrac{- 30}{12}
\]
\item
\[
\dfrac{- 5}{7} \times - 6=\dfrac{- 5 \times - 6}{7}=\dfrac{30}{7}
\]
\end{enumerate}
\end{solution}
\begin{exercise}[subtitle={Développer réduire}]
Développer puis réduire les expressions suivantes
\begin{multicols}{2}
\begin{enumerate}
\item $A = (- 4x + 6)(- 5x + 6)$
\item $B = (8x + 1)(- 5x + 1)$
\item $C = (6x - 8)^{2}$
\item $D = - 8 + x(- 9x + 8)$
\item $E = - 3x^{2} + x(- 2x - 10)$
\item $F = 2(x + 6)(x + 9)$
\end{enumerate}
\end{multicols}
\end{exercise}
\begin{solution}
\begin{enumerate}
\item
\begin{align*}
A &= (- 4x + 6)(- 5x + 6)\\&= - 4x \times - 5x - 4x \times 6 + 6 \times - 5x + 6 \times 6\\&= - 4 \times - 5 \times x^{1 + 1} + 6 \times - 4 \times x + 6 \times - 5 \times x + 36\\&= - 24x - 30x + 20x^{2} + 36\\&= (- 24 - 30) \times x + 20x^{2} + 36\\&= 20x^{2} - 54x + 36
\end{align*}
\item
\begin{align*}
B &= (8x + 1)(- 5x + 1)\\&= 8x \times - 5x + 8x \times 1 + 1 \times - 5x + 1 \times 1\\&= 8 \times - 5 \times x^{1 + 1} + 8x - 5x + 1\\&= - 40x^{2} + 8x - 5x + 1\\&= - 40x^{2} + (8 - 5) \times x + 1\\&= - 40x^{2} + 3x + 1
\end{align*}
\item
\begin{align*}
C &= (6x - 8)^{2}\\&= (6x - 8)(6x - 8)\\&= 6x \times 6x + 6x \times - 8 - 8 \times 6x - 8 \times - 8\\&= 6 \times 6 \times x^{1 + 1} - 8 \times 6 \times x - 8 \times 6 \times x + 64\\&= - 48x - 48x + 36x^{2} + 64\\&= (- 48 - 48) \times x + 36x^{2} + 64\\&= 36x^{2} - 96x + 64
\end{align*}
\item
\begin{align*}
D &= - 8 + x(- 9x + 8)\\&= - 8 + x \times - 9x + x \times 8\\&= - 9x^{2} + 8x - 8
\end{align*}
\item
\begin{align*}
E &= - 3x^{2} + x(- 2x - 10)\\&= - 3x^{2} + x \times - 2x + x \times - 10\\&= - 3x^{2} - 2x^{2} - 10x\\&= - 3x^{2} - 2x^{2} - 10x\\&= (- 3 - 2) \times x^{2} - 10x\\&= - 5x^{2} - 10x
\end{align*}
\item
\begin{align*}
F &= 2(x + 6)(x + 9)\\&= (2x + 2 \times 6)(x + 9)\\&= (2x + 12)(x + 9)\\&= 2x \times x + 2x \times 9 + 12x + 12 \times 9\\&= 9 \times 2 \times x + 108 + 2x^{2} + 12x\\&= 18x + 108 + 2x^{2} + 12x\\&= 2x^{2} + 18x + 12x + 108\\&= 2x^{2} + (18 + 12) \times x + 108\\&= 2x^{2} + 30x + 108
\end{align*}
\end{enumerate}
\end{solution}
\begin{exercise}[subtitle={Étude de fonctions}]
Soit $f(x) = 3x^{2} - 24x + 48$ une fonction définie sur $\R$.
\begin{enumerate}
\item Calculer les valeurs suivantes
\[
f(1) \qquad f(-2)
\]
\item Dériver la fonction $f$
\item Étudier le signe de $f'$ puis en déduire les variations de $f$.
\item Est-ce que $f$ admet un maximum? un minimum? Calculer sa valeur.
\end{enumerate}
\end{exercise}
\begin{solution}
\begin{enumerate}
\item On remplace $x$ par les valeurs demandées
\[
f(1) = 3 \times 1^{2} - 24 \times 1 + 48=3 \times 1 - 24 + 48=3 + 24=27
\]
\[
f(-1) = 3 \times - 1^{2} - 24 \times - 1 + 48=3 \times 1 + 24 + 48=3 + 72=75
\]
\item Pas de solutions automatiques.
\item Pas de solutions automatiques.
\end{enumerate}
\end{solution}
%\printsolutionstype{exercise}
\end{document}
%%% Local Variables:
%%% mode: latex
%%% TeX-master: "master"
%%% End:

View File

@ -0,0 +1,141 @@
\documentclass[a5paper,10pt]{article}
\usepackage{myXsim}
\usepackage{tasks}
% Title Page
\title{DM1 \hfill MAGRO Robin}
\tribe{TST}
\date{Toussain 2020}
\begin{document}
\maketitle
\begin{exercise}[subtitle={Fractions}]
Faire les calculs avec les fraction suivants
\begin{multicols}{3}
\begin{enumerate}
\item $A = \dfrac{- 2}{6} - \dfrac{- 4}{6}$
\item $B = \dfrac{5}{6} - \dfrac{- 2}{48}$
\item $C = \dfrac{6}{9} + \dfrac{8}{8}$
\item $D = \dfrac{- 5}{3} - 10$
\item $E = \dfrac{- 9}{7} \times \dfrac{- 9}{6}$
\item $F = \dfrac{- 3}{6} \times - 1$
\end{enumerate}
\end{multicols}
\end{exercise}
\begin{solution}
\begin{enumerate}
\item
\[
\dfrac{- 2}{6} - \dfrac{- 4}{6}=\dfrac{- 2}{6} + \dfrac{4}{6}=\dfrac{- 2 + 4}{6}=\dfrac{2}{6}
\]
\item
\[
\dfrac{5}{6} - \dfrac{- 2}{48}=\dfrac{5}{6} + \dfrac{2}{48}=\dfrac{5 \times 8}{6 \times 8} + \dfrac{2}{48}=\dfrac{40}{48} + \dfrac{2}{48}=\dfrac{40 + 2}{48}=\dfrac{42}{48}
\]
\item
\[
\dfrac{6}{9} + \dfrac{8}{8}=\dfrac{6 \times 8}{9 \times 8} + \dfrac{8 \times 9}{8 \times 9}=\dfrac{48}{72} + \dfrac{72}{72}=\dfrac{48 + 72}{72}=\dfrac{120}{72}
\]
\item
\[
\dfrac{- 5}{3} - 10=\dfrac{- 5}{3} + \dfrac{- 10}{1}=\dfrac{- 5}{3} + \dfrac{- 10 \times 3}{1 \times 3}=\dfrac{- 5}{3} + \dfrac{- 30}{3}=\dfrac{- 5 - 30}{3}=\dfrac{- 35}{3}
\]
\item
\[
\dfrac{- 9}{7} \times \dfrac{- 9}{6}=\dfrac{- 9 \times - 9}{7 \times 6}=\dfrac{81}{42}
\]
\item
\[
\dfrac{- 3}{6} \times - 1=\dfrac{- 3 \times - 1}{6}=\dfrac{3}{6}
\]
\end{enumerate}
\end{solution}
\begin{exercise}[subtitle={Développer réduire}]
Développer puis réduire les expressions suivantes
\begin{multicols}{2}
\begin{enumerate}
\item $A = (3x + 4)(- 1x + 4)$
\item $B = (3x - 2)(- 3x - 2)$
\item $C = (- 4x - 2)^{2}$
\item $D = 5 + x(2x + 7)$
\item $E = - 8x^{2} + x(6x + 2)$
\item $F = - 2(x - 10)(x - 3)$
\end{enumerate}
\end{multicols}
\end{exercise}
\begin{solution}
\begin{enumerate}
\item
\begin{align*}
A &= (3x + 4)(- 1x + 4)\\&= 3x \times - x + 3x \times 4 + 4 \times - x + 4 \times 4\\&= 3 \times - 1 \times x^{1 + 1} + 4 \times 3 \times x + 4 \times - 1 \times x + 16\\&= 12x - 4x - 3x^{2} + 16\\&= (12 - 4) \times x - 3x^{2} + 16\\&= - 3x^{2} + 8x + 16
\end{align*}
\item
\begin{align*}
B &= (3x - 2)(- 3x - 2)\\&= 3x \times - 3x + 3x \times - 2 - 2 \times - 3x - 2 \times - 2\\&= 3 \times - 3 \times x^{1 + 1} - 2 \times 3 \times x - 2 \times - 3 \times x + 4\\&= - 6x + 6x - 9x^{2} + 4\\&= (- 6 + 6) \times x - 9x^{2} + 4\\&= 0x - 9x^{2} + 4\\&= - 9x^{2} + 4
\end{align*}
\item
\begin{align*}
C &= (- 4x - 2)^{2}\\&= (- 4x - 2)(- 4x - 2)\\&= - 4x \times - 4x - 4x \times - 2 - 2 \times - 4x - 2 \times - 2\\&= - 4 \times - 4 \times x^{1 + 1} - 2 \times - 4 \times x - 2 \times - 4 \times x + 4\\&= 8x + 8x + 16x^{2} + 4\\&= (8 + 8) \times x + 16x^{2} + 4\\&= 16x^{2} + 16x + 4
\end{align*}
\item
\begin{align*}
D &= 5 + x(2x + 7)\\&= 5 + x \times 2x + x \times 7\\&= 2x^{2} + 7x + 5
\end{align*}
\item
\begin{align*}
E &= - 8x^{2} + x(6x + 2)\\&= - 8x^{2} + x \times 6x + x \times 2\\&= - 8x^{2} + 6x^{2} + 2x\\&= - 8x^{2} + 6x^{2} + 2x\\&= (- 8 + 6) \times x^{2} + 2x\\&= - 2x^{2} + 2x
\end{align*}
\item
\begin{align*}
F &= - 2(x - 10)(x - 3)\\&= (- 2x - 2 \times - 10)(x - 3)\\&= (- 2x + 20)(x - 3)\\&= - 2x \times x - 2x \times - 3 + 20x + 20 \times - 3\\&= - 3 \times - 2 \times x - 60 - 2x^{2} + 20x\\&= 6x - 60 - 2x^{2} + 20x\\&= - 2x^{2} + 6x + 20x - 60\\&= - 2x^{2} + (6 + 20) \times x - 60\\&= - 2x^{2} + 26x - 60
\end{align*}
\end{enumerate}
\end{solution}
\begin{exercise}[subtitle={Étude de fonctions}]
Soit $f(x) = 6x^{2} - 150$ une fonction définie sur $\R$.
\begin{enumerate}
\item Calculer les valeurs suivantes
\[
f(1) \qquad f(-2)
\]
\item Dériver la fonction $f$
\item Étudier le signe de $f'$ puis en déduire les variations de $f$.
\item Est-ce que $f$ admet un maximum? un minimum? Calculer sa valeur.
\end{enumerate}
\end{exercise}
\begin{solution}
\begin{enumerate}
\item On remplace $x$ par les valeurs demandées
\[
f(1) = 6 \times 1^{2} - 150=6 \times 1 - 150=6 - 150=- 144
\]
\[
f(-1) = 6 \times - 1^{2} - 150=6 \times 1 - 150=6 - 150=- 144
\]
\item Pas de solutions automatiques.
\item Pas de solutions automatiques.
\end{enumerate}
\end{solution}
%\printsolutionstype{exercise}
\end{document}
%%% Local Variables:
%%% mode: latex
%%% TeX-master: "master"
%%% End:

View File

@ -0,0 +1,141 @@
\documentclass[a5paper,10pt]{article}
\usepackage{myXsim}
\usepackage{tasks}
% Title Page
\title{DM1 \hfill MORFIN Chloé}
\tribe{TST}
\date{Toussain 2020}
\begin{document}
\maketitle
\begin{exercise}[subtitle={Fractions}]
Faire les calculs avec les fraction suivants
\begin{multicols}{3}
\begin{enumerate}
\item $A = \dfrac{- 4}{10} - \dfrac{- 6}{10}$
\item $B = \dfrac{- 3}{4} - \dfrac{- 2}{12}$
\item $C = \dfrac{10}{3} + \dfrac{6}{2}$
\item $D = \dfrac{- 6}{6} + 2$
\item $E = \dfrac{- 8}{10} \times \dfrac{7}{9}$
\item $F = \dfrac{- 2}{3} \times - 6$
\end{enumerate}
\end{multicols}
\end{exercise}
\begin{solution}
\begin{enumerate}
\item
\[
\dfrac{- 4}{10} - \dfrac{- 6}{10}=\dfrac{- 4}{10} + \dfrac{6}{10}=\dfrac{- 4 + 6}{10}=\dfrac{2}{10}
\]
\item
\[
\dfrac{- 3}{4} - \dfrac{- 2}{12}=\dfrac{- 3}{4} + \dfrac{2}{12}=\dfrac{- 3 \times 3}{4 \times 3} + \dfrac{2}{12}=\dfrac{- 9}{12} + \dfrac{2}{12}=\dfrac{- 9 + 2}{12}=\dfrac{- 7}{12}
\]
\item
\[
\dfrac{10}{3} + \dfrac{6}{2}=\dfrac{10 \times 2}{3 \times 2} + \dfrac{6 \times 3}{2 \times 3}=\dfrac{20}{6} + \dfrac{18}{6}=\dfrac{20 + 18}{6}=\dfrac{38}{6}
\]
\item
\[
\dfrac{- 6}{6} + 2=\dfrac{- 6}{6} + \dfrac{2}{1}=\dfrac{- 6}{6} + \dfrac{2 \times 6}{1 \times 6}=\dfrac{- 6}{6} + \dfrac{12}{6}=\dfrac{- 6 + 12}{6}=\dfrac{6}{6}
\]
\item
\[
\dfrac{- 8}{10} \times \dfrac{7}{9}=\dfrac{- 8 \times 7}{10 \times 9}=\dfrac{- 56}{90}
\]
\item
\[
\dfrac{- 2}{3} \times - 6=\dfrac{- 2 \times - 6}{3}=\dfrac{12}{3}
\]
\end{enumerate}
\end{solution}
\begin{exercise}[subtitle={Développer réduire}]
Développer puis réduire les expressions suivantes
\begin{multicols}{2}
\begin{enumerate}
\item $A = (8x - 1)(8x - 1)$
\item $B = (- 2x + 3)(- 1x + 3)$
\item $C = (2x + 3)^{2}$
\item $D = - 9 + x(- 8x - 6)$
\item $E = - 6x^{2} + x(5x - 1)$
\item $F = 9(x - 3)(x + 5)$
\end{enumerate}
\end{multicols}
\end{exercise}
\begin{solution}
\begin{enumerate}
\item
\begin{align*}
A &= (8x - 1)(8x - 1)\\&= 8x \times 8x + 8x \times - 1 - 1 \times 8x - 1 \times - 1\\&= 8 \times 8 \times x^{1 + 1} - 1 \times 8 \times x - 1 \times 8 \times x + 1\\&= - 8x - 8x + 64x^{2} + 1\\&= (- 8 - 8) \times x + 64x^{2} + 1\\&= 64x^{2} - 16x + 1
\end{align*}
\item
\begin{align*}
B &= (- 2x + 3)(- 1x + 3)\\&= - 2x \times - x - 2x \times 3 + 3 \times - x + 3 \times 3\\&= - 2 \times - 1 \times x^{1 + 1} + 3 \times - 2 \times x + 3 \times - 1 \times x + 9\\&= - 6x - 3x + 2x^{2} + 9\\&= (- 6 - 3) \times x + 2x^{2} + 9\\&= 2x^{2} - 9x + 9
\end{align*}
\item
\begin{align*}
C &= (2x + 3)^{2}\\&= (2x + 3)(2x + 3)\\&= 2x \times 2x + 2x \times 3 + 3 \times 2x + 3 \times 3\\&= 2 \times 2 \times x^{1 + 1} + 3 \times 2 \times x + 3 \times 2 \times x + 9\\&= 6x + 6x + 4x^{2} + 9\\&= (6 + 6) \times x + 4x^{2} + 9\\&= 4x^{2} + 12x + 9
\end{align*}
\item
\begin{align*}
D &= - 9 + x(- 8x - 6)\\&= - 9 + x \times - 8x + x \times - 6\\&= - 8x^{2} - 6x - 9
\end{align*}
\item
\begin{align*}
E &= - 6x^{2} + x(5x - 1)\\&= - 6x^{2} + x \times 5x + x \times - 1\\&= - 6x^{2} + 5x^{2} - x\\&= - 6x^{2} + 5x^{2} - x\\&= (- 6 + 5) \times x^{2} - x\\&= - x^{2} - x
\end{align*}
\item
\begin{align*}
F &= 9(x - 3)(x + 5)\\&= (9x + 9 \times - 3)(x + 5)\\&= (9x - 27)(x + 5)\\&= 9x \times x + 9x \times 5 - 27x - 27 \times 5\\&= 5 \times 9 \times x - 135 + 9x^{2} - 27x\\&= 45x - 135 + 9x^{2} - 27x\\&= 9x^{2} + 45x - 27x - 135\\&= 9x^{2} + (45 - 27) \times x - 135\\&= 9x^{2} + 18x - 135
\end{align*}
\end{enumerate}
\end{solution}
\begin{exercise}[subtitle={Étude de fonctions}]
Soit $f(x) = 4x^{2} - 36x + 56$ une fonction définie sur $\R$.
\begin{enumerate}
\item Calculer les valeurs suivantes
\[
f(1) \qquad f(-2)
\]
\item Dériver la fonction $f$
\item Étudier le signe de $f'$ puis en déduire les variations de $f$.
\item Est-ce que $f$ admet un maximum? un minimum? Calculer sa valeur.
\end{enumerate}
\end{exercise}
\begin{solution}
\begin{enumerate}
\item On remplace $x$ par les valeurs demandées
\[
f(1) = 4 \times 1^{2} - 36 \times 1 + 56=4 \times 1 - 36 + 56=4 + 20=24
\]
\[
f(-1) = 4 \times - 1^{2} - 36 \times - 1 + 56=4 \times 1 + 36 + 56=4 + 92=96
\]
\item Pas de solutions automatiques.
\item Pas de solutions automatiques.
\end{enumerate}
\end{solution}
%\printsolutionstype{exercise}
\end{document}
%%% Local Variables:
%%% mode: latex
%%% TeX-master: "master"
%%% End:

View File

@ -0,0 +1,141 @@
\documentclass[a5paper,10pt]{article}
\usepackage{myXsim}
\usepackage{tasks}
% Title Page
\title{DM1 \hfill PERES RAMALHO Emeric}
\tribe{TST}
\date{Toussain 2020}
\begin{document}
\maketitle
\begin{exercise}[subtitle={Fractions}]
Faire les calculs avec les fraction suivants
\begin{multicols}{3}
\begin{enumerate}
\item $A = \dfrac{- 10}{5} - \dfrac{- 5}{5}$
\item $B = \dfrac{5}{3} - \dfrac{- 10}{6}$
\item $C = \dfrac{- 2}{8} + \dfrac{1}{7}$
\item $D = \dfrac{5}{10} - 1$
\item $E = \dfrac{9}{9} \times \dfrac{10}{8}$
\item $F = \dfrac{- 7}{7} \times 6$
\end{enumerate}
\end{multicols}
\end{exercise}
\begin{solution}
\begin{enumerate}
\item
\[
\dfrac{- 10}{5} - \dfrac{- 5}{5}=\dfrac{- 10}{5} + \dfrac{5}{5}=\dfrac{- 10 + 5}{5}=\dfrac{- 5}{5}
\]
\item
\[
\dfrac{5}{3} - \dfrac{- 10}{6}=\dfrac{5}{3} + \dfrac{10}{6}=\dfrac{5 \times 2}{3 \times 2} + \dfrac{10}{6}=\dfrac{10}{6} + \dfrac{10}{6}=\dfrac{10 + 10}{6}=\dfrac{20}{6}
\]
\item
\[
\dfrac{- 2}{8} + \dfrac{1}{7}=\dfrac{- 2 \times 7}{8 \times 7} + \dfrac{1 \times 8}{7 \times 8}=\dfrac{- 14}{56} + \dfrac{8}{56}=\dfrac{- 14 + 8}{56}=\dfrac{- 6}{56}
\]
\item
\[
\dfrac{5}{10} - 1=\dfrac{5}{10} + \dfrac{- 1}{1}=\dfrac{5}{10} + \dfrac{- 1 \times 10}{1 \times 10}=\dfrac{5}{10} + \dfrac{- 10}{10}=\dfrac{5 - 10}{10}=\dfrac{- 5}{10}
\]
\item
\[
\dfrac{9}{9} \times \dfrac{10}{8}=\dfrac{9 \times 10}{9 \times 8}=\dfrac{90}{72}
\]
\item
\[
\dfrac{- 7}{7} \times 6=\dfrac{- 7 \times 6}{7}=\dfrac{- 42}{7}
\]
\end{enumerate}
\end{solution}
\begin{exercise}[subtitle={Développer réduire}]
Développer puis réduire les expressions suivantes
\begin{multicols}{2}
\begin{enumerate}
\item $A = (4x - 9)(7x - 9)$
\item $B = (5x + 1)(9x + 1)$
\item $C = (6x + 6)^{2}$
\item $D = 8 + x(10x + 6)$
\item $E = - 9x^{2} + x(7x + 1)$
\item $F = 3(x + 6)(x - 5)$
\end{enumerate}
\end{multicols}
\end{exercise}
\begin{solution}
\begin{enumerate}
\item
\begin{align*}
A &= (4x - 9)(7x - 9)\\&= 4x \times 7x + 4x \times - 9 - 9 \times 7x - 9 \times - 9\\&= 4 \times 7 \times x^{1 + 1} - 9 \times 4 \times x - 9 \times 7 \times x + 81\\&= - 36x - 63x + 28x^{2} + 81\\&= (- 36 - 63) \times x + 28x^{2} + 81\\&= 28x^{2} - 99x + 81
\end{align*}
\item
\begin{align*}
B &= (5x + 1)(9x + 1)\\&= 5x \times 9x + 5x \times 1 + 1 \times 9x + 1 \times 1\\&= 5 \times 9 \times x^{1 + 1} + 5x + 9x + 1\\&= 45x^{2} + 5x + 9x + 1\\&= 45x^{2} + (5 + 9) \times x + 1\\&= 45x^{2} + 14x + 1
\end{align*}
\item
\begin{align*}
C &= (6x + 6)^{2}\\&= (6x + 6)(6x + 6)\\&= 6x \times 6x + 6x \times 6 + 6 \times 6x + 6 \times 6\\&= 6 \times 6 \times x^{1 + 1} + 6 \times 6 \times x + 6 \times 6 \times x + 36\\&= 36x + 36x + 36x^{2} + 36\\&= (36 + 36) \times x + 36x^{2} + 36\\&= 36x^{2} + 72x + 36
\end{align*}
\item
\begin{align*}
D &= 8 + x(10x + 6)\\&= 8 + x \times 10x + x \times 6\\&= 10x^{2} + 6x + 8
\end{align*}
\item
\begin{align*}
E &= - 9x^{2} + x(7x + 1)\\&= - 9x^{2} + x \times 7x + x \times 1\\&= - 9x^{2} + 7x^{2} + x\\&= - 9x^{2} + 7x^{2} + x\\&= (- 9 + 7) \times x^{2} + x\\&= - 2x^{2} + x
\end{align*}
\item
\begin{align*}
F &= 3(x + 6)(x - 5)\\&= (3x + 3 \times 6)(x - 5)\\&= (3x + 18)(x - 5)\\&= 3x \times x + 3x \times - 5 + 18x + 18 \times - 5\\&= - 5 \times 3 \times x - 90 + 3x^{2} + 18x\\&= - 15x - 90 + 3x^{2} + 18x\\&= 3x^{2} - 15x + 18x - 90\\&= 3x^{2} + (- 15 + 18) \times x - 90\\&= 3x^{2} + 3x - 90
\end{align*}
\end{enumerate}
\end{solution}
\begin{exercise}[subtitle={Étude de fonctions}]
Soit $f(x) = 6x^{2} + 54x - 60$ une fonction définie sur $\R$.
\begin{enumerate}
\item Calculer les valeurs suivantes
\[
f(1) \qquad f(-2)
\]
\item Dériver la fonction $f$
\item Étudier le signe de $f'$ puis en déduire les variations de $f$.
\item Est-ce que $f$ admet un maximum? un minimum? Calculer sa valeur.
\end{enumerate}
\end{exercise}
\begin{solution}
\begin{enumerate}
\item On remplace $x$ par les valeurs demandées
\[
f(1) = 6 \times 1^{2} + 54 \times 1 - 60=6 \times 1 + 54 - 60=6 - 6=0
\]
\[
f(-1) = 6 \times - 1^{2} + 54 \times - 1 - 60=6 \times 1 - 54 - 60=6 - 114=- 108
\]
\item Pas de solutions automatiques.
\item Pas de solutions automatiques.
\end{enumerate}
\end{solution}
%\printsolutionstype{exercise}
\end{document}
%%% Local Variables:
%%% mode: latex
%%% TeX-master: "master"
%%% End:

View File

@ -0,0 +1,141 @@
\documentclass[a5paper,10pt]{article}
\usepackage{myXsim}
\usepackage{tasks}
% Title Page
\title{DM1 \hfill RADOUAA Saleh}
\tribe{TST}
\date{Toussain 2020}
\begin{document}
\maketitle
\begin{exercise}[subtitle={Fractions}]
Faire les calculs avec les fraction suivants
\begin{multicols}{3}
\begin{enumerate}
\item $A = \dfrac{5}{10} - \dfrac{10}{10}$
\item $B = \dfrac{- 3}{2} - \dfrac{6}{14}$
\item $C = \dfrac{- 7}{3} + \dfrac{- 3}{2}$
\item $D = \dfrac{- 9}{7} + 2$
\item $E = \dfrac{- 3}{7} \times \dfrac{4}{6}$
\item $F = \dfrac{9}{3} \times - 8$
\end{enumerate}
\end{multicols}
\end{exercise}
\begin{solution}
\begin{enumerate}
\item
\[
\dfrac{5}{10} - \dfrac{10}{10}=\dfrac{5}{10} - \dfrac{10}{10}=\dfrac{5 - 10}{10}=\dfrac{5 - 10}{10}=\dfrac{- 5}{10}
\]
\item
\[
\dfrac{- 3}{2} - \dfrac{6}{14}=\dfrac{- 3}{2} - \dfrac{6}{14}=\dfrac{- 3 \times 7}{2 \times 7} - \dfrac{6}{14}=\dfrac{- 21}{14} - \dfrac{6}{14}=\dfrac{- 21 - 6}{14}=\dfrac{- 21 - 6}{14}=\dfrac{- 27}{14}
\]
\item
\[
\dfrac{- 7}{3} + \dfrac{- 3}{2}=\dfrac{- 7 \times 2}{3 \times 2} + \dfrac{- 3 \times 3}{2 \times 3}=\dfrac{- 14}{6} + \dfrac{- 9}{6}=\dfrac{- 14 - 9}{6}=\dfrac{- 23}{6}
\]
\item
\[
\dfrac{- 9}{7} + 2=\dfrac{- 9}{7} + \dfrac{2}{1}=\dfrac{- 9}{7} + \dfrac{2 \times 7}{1 \times 7}=\dfrac{- 9}{7} + \dfrac{14}{7}=\dfrac{- 9 + 14}{7}=\dfrac{5}{7}
\]
\item
\[
\dfrac{- 3}{7} \times \dfrac{4}{6}=\dfrac{- 3 \times 4}{7 \times 6}=\dfrac{- 12}{42}
\]
\item
\[
\dfrac{9}{3} \times - 8=\dfrac{9 \times - 8}{3}=\dfrac{- 72}{3}
\]
\end{enumerate}
\end{solution}
\begin{exercise}[subtitle={Développer réduire}]
Développer puis réduire les expressions suivantes
\begin{multicols}{2}
\begin{enumerate}
\item $A = (4x + 6)(10x + 6)$
\item $B = (7x + 5)(5x + 5)$
\item $C = (6x + 10)^{2}$
\item $D = 8 + x(9x + 5)$
\item $E = - 7x^{2} + x(- 8x - 4)$
\item $F = 1(x - 8)(x - 4)$
\end{enumerate}
\end{multicols}
\end{exercise}
\begin{solution}
\begin{enumerate}
\item
\begin{align*}
A &= (4x + 6)(10x + 6)\\&= 4x \times 10x + 4x \times 6 + 6 \times 10x + 6 \times 6\\&= 4 \times 10 \times x^{1 + 1} + 6 \times 4 \times x + 6 \times 10 \times x + 36\\&= 24x + 60x + 40x^{2} + 36\\&= (24 + 60) \times x + 40x^{2} + 36\\&= 40x^{2} + 84x + 36
\end{align*}
\item
\begin{align*}
B &= (7x + 5)(5x + 5)\\&= 7x \times 5x + 7x \times 5 + 5 \times 5x + 5 \times 5\\&= 7 \times 5 \times x^{1 + 1} + 5 \times 7 \times x + 5 \times 5 \times x + 25\\&= 35x + 25x + 35x^{2} + 25\\&= (35 + 25) \times x + 35x^{2} + 25\\&= 35x^{2} + 60x + 25
\end{align*}
\item
\begin{align*}
C &= (6x + 10)^{2}\\&= (6x + 10)(6x + 10)\\&= 6x \times 6x + 6x \times 10 + 10 \times 6x + 10 \times 10\\&= 6 \times 6 \times x^{1 + 1} + 10 \times 6 \times x + 10 \times 6 \times x + 100\\&= 60x + 60x + 36x^{2} + 100\\&= (60 + 60) \times x + 36x^{2} + 100\\&= 36x^{2} + 120x + 100
\end{align*}
\item
\begin{align*}
D &= 8 + x(9x + 5)\\&= 8 + x \times 9x + x \times 5\\&= 9x^{2} + 5x + 8
\end{align*}
\item
\begin{align*}
E &= - 7x^{2} + x(- 8x - 4)\\&= - 7x^{2} + x \times - 8x + x \times - 4\\&= - 7x^{2} - 8x^{2} - 4x\\&= - 7x^{2} - 8x^{2} - 4x\\&= (- 7 - 8) \times x^{2} - 4x\\&= - 15x^{2} - 4x
\end{align*}
\item
\begin{align*}
F &= 1(x - 8)(x - 4)\\&= (x - 8)(x - 4)\\&= x \times x + x \times - 4 - 8x - 8 \times - 4\\&= x^{2} + 32 + (- 4 - 8) \times x\\&= x^{2} - 12x + 32
\end{align*}
\end{enumerate}
\end{solution}
\begin{exercise}[subtitle={Étude de fonctions}]
Soit $f(x) = - 5x^{2} + 30x - 40$ une fonction définie sur $\R$.
\begin{enumerate}
\item Calculer les valeurs suivantes
\[
f(1) \qquad f(-2)
\]
\item Dériver la fonction $f$
\item Étudier le signe de $f'$ puis en déduire les variations de $f$.
\item Est-ce que $f$ admet un maximum? un minimum? Calculer sa valeur.
\end{enumerate}
\end{exercise}
\begin{solution}
\begin{enumerate}
\item On remplace $x$ par les valeurs demandées
\[
f(1) = - 5 \times 1^{2} + 30 \times 1 - 40=- 5 \times 1 + 30 - 40=- 5 - 10=- 15
\]
\[
f(-1) = - 5 \times - 1^{2} + 30 \times - 1 - 40=- 5 \times 1 - 30 - 40=- 5 - 70=- 75
\]
\item Pas de solutions automatiques.
\item Pas de solutions automatiques.
\end{enumerate}
\end{solution}
%\printsolutionstype{exercise}
\end{document}
%%% Local Variables:
%%% mode: latex
%%% TeX-master: "master"
%%% End:

View File

@ -0,0 +1,141 @@
\documentclass[a5paper,10pt]{article}
\usepackage{myXsim}
\usepackage{tasks}
% Title Page
\title{DM1 \hfill TAY Ummuhan}
\tribe{TST}
\date{Toussain 2020}
\begin{document}
\maketitle
\begin{exercise}[subtitle={Fractions}]
Faire les calculs avec les fraction suivants
\begin{multicols}{3}
\begin{enumerate}
\item $A = \dfrac{9}{6} - \dfrac{9}{6}$
\item $B = \dfrac{7}{6} - \dfrac{5}{30}$
\item $C = \dfrac{- 5}{9} + \dfrac{- 2}{8}$
\item $D = \dfrac{7}{6} - 1$
\item $E = \dfrac{- 1}{4} \times \dfrac{- 5}{3}$
\item $F = \dfrac{- 5}{10} \times 3$
\end{enumerate}
\end{multicols}
\end{exercise}
\begin{solution}
\begin{enumerate}
\item
\[
\dfrac{9}{6} - \dfrac{9}{6}=\dfrac{9}{6} - \dfrac{9}{6}=\dfrac{9 - 9}{6}=\dfrac{9 - 9}{6}=\dfrac{0}{6}
\]
\item
\[
\dfrac{7}{6} - \dfrac{5}{30}=\dfrac{7}{6} - \dfrac{5}{30}=\dfrac{7 \times 5}{6 \times 5} - \dfrac{5}{30}=\dfrac{35}{30} - \dfrac{5}{30}=\dfrac{35 - 5}{30}=\dfrac{35 - 5}{30}=\dfrac{30}{30}
\]
\item
\[
\dfrac{- 5}{9} + \dfrac{- 2}{8}=\dfrac{- 5 \times 8}{9 \times 8} + \dfrac{- 2 \times 9}{8 \times 9}=\dfrac{- 40}{72} + \dfrac{- 18}{72}=\dfrac{- 40 - 18}{72}=\dfrac{- 58}{72}
\]
\item
\[
\dfrac{7}{6} - 1=\dfrac{7}{6} + \dfrac{- 1}{1}=\dfrac{7}{6} + \dfrac{- 1 \times 6}{1 \times 6}=\dfrac{7}{6} + \dfrac{- 6}{6}=\dfrac{7 - 6}{6}=\dfrac{1}{6}
\]
\item
\[
\dfrac{- 1}{4} \times \dfrac{- 5}{3}=\dfrac{- 1 \times - 5}{4 \times 3}=\dfrac{5}{12}
\]
\item
\[
\dfrac{- 5}{10} \times 3=\dfrac{- 5 \times 3}{10}=\dfrac{- 15}{10}
\]
\end{enumerate}
\end{solution}
\begin{exercise}[subtitle={Développer réduire}]
Développer puis réduire les expressions suivantes
\begin{multicols}{2}
\begin{enumerate}
\item $A = (- 3x + 2)(9x + 2)$
\item $B = (- 4x - 8)(3x - 8)$
\item $C = (10x + 9)^{2}$
\item $D = 1 + x(8x + 10)$
\item $E = 4x^{2} + x(- 5x - 1)$
\item $F = 9(x + 4)(x - 3)$
\end{enumerate}
\end{multicols}
\end{exercise}
\begin{solution}
\begin{enumerate}
\item
\begin{align*}
A &= (- 3x + 2)(9x + 2)\\&= - 3x \times 9x - 3x \times 2 + 2 \times 9x + 2 \times 2\\&= - 3 \times 9 \times x^{1 + 1} + 2 \times - 3 \times x + 2 \times 9 \times x + 4\\&= - 6x + 18x - 27x^{2} + 4\\&= (- 6 + 18) \times x - 27x^{2} + 4\\&= - 27x^{2} + 12x + 4
\end{align*}
\item
\begin{align*}
B &= (- 4x - 8)(3x - 8)\\&= - 4x \times 3x - 4x \times - 8 - 8 \times 3x - 8 \times - 8\\&= - 4 \times 3 \times x^{1 + 1} - 8 \times - 4 \times x - 8 \times 3 \times x + 64\\&= 32x - 24x - 12x^{2} + 64\\&= (32 - 24) \times x - 12x^{2} + 64\\&= - 12x^{2} + 8x + 64
\end{align*}
\item
\begin{align*}
C &= (10x + 9)^{2}\\&= (10x + 9)(10x + 9)\\&= 10x \times 10x + 10x \times 9 + 9 \times 10x + 9 \times 9\\&= 10 \times 10 \times x^{1 + 1} + 9 \times 10 \times x + 9 \times 10 \times x + 81\\&= 90x + 90x + 100x^{2} + 81\\&= (90 + 90) \times x + 100x^{2} + 81\\&= 100x^{2} + 180x + 81
\end{align*}
\item
\begin{align*}
D &= 1 + x(8x + 10)\\&= 1 + x \times 8x + x \times 10\\&= 8x^{2} + 10x + 1
\end{align*}
\item
\begin{align*}
E &= 4x^{2} + x(- 5x - 1)\\&= 4x^{2} + x \times - 5x + x \times - 1\\&= 4x^{2} - 5x^{2} - x\\&= 4x^{2} - 5x^{2} - x\\&= (4 - 5) \times x^{2} - x\\&= - x^{2} - x
\end{align*}
\item
\begin{align*}
F &= 9(x + 4)(x - 3)\\&= (9x + 9 \times 4)(x - 3)\\&= (9x + 36)(x - 3)\\&= 9x \times x + 9x \times - 3 + 36x + 36 \times - 3\\&= - 3 \times 9 \times x - 108 + 9x^{2} + 36x\\&= - 27x - 108 + 9x^{2} + 36x\\&= 9x^{2} - 27x + 36x - 108\\&= 9x^{2} + (- 27 + 36) \times x - 108\\&= 9x^{2} + 9x - 108
\end{align*}
\end{enumerate}
\end{solution}
\begin{exercise}[subtitle={Étude de fonctions}]
Soit $f(x) = - 3x^{2} + 45x - 162$ une fonction définie sur $\R$.
\begin{enumerate}
\item Calculer les valeurs suivantes
\[
f(1) \qquad f(-2)
\]
\item Dériver la fonction $f$
\item Étudier le signe de $f'$ puis en déduire les variations de $f$.
\item Est-ce que $f$ admet un maximum? un minimum? Calculer sa valeur.
\end{enumerate}
\end{exercise}
\begin{solution}
\begin{enumerate}
\item On remplace $x$ par les valeurs demandées
\[
f(1) = - 3 \times 1^{2} + 45 \times 1 - 162=- 3 \times 1 + 45 - 162=- 3 - 117=- 120
\]
\[
f(-1) = - 3 \times - 1^{2} + 45 \times - 1 - 162=- 3 \times 1 - 45 - 162=- 3 - 207=- 210
\]
\item Pas de solutions automatiques.
\item Pas de solutions automatiques.
\end{enumerate}
\end{solution}
%\printsolutionstype{exercise}
\end{document}
%%% Local Variables:
%%% mode: latex
%%% TeX-master: "master"
%%% End:

View File

@ -0,0 +1,141 @@
\documentclass[a5paper,10pt]{article}
\usepackage{myXsim}
\usepackage{tasks}
% Title Page
\title{DM1 \hfill VIALON-DUPERRON Victorien}
\tribe{TST}
\date{Toussain 2020}
\begin{document}
\maketitle
\begin{exercise}[subtitle={Fractions}]
Faire les calculs avec les fraction suivants
\begin{multicols}{3}
\begin{enumerate}
\item $A = \dfrac{3}{5} - \dfrac{- 10}{5}$
\item $B = \dfrac{5}{7} - \dfrac{6}{14}$
\item $C = \dfrac{- 3}{5} + \dfrac{- 8}{4}$
\item $D = \dfrac{8}{8} + 10$
\item $E = \dfrac{7}{6} \times \dfrac{- 1}{5}$
\item $F = \dfrac{- 7}{10} \times 1$
\end{enumerate}
\end{multicols}
\end{exercise}
\begin{solution}
\begin{enumerate}
\item
\[
\dfrac{3}{5} - \dfrac{- 10}{5}=\dfrac{3}{5} + \dfrac{10}{5}=\dfrac{3 + 10}{5}=\dfrac{13}{5}
\]
\item
\[
\dfrac{5}{7} - \dfrac{6}{14}=\dfrac{5}{7} - \dfrac{6}{14}=\dfrac{5 \times 2}{7 \times 2} - \dfrac{6}{14}=\dfrac{10}{14} - \dfrac{6}{14}=\dfrac{10 - 6}{14}=\dfrac{10 - 6}{14}=\dfrac{4}{14}
\]
\item
\[
\dfrac{- 3}{5} + \dfrac{- 8}{4}=\dfrac{- 3 \times 4}{5 \times 4} + \dfrac{- 8 \times 5}{4 \times 5}=\dfrac{- 12}{20} + \dfrac{- 40}{20}=\dfrac{- 12 - 40}{20}=\dfrac{- 52}{20}
\]
\item
\[
\dfrac{8}{8} + 10=\dfrac{8}{8} + \dfrac{10}{1}=\dfrac{8}{8} + \dfrac{10 \times 8}{1 \times 8}=\dfrac{8}{8} + \dfrac{80}{8}=\dfrac{8 + 80}{8}=\dfrac{88}{8}
\]
\item
\[
\dfrac{7}{6} \times \dfrac{- 1}{5}=\dfrac{7 \times - 1}{6 \times 5}=\dfrac{- 7}{30}
\]
\item
\[
\dfrac{- 7}{10} \times 1=\dfrac{- 7}{10}
\]
\end{enumerate}
\end{solution}
\begin{exercise}[subtitle={Développer réduire}]
Développer puis réduire les expressions suivantes
\begin{multicols}{2}
\begin{enumerate}
\item $A = (- 2x + 1)(- 2x + 1)$
\item $B = (5x - 6)(10x - 6)$
\item $C = (5x - 6)^{2}$
\item $D = - 4 + x(9x - 2)$
\item $E = - 6x^{2} + x(8x - 10)$
\item $F = - 9(x - 5)(x + 1)$
\end{enumerate}
\end{multicols}
\end{exercise}
\begin{solution}
\begin{enumerate}
\item
\begin{align*}
A &= (- 2x + 1)(- 2x + 1)\\&= - 2x \times - 2x - 2x \times 1 + 1 \times - 2x + 1 \times 1\\&= - 2 \times - 2 \times x^{1 + 1} - 2x - 2x + 1\\&= 4x^{2} - 2x - 2x + 1\\&= 4x^{2} + (- 2 - 2) \times x + 1\\&= 4x^{2} - 4x + 1
\end{align*}
\item
\begin{align*}
B &= (5x - 6)(10x - 6)\\&= 5x \times 10x + 5x \times - 6 - 6 \times 10x - 6 \times - 6\\&= 5 \times 10 \times x^{1 + 1} - 6 \times 5 \times x - 6 \times 10 \times x + 36\\&= - 30x - 60x + 50x^{2} + 36\\&= (- 30 - 60) \times x + 50x^{2} + 36\\&= 50x^{2} - 90x + 36
\end{align*}
\item
\begin{align*}
C &= (5x - 6)^{2}\\&= (5x - 6)(5x - 6)\\&= 5x \times 5x + 5x \times - 6 - 6 \times 5x - 6 \times - 6\\&= 5 \times 5 \times x^{1 + 1} - 6 \times 5 \times x - 6 \times 5 \times x + 36\\&= - 30x - 30x + 25x^{2} + 36\\&= (- 30 - 30) \times x + 25x^{2} + 36\\&= 25x^{2} - 60x + 36
\end{align*}
\item
\begin{align*}
D &= - 4 + x(9x - 2)\\&= - 4 + x \times 9x + x \times - 2\\&= 9x^{2} - 2x - 4
\end{align*}
\item
\begin{align*}
E &= - 6x^{2} + x(8x - 10)\\&= - 6x^{2} + x \times 8x + x \times - 10\\&= - 6x^{2} + 8x^{2} - 10x\\&= - 6x^{2} + 8x^{2} - 10x\\&= (- 6 + 8) \times x^{2} - 10x\\&= 2x^{2} - 10x
\end{align*}
\item
\begin{align*}
F &= - 9(x - 5)(x + 1)\\&= (- 9x - 9 \times - 5)(x + 1)\\&= (- 9x + 45)(x + 1)\\&= - 9x \times x - 9x \times 1 + 45x + 45 \times 1\\&= - 9x + 45 - 9x^{2} + 45x\\&= - 9x^{2} - 9x + 45x + 45\\&= - 9x^{2} + (- 9 + 45) \times x + 45\\&= - 9x^{2} + 36x + 45
\end{align*}
\end{enumerate}
\end{solution}
\begin{exercise}[subtitle={Étude de fonctions}]
Soit $f(x) = 9x^{2} + 171x + 810$ une fonction définie sur $\R$.
\begin{enumerate}
\item Calculer les valeurs suivantes
\[
f(1) \qquad f(-2)
\]
\item Dériver la fonction $f$
\item Étudier le signe de $f'$ puis en déduire les variations de $f$.
\item Est-ce que $f$ admet un maximum? un minimum? Calculer sa valeur.
\end{enumerate}
\end{exercise}
\begin{solution}
\begin{enumerate}
\item On remplace $x$ par les valeurs demandées
\[
f(1) = 9 \times 1^{2} + 171 \times 1 + 810=9 \times 1 + 171 + 810=9 + 981=990
\]
\[
f(-1) = 9 \times - 1^{2} + 171 \times - 1 + 810=9 \times 1 - 171 + 810=9 + 639=648
\]
\item Pas de solutions automatiques.
\item Pas de solutions automatiques.
\end{enumerate}
\end{solution}
%\printsolutionstype{exercise}
\end{document}
%%% Local Variables:
%%% mode: latex
%%% TeX-master: "master"
%%% End:

View File

@ -0,0 +1,141 @@
\documentclass[a5paper,10pt]{article}
\usepackage{myXsim}
\usepackage{tasks}
% Title Page
\title{DM1 \hfill ZENAGUI Yanis}
\tribe{TST}
\date{Toussain 2020}
\begin{document}
\maketitle
\begin{exercise}[subtitle={Fractions}]
Faire les calculs avec les fraction suivants
\begin{multicols}{3}
\begin{enumerate}
\item $A = \dfrac{- 5}{5} - \dfrac{3}{5}$
\item $B = \dfrac{- 10}{8} - \dfrac{7}{24}$
\item $C = \dfrac{10}{9} + \dfrac{10}{8}$
\item $D = \dfrac{1}{10} - 6$
\item $E = \dfrac{5}{9} \times \dfrac{- 5}{8}$
\item $F = \dfrac{- 3}{3} \times - 9$
\end{enumerate}
\end{multicols}
\end{exercise}
\begin{solution}
\begin{enumerate}
\item
\[
\dfrac{- 5}{5} - \dfrac{3}{5}=\dfrac{- 5}{5} - \dfrac{3}{5}=\dfrac{- 5 - 3}{5}=\dfrac{- 5 - 3}{5}=\dfrac{- 8}{5}
\]
\item
\[
\dfrac{- 10}{8} - \dfrac{7}{24}=\dfrac{- 10}{8} - \dfrac{7}{24}=\dfrac{- 10 \times 3}{8 \times 3} - \dfrac{7}{24}=\dfrac{- 30}{24} - \dfrac{7}{24}=\dfrac{- 30 - 7}{24}=\dfrac{- 30 - 7}{24}=\dfrac{- 37}{24}
\]
\item
\[
\dfrac{10}{9} + \dfrac{10}{8}=\dfrac{10 \times 8}{9 \times 8} + \dfrac{10 \times 9}{8 \times 9}=\dfrac{80}{72} + \dfrac{90}{72}=\dfrac{80 + 90}{72}=\dfrac{170}{72}
\]
\item
\[
\dfrac{1}{10} - 6=\dfrac{1}{10} + \dfrac{- 6}{1}=\dfrac{1}{10} + \dfrac{- 6 \times 10}{1 \times 10}=\dfrac{1}{10} + \dfrac{- 60}{10}=\dfrac{1 - 60}{10}=\dfrac{- 59}{10}
\]
\item
\[
\dfrac{5}{9} \times \dfrac{- 5}{8}=\dfrac{5 \times - 5}{9 \times 8}=\dfrac{- 25}{72}
\]
\item
\[
\dfrac{- 3}{3} \times - 9=\dfrac{- 3 \times - 9}{3}=\dfrac{27}{3}
\]
\end{enumerate}
\end{solution}
\begin{exercise}[subtitle={Développer réduire}]
Développer puis réduire les expressions suivantes
\begin{multicols}{2}
\begin{enumerate}
\item $A = (10x - 6)(1x - 6)$
\item $B = (- 10x + 8)(- 8x + 8)$
\item $C = (8x + 2)^{2}$
\item $D = 8 + x(- 5x + 5)$
\item $E = 4x^{2} + x(7x + 10)$
\item $F = 8(x - 7)(x + 8)$
\end{enumerate}
\end{multicols}
\end{exercise}
\begin{solution}
\begin{enumerate}
\item
\begin{align*}
A &= (10x - 6)(1x - 6)\\&= 10x \times x + 10x \times - 6 - 6x - 6 \times - 6\\&= - 6 \times 10 \times x + 36 + 10x^{2} - 6x\\&= - 60x + 36 + 10x^{2} - 6x\\&= 10x^{2} - 60x - 6x + 36\\&= 10x^{2} + (- 60 - 6) \times x + 36\\&= 10x^{2} - 66x + 36
\end{align*}
\item
\begin{align*}
B &= (- 10x + 8)(- 8x + 8)\\&= - 10x \times - 8x - 10x \times 8 + 8 \times - 8x + 8 \times 8\\&= - 10 \times - 8 \times x^{1 + 1} + 8 \times - 10 \times x + 8 \times - 8 \times x + 64\\&= - 80x - 64x + 80x^{2} + 64\\&= (- 80 - 64) \times x + 80x^{2} + 64\\&= 80x^{2} - 144x + 64
\end{align*}
\item
\begin{align*}
C &= (8x + 2)^{2}\\&= (8x + 2)(8x + 2)\\&= 8x \times 8x + 8x \times 2 + 2 \times 8x + 2 \times 2\\&= 8 \times 8 \times x^{1 + 1} + 2 \times 8 \times x + 2 \times 8 \times x + 4\\&= 16x + 16x + 64x^{2} + 4\\&= (16 + 16) \times x + 64x^{2} + 4\\&= 64x^{2} + 32x + 4
\end{align*}
\item
\begin{align*}
D &= 8 + x(- 5x + 5)\\&= 8 + x \times - 5x + x \times 5\\&= - 5x^{2} + 5x + 8
\end{align*}
\item
\begin{align*}
E &= 4x^{2} + x(7x + 10)\\&= 4x^{2} + x \times 7x + x \times 10\\&= 4x^{2} + 7x^{2} + 10x\\&= 4x^{2} + 7x^{2} + 10x\\&= (4 + 7) \times x^{2} + 10x\\&= 11x^{2} + 10x
\end{align*}
\item
\begin{align*}
F &= 8(x - 7)(x + 8)\\&= (8x + 8 \times - 7)(x + 8)\\&= (8x - 56)(x + 8)\\&= 8x \times x + 8x \times 8 - 56x - 56 \times 8\\&= 8 \times 8 \times x - 448 + 8x^{2} - 56x\\&= 64x - 448 + 8x^{2} - 56x\\&= 8x^{2} + 64x - 56x - 448\\&= 8x^{2} + (64 - 56) \times x - 448\\&= 8x^{2} + 8x - 448
\end{align*}
\end{enumerate}
\end{solution}
\begin{exercise}[subtitle={Étude de fonctions}]
Soit $f(x) = - x^{2} + 100$ une fonction définie sur $\R$.
\begin{enumerate}
\item Calculer les valeurs suivantes
\[
f(1) \qquad f(-2)
\]
\item Dériver la fonction $f$
\item Étudier le signe de $f'$ puis en déduire les variations de $f$.
\item Est-ce que $f$ admet un maximum? un minimum? Calculer sa valeur.
\end{enumerate}
\end{exercise}
\begin{solution}
\begin{enumerate}
\item On remplace $x$ par les valeurs demandées
\[
f(1) = - 1 \times 1^{2} + 100=- 1 \times 1 + 100=- 1 + 100=99
\]
\[
f(-1) = - 1 \times - 1^{2} + 100=- 1 \times 1 + 100=- 1 + 100=99
\]
\item Pas de solutions automatiques.
\item Pas de solutions automatiques.
\end{enumerate}
\end{solution}
%\printsolutionstype{exercise}
\end{document}
%%% Local Variables:
%%% mode: latex
%%% TeX-master: "master"
%%% End:

Binary file not shown.

View File

@ -0,0 +1,155 @@
\documentclass[a5paper,10pt]{article}
\usepackage{myXsim}
\usepackage{tasks}
% Title Page
\title{DM1 \hfill \Var{Nom}}
\tribe{TST}
\date{Toussain 2020}
\begin{document}
\maketitle
\begin{exercise}[subtitle={Fractions}]
Faire les calculs avec les fraction suivants
\begin{multicols}{3}
\begin{enumerate}
%- set a = Expression.random("{a} / {b} - {c} / {b}", ["b > 1"])
\item $A = \Var{a}$
%- set b = Expression.random("{a} / {b} - {c} / {k*b}", ["b > 1", "k>1"])
\item $B = \Var{b}$
%- set c = Expression.random("{a} / {b} + {c} / {b-1}", ["b > 1"])
\item $C = \Var{c}$
%- set d = Expression.random("{a} / {b} + {c}", ["b > 1"])
\item $D = \Var{d}$
%- set e = Expression.random("{a} / {b} * {c} / {b-1}", ["b > 1"])
\item $E = \Var{e}$
%- set f = Expression.random("{a} / {b} * {c}", ["b > 1"])
\item $F = \Var{f}$
\end{enumerate}
\end{multicols}
\end{exercise}
\begin{solution}
\begin{enumerate}
\item
\[
\Var{a.simplify().explain() | join('=')}
\]
\item
\[
\Var{b.simplify().explain() | join('=')}
\]
\item
\[
\Var{c.simplify().explain() | join('=')}
\]
\item
\[
\Var{d.simplify().explain() | join('=')}
\]
\item
\[
\Var{e.simplify().explain() | join('=')}
\]
\item
\[
\Var{f.simplify().explain() | join('=')}
\]
\end{enumerate}
\end{solution}
\begin{exercise}[subtitle={Développer réduire}]
Développer puis réduire les expressions suivantes
\begin{multicols}{2}
\begin{enumerate}
%- set a = Expression.random("({a}x + {b})*({c}x + {b})")
\item $A = \Var{a}$
%- set b = Expression.random("({a}x + {b})*({c}x + {b})")
\item $B = \Var{b}$
%- set c = Expression.random("({a}x + {b})^2")
\item $C = \Var{c}$
%- set d = Expression.random("{c} + x*({a}x + {b})")
\item $D = \Var{d}$
%- set e = Expression.random("{c}*x^2 + x*({a}x + {b})")
\item $E = \Var{e}$
%- set f = Expression.random("{a}(x+{b})(x+{c})")
\item $F = \Var{f}$
\end{enumerate}
\end{multicols}
\end{exercise}
\begin{solution}
\begin{enumerate}
\item
\begin{align*}
A &= \Var{a.simplify().explain() | join('\\\\&= ')}
\end{align*}
\item
\begin{align*}
B &= \Var{b.simplify().explain() | join('\\\\&= ')}
\end{align*}
\item
\begin{align*}
C &= \Var{c.simplify().explain() | join('\\\\&= ')}
\end{align*}
\item
\begin{align*}
D &= \Var{d.simplify().explain() | join('\\\\&= ')}
\end{align*}
\item
\begin{align*}
E &= \Var{e.simplify().explain() | join('\\\\&= ')}
\end{align*}
\item
\begin{align*}
F &= \Var{f.simplify().explain() | join('\\\\&= ')}
\end{align*}
\end{enumerate}
\end{solution}
\begin{exercise}[subtitle={Étude de fonctions}]
%- set f = Expression.random("{a}(x-{b})(x-{c})")
Soit $f(x) = \Var{f.simplify()}$ une fonction définie sur $\R$.
\begin{enumerate}
\item Calculer les valeurs suivantes
\[
f(1) \qquad f(-2)
\]
\item Dériver la fonction $f$
\item Étudier le signe de $f'$ puis en déduire les variations de $f$.
\item Est-ce que $f$ admet un maximum? un minimum? Calculer sa valeur.
\end{enumerate}
\end{exercise}
\begin{solution}
\begin{enumerate}
\item On remplace $x$ par les valeurs demandées
\[
f(1) = \Var{f.simplify()(1).explain() | join('=')}
\]
\[
f(-1) = \Var{f.simplify()(-1).explain() | join('=')}
\]
\item Pas de solutions automatiques.
\item Pas de solutions automatiques.
\end{enumerate}
\end{solution}
%\printsolutionstype{exercise}
\end{document}
%%% Local Variables:
%%% mode: latex
%%% TeX-master: "master"
%%% End: